MyPasTest: MRCS A Online - Jan Exam 2015 03. Anatomy; Aabdomen, Pelvis & Perineum (290Qs) ------01. You are assisting in fitting an inferior vena cava (IVC) filter. The consultant asks you about the anatomy of the IVC. Which one of the following is true? Select one answer only. It commences at the level of L3« YOUR ANSWER It enters the thoracic cavity at the level of T8« CORRECT ANSWER It lies anterior to the aorta Page | It lies anterior to the caudate lobe of the 124 It lies to the left of the aorta.

The IVC commences opposite the L5 vertebra, formed by the confluence of the right and left common iliac veins. It runs on the right of the aorta upwards towards the diaphragm and extends to the central tendinous diaphragm at the level of the body of the T8 vertebra. The IVC lies posterior to the bare area and caudate lobe of the liver. Due to its compound embryological origin, the tributaries are not identical to the branches of the abdominal aorta. On the right side, the testicular vein drains into the inferior vena cava; but on the left, the testicular vein drains into the left renal vein.

02. A 72-year-old male is undergoing a coronary artery bypass grafting procedure. To allow access to the thorax, a median sternotomy is performed by the operating surgeon, disrupting the manubriosternal joint. What type of joint is this? Single best answer - select one answer only. Condylar« YOUR ANSWER Fibrous Primary cartilagenous Secondary cartilagenous« CORRECT ANSWER Synovial.

All joints in the body can be broadly divided into three types: fibrous, cartilaginous (primary and secondary) and synovial (typical and atypical). Fibrous joints unite the bones of the vault of the skull at the sutures; the bone ends are joined by fibrous tissue. Movement is negligible in fibrous joints.

All midline joints – symphysis pubis, manubriosternum, xiphisternum and intervertebral discs – are examples of secondary cartilaginous joints. Primary cartilaginous joints are formed when a bone unites with a cartilage. They are immobile and strong. All epiphyses and the attachments of ribs to their costal cartilages are examples of primary cartilaginous joints.

03. You are treating a patient with portal hypertension and request a scan to look for a portal vein thrombosis. Which of the following would help you locate the portal vein on the scan images? Single best answer question – choose ONE true option only. It arises at the union of the splenic and inferior mesenteric veins« YOUR ANSWER It lies directly anterior to the aorta It lies in the free edge of the lesser omentum« CORRECT ANSWER

------Dr Mohammed Shamsul Islam Khan, Medical Officer, Clinical Neuro-Surgery, National Institute of Neuro-Sciences and Hospital Sher-E-Bangla Nagar, Dhaka-1207, Bangladesh. Mobile: +880 1713 455 662, E-mail: [email protected]

MyPasTest: MRCS A Online - Jan Exam 2015 03. Anatomy; Aabdomen, Pelvis & Perineum (290Qs) ------It lies in front of the hepatic artery at the porta hepatis It receives the ligamentum teres at its right branch.

The portal vein is the continuation of the superior mesenteric vein after it has received the splenic vein behind the neck of the pancreas. It lies behind the bile duct and the hepatic artery. It lies anterior to the inferior vena cava. The ligamentum teres and the paraumbilical veins join Page | the left branch. 125

04. In locating the ureter on a plain abdominal x-ray, which landmark would you use to find it? It crosses the pelvic brim overlying the sacral foramina« YOUR ANSWER It extends inferiorly from the 12th thoracic vertebra It lies slightly medial to the tips of the transverse processes of the lumbar vertebrae « CORRECT ANSWER It passes towards the contralateral ischial spine It extends beyond the pubic tubercle.

The hila of the kidney lie at around the level of the first lumbar vertebra. The ureters extend inferiorly from here and are projected slightly medial to the tips of the transverse processes of the lumbar vertebrae. The ureters cross the pelvic brim at the sacroiliac joints, pass over the aforementioned joints and then towards the ischial spine and then passes towards (and not beyond, on radiographs) the pubic tubercle.

05. A 47-year-old female undergoes low anterior resection to treat a rectal adenocarcinoma. During the surgery, great care is taken to preserve the pelvic splanchnic nerves. From which nerve roots do these arise? Single best answer - select one answer only. L2-4« YOUR ANSWER L3-5 L4,5-S1 S1-3 S2-4« CORRECT ANSWER.

The pelvic plexuses supply the viscera of the . They are situated at the sides of the rectum in men and at the sides of the rectum and vagina in women. The pelvic plexuses are formed on either side by a continuation of the hypogastric plexus, by the sacral parasympathetic efferent fibers from the second, third and fourth sacral nerves and by a few filaments from the first two sacral ganglia.

In general, the muscles of the bladder (detrusor muscle) and rectum are innervated by the pelvic splanchnic nerves, the smooth muscle of the internal sphincter of the bladder through the superior hypogastric plexus and the smooth muscles of the internal sphincter of the anal canal by the branches from the sacral ganglia (which pass through the sacral plexuses). The pelvic splanchnic nerves supply the colon distal to the splenic flexure. Normal sensations of distension of bladder and rectum pass through the pelvic splanchnic nerves.

------Dr Mohammed Shamsul Islam Khan, Medical Officer, Clinical Neuro-Surgery, National Institute of Neuro-Sciences and Hospital Sher-E-Bangla Nagar, Dhaka-1207, Bangladesh. Mobile: +880 1713 455 662, E-mail: [email protected]

MyPasTest: MRCS A Online - Jan Exam 2015 03. Anatomy; Aabdomen, Pelvis & Perineum (290Qs) ------06. A 25-year-old is involved in a high speed road traffic accident and complains of left upper quadrant pain. After a normal primary survey and fluid resuscitation, he is deemed haemodynamically stable but complains of persistent left upper quadrant pain. Examination reveals a soft with moderate left upper quadrant tenderness, and no evidence of peritonism.What would be the most appropriate next step in his management? Continue the trauma management sequence« YOUR ANSWER Page | Diagnostic peritoneal lavage 126 Abdominal ultrasonography CT Scan of the abdomen with continued resuscitation« CORRECT ANSWER Emergency laparotomy.

The spleen should never normally be palpable even on deep inspiration. Mobilisation of the splenic flexure can cause troublesome fractures to the splenic capsule resulting in significant blood loss that can be difficult to stop. Occasionally, splenectomy may be required to stop this. The splenic artery is a direct branch of the coeliac plexus. The tail of pancreas abuts the splenic hilum and although classified as a „disaster‟ it is possible to cause irreparable damage to this area which contains the islet (B) cells that produce insulin. Appreciation of this anatomy is therefore essential.

The gastro-splenic and spleno-renal (Lieno renal) : the former carries the short gastric vessels that require ligation during splenectomy and gastrectomy. The scenario suggests splenic trauma. The mainstay of treatment for splenic trauma is conservation wherever possible. It would be important to confirm splenic injury and severity. The latter is described by the American Association for the Surgery of Trauma and is graded from 1 to 5. Each grade is subdivided into haematoma and laceration with grade 5 including vascular injury to the hilum.

Grades 1 to 4 can be treated non-operatively and approximately 65% of blunt splenic injuries can be treated non-operatively with a 98% success rate. Haematomas are usually subcapsular and lacerations involve the capsule and parenchyma. Grade 4 injuries involve segmental or hilar vessel lacerations with major devascularisation to > 25% of the spleen. Of the options offered in the question, the above-mentioned injuries are best delineated by CT scanning of the abdomen.

07. In a 25-year-old football player undergoing an repair, what would the surgeon note in order to identify the superficial inguinal ring? Single best answer - select one answer only. A V-shaped defect in the external oblique« YOUR ANSWER Intercrural fibres« CORRECT ANSWER The conjoint tendon and the transversalis fascia That the ilioinguinal nerve is within the spermatic cord That the inferior epigastric vessels are lateral to the deep inguinal ring

The inguinal canal is a short oblique passage through the anterior . The relatively weak transversalis fascia and the conjoint tendon form the posterior wall. The deep ring is a

------Dr Mohammed Shamsul Islam Khan, Medical Officer, Clinical Neuro-Surgery, National Institute of Neuro-Sciences and Hospital Sher-E-Bangla Nagar, Dhaka-1207, Bangladesh. Mobile: +880 1713 455 662, E-mail: [email protected]

MyPasTest: MRCS A Online - Jan Exam 2015 03. Anatomy; Aabdomen, Pelvis & Perineum (290Qs) ------hiatus in the transversalis fascia, with the inferior epigastric vessels lying immediately medial to it.

The superficial inguinal ring is a triangular hiatus in the external oblique aponeurosis. The visible landmark for the superficial inguinal ring are the intercrural fibres that run at right angles across the external oblique aponeurosis. Page | In females the inguinal canal carries the round of the and the ilioinguinal nerve. In males, the ilioinguinal nerve is carried in the inguinal canal but is not found within the 127 spermatic cord.

08. What is the normal location of the major duodenal papilla? Single best answer question – choose ONE true option only. Superior part of the duodenum« YOUR ANSWER Ascending part of the duodenum Descending part of the duodenum« CORRECT ANSWER Horizontal part of the duodenum Duodenal bulb.

The major duodenal papilla is situated at the medial side of the descending portion of the duodenum, a little below its middle and about 7 to 10 cm from the pylorus. The common bile and pancreatic ducts unite and open by a common orifice on the summit of the duodenal papilla.

09. In an Urology MDT, a pelvic MRI of a 45-year-old gentleman with HIV, presenting with acute on chronic urinary retention is discussed. It shows an irregular mass encasing the urethra within the deep perineal pouch. In relation to the deep perineal pouch, which is the most likely structure/tissue that the mass will invade? Single best answer - select one answer only. The bulbourethral glands« YOUR ANSWER The internal urethral sphincter The penile urethra The pudendal nerve The superficial perineal fascia.

In men, the deep perineal pouch contains the membranous urethra, sphincter urethra, bulbourethral glands, deep transverse perineal muscles, internal pudendal vessels and dorsal nerves of the penis. The bulb of the penis lies in the superficial perineal pouch. The superficial perineal fascia forms the boundary for the superficial perineal pouch. The bulbourthethral glands lie on each side of the membranous urethra.

10. A 45-year-old male with a history of gastro-oesophageal reflux disease undergoes laparoscopic reduction of hiatus hernia and fundoplication. During the procedure, dissection is performed to identify the boundaries of the oesophageal hiatus. What structure accompanies the oesophagus through this hiatus? Single best answer - select one answer only. Azygous vein« YOUR ANSWER

------Dr Mohammed Shamsul Islam Khan, Medical Officer, Clinical Neuro-Surgery, National Institute of Neuro-Sciences and Hospital Sher-E-Bangla Nagar, Dhaka-1207, Bangladesh. Mobile: +880 1713 455 662, E-mail: [email protected]

MyPasTest: MRCS A Online - Jan Exam 2015 03. Anatomy; Aabdomen, Pelvis & Perineum (290Qs) ------Inferior vena cava Phrenic nerves Thoracic duct Vagus nerves« CORRECT ANSWER. Page | The oesophagus passes through the diaphragm at the level of the T10 vetebrae, and is accompanied by the left and right branches of the vagus nerve. The aorta passes through at 128 T12, accompanied by the azygous vein and thoracic duct, and the IVC passes through at the level of T8 and is accompanied by the right phrenic nerve.

11. The blade of a retractor has rested on the psoas muscle during a lengthy operative procedure. This has resulted in a femoral nerve palsy. In the post-operative period the patient will experience: Single best answer question – choose ONE true option only. Inability to flex the knee only« YOUR ANSWER Inability to flex the knee and numbness over the thigh Numbness over the anterior thigh only Inability to extend the knee and numbness over the anterior thigh« CORRECT ANSWER Inability to flex the hip and numbness over the anterior thigh.

The femoral nerve is the largest branch of the lumbar plexus (L2-4). It forms in the abdomen within the substance of the psoas major muscle and descends posterolaterally through the pelvis to the midpoint of the inguinal ligament.

It supplies the anterior thigh muscles – the quadriceps group extend the leg at the knee, it also supplies other anterior thigh muscles - iliacus and sartorius which allow flexion of the thigh at the hip joint. The femoral nerve also gives several branches to the skin on the anteriomedial side of the lower limb.

12. You are treating a patient with an abdominal stab wound, which you suspect has lacerated the hepatic portal vein. Which of the following facts would allow you to locate the hepatic portal vein? Single best answer - choose ONE correct answer. It forms anterior to the neck of the pancreas« YOUR ANSWER It lies anterior to the first part of the duodenum It lies posterior to the hepatic artery proper« CORRECT ANSWER It lies posterior to the inferior vena cava It runs posterior to the epiploic foramen.

The portal vein forms the anterior boundary of the epiploic foramen, lying behind the bile duct and hepatic artery proper. It lies in front of the inferior vena cava, as it lies behind the pancreas and the first part of the duodenum. It is formed by the union of the splenic and superior mesenteric veins.

------Dr Mohammed Shamsul Islam Khan, Medical Officer, Clinical Neuro-Surgery, National Institute of Neuro-Sciences and Hospital Sher-E-Bangla Nagar, Dhaka-1207, Bangladesh. Mobile: +880 1713 455 662, E-mail: [email protected]

MyPasTest: MRCS A Online - Jan Exam 2015 03. Anatomy; Aabdomen, Pelvis & Perineum (290Qs) ------13. Following an elective splenectomy in a 21-year-old gentleman with Thalassaemia you are asked by the discharge co-ordinating team to review a discharge summary detailing the operative steps and subsequent treatment. Which of the following would be correct? Single best answer. The splenic artery was ligated after the splenic vein and lifelong oral Penicillin prophylaxis should not be offered to patients« YOUR ANSWER The splenic artery was ligated after the splenic vein and lifelong oral Penicillin prophylaxis should Page | be offered to patients 129 The splenic artery was ligated before the splenic vein and lifelong oral Penicillin prophylaxis should be offered to patients« CORRECT ANSWER The splenic artery was ligated before the splenic vein and lifelong oral Penicillin prophylaxis should not be offered to patients The splenic artery was ligated at the hilum and lifelong oral Penicillin prophylaxis should be offered to patients.

The spleen lies on the diaphragm, separated from the ninth to 11th ribs by the costodiaphragmatic recess. The left extremity of the lesser sac extends into the hilum. The peritoneal fold known as the lienorenal ligament (which contains the splenic vessels and the tail of the pancreas touching the spleen) passes from the left kidney to the spleen. Another peritoneal fold, the , carries the short gastric and gastroepiploic vessels and joins the spleen to the stomach. The spleen is not palpable in the healthy individual, and must be enlarged by two to four times its normal size before it appears in the left hypochondrium.

The short gastric vessels need to be divided. In this scenario, the Splenic artery is usually ligated along the upper border of the tail of the Pancreas. The artery is ligated before the vein to minimise splenic engorgement.

Current guidelines http://www.ncbi.nlm.nih.gov/pubmed/21988145 dictate that after splenectomy, lifelong prophylactic antibiotics should be offered to patients considered at continued high risk of pneumococcal infection using oral penicillins or macrolides (this advice is regularly reviewed in light of local pneumococcal resistance patterns).

Patients not at high risk should be counselled regarding the risks and benefits of lifelong antibiotics and may choose to discontinue them. All patients should carry a supply of appropriate antibiotics for emergency use.

14. A 23-year-old male presents with a unilateral testicular swelling and is diagnosed with a teratoma. He undergoes orchidectomy and makes a full recovery. During his regular follow-up, a CT scan of the abdomen and pelvis reveals lymph nodes of more than 1 cm in diameter. Which lymph nodes are likely to be enlarged? External iliac lymph nodes« YOUR ANSWER Inguinal lymph nodes Internal iliac lymph nodes Para-aortic lymph nodes« CORRECT ANSWER Sacral lymph nodes.

------Dr Mohammed Shamsul Islam Khan, Medical Officer, Clinical Neuro-Surgery, National Institute of Neuro-Sciences and Hospital Sher-E-Bangla Nagar, Dhaka-1207, Bangladesh. Mobile: +880 1713 455 662, E-mail: [email protected]

MyPasTest: MRCS A Online - Jan Exam 2015 03. Anatomy; Aabdomen, Pelvis & Perineum (290Qs) ------

The testes develop in the embryological abdomen, and so lymphatic drainage follows the route of the gonadal vessels to arrive at the para-aortic lymph nodes. The scrotum drains to the inguinal nodes.

15. During the course of an abdominal hysterectomy, you are asked to ligate and divide the uterine arteries. From which artery do they orginate? Single best answer - select one answer only. Page | Abdominal aorta« YOUR ANSWER 130 Common iliac artery External iliac artery Internal iliac artery« CORRECT ANSWER Median sacral artery.

The uterine arteries originate from the internal iliac artery. From there, they traverse the ureter and travel within the cardinal and broad ligaments before anastomosing with the ovarian artery.

16. During a laparoscopic low anterior resection for a mid-rectal neoplasm, the splenic flexure has to be mobilized to ensure a tension-free anastomosis. During dissection, the anatomical structure in relation to the body of the pancreas is which of the following? The greater curve of the stomach« YOUR ANSWER The inferior mesenteric vein The right kidney The transverse mesocolon« CORRECT ANSWER Lies in the transpyloric plane.

The head of the pancreas is related to the hilum but does not overlie the right kidney. It is, however, anterior to the left kidney. The transpyloric plane (L1) transects the pancreas obliquely, passing through the midpoint of the neck, with most of the head below the plane, and most of the body and tail above.

The transverse mesocolon is attached to the head, neck and body of the pancreas. The uncinate process lies posterior to the superior mesenteric vessels, and the inferior mesenteric vein passes behind the body of the pancreas, where it joins the splenic vein.

Entering the lesser sac between the transverse mesocolon and body of pancreas is a common method for laparoscopic mobilization of the splenic flexure.

17. A 48-year-old male presents with symptoms of fulminant hepatic failure secondary to alcohol abuse. He is noted to have hepatosplenomegaly, anorectal varices and caput medusa. Which anatomical structure is just anterior to the origin of the hepatic portal vein? Hepatic flexure« YOUR ANSWER Inferior vena cava Pancreatic body

------Dr Mohammed Shamsul Islam Khan, Medical Officer, Clinical Neuro-Surgery, National Institute of Neuro-Sciences and Hospital Sher-E-Bangla Nagar, Dhaka-1207, Bangladesh. Mobile: +880 1713 455 662, E-mail: [email protected]

MyPasTest: MRCS A Online - Jan Exam 2015 03. Anatomy; Aabdomen, Pelvis & Perineum (290Qs) ------Pancreatic neck« CORRECT ANSWER Pancreatic tail.

The hepatic portal vein is formed of the confluence of the splenic and superior mesenteric veins, just posterior to the pancreatic neck. It supplies 75% of all blood flow to the liver. The inferior vena cava lies posterior to its origin. Page |

18. A 25-year-old woman attends the Coloproctology clinic with a 4-month history of intermittent, 131 painful, fresh PR bleeding. She is found to have anal spasm and a chronic posterior anal fissure. What will be the most appropriate first line management? Single best answer - select one answer only. Advancement flap« YOUR ANSWER Examination under anaesthesia and Botulinum A toxin Flexible sigmoidoscopy and high fibre diet Lateral sphincterotomy 2% Diltiazem ointment« CORRECT ANSWER.

Anterior anal fissures are commoner in women due to the lack of support to the anal canal anteriorly. Recent studies have demonstrated good results with topical Diltiazem. Other options include lateral sphincterotomy, Botulinum toxin and advancement flap.

The latter is considered for low pressure fissures. The accepted treatment protocol for anal fissures is firstly topical 0.2% GTN or 2% Diltiazem followed by BOTOX and then lateral sphincterotomy or advancement flap. The latter is considered in women with low pressure anal fissure.

19. A 34-year-old man has been admitted following a stab wound to the right upper quadrant. During laparotomy profuse hepatic bleeding is encountered and your consultant performs Pringle‟s manoeuvre which is successful and then packs the abdomen. Which of the following is correct concerning the lesser sac and its related anatomy? Single best answer - select one answer only. Its left lateral border is formed by the « YOUR ANSWER Its anterior relation is the posterior aspect of the stomach« CORRECT ANSWER Its posterior relations include the spleen The aditus to the lesser sac is bounded anteriorly by the left free margin of the lesser omentum containing the portal vein, hepatic artery and bile duct The posterior wall is formed partly by the lesser omentum.

The lesser sac is a located behind the stomach. It may be entered by passing a finger into the epiploic foramen, immediately behind the portal vein in the free edge of the lesser omentum. The transverse mesocolon and pancreas form the posterior wall. The gastrosplenic and lienorenal ligaments form its left lateral border. The falciform ligament arises from the anterior abdominal wall and demarcates the left and the right lobe of the liver. ------Dr Mohammed Shamsul Islam Khan, Medical Officer, Clinical Neuro-Surgery, National Institute of Neuro-Sciences and Hospital Sher-E-Bangla Nagar, Dhaka-1207, Bangladesh. Mobile: +880 1713 455 662, E-mail: [email protected]

MyPasTest: MRCS A Online - Jan Exam 2015 03. Anatomy; Aabdomen, Pelvis & Perineum (290Qs) ------

Pringles manoeuvre involves compressing the structures in the free edge of the lesser omentum anterior to the aditus to the lesser sac. This is the right free margin rather than the left.

20. The psoas major muscle: Single best answer question – choose ONE true option only. Page | flexes the thigh at the hip joint« YOUR ANSWER 132 extends the thigh at the hip joint adducts the thigh at the hip joint abducts the thigh at the hip joint assists in the full contraction of the diaphragm.

The psoas major muscle joins the iliacus muscle which orginates broadly over the inner aspect of the iliac wing of the pelvis. This becomes the iliopsoas muscle and inserts on the lesser trochanter of the femur and thus flexes the thigh at the hip joint.

21. A man undergoes a at the beginning of surgery for cholecystectomy. He is found to have an incidental patent processus vaginalis. Which of the following is he predisposed to? Single best answer - select one answer only. Direct inguinal hernia « YOUR ANSWER Indirect inguinal hernia« CORRECT ANSWER Recurrent prostatitis Testicular torsion Undescended testis.

The processus vaginalis is a parietal peritoneal sac which passes through the internal ring of the inguinal canal in the fetus, but which is normally obliterated after birth except for a small part that becomes the tunica vaginalis of the testis.

The testis descends through the canal as a retroperitoneal structure and is therefore outside and behind the processus vaginalis. In cases of a persistent processus vaginalis, indirect inguinal hernias can ensue.

22. You are excising the right suprarenal gland and your consultant asks you about its anatomy. Which of the following descriptions is correct? Single best answer - select one correct answer. It is crescentic in shape« YOUR ANSWER It has a longer vein than the left suprarenal gland It lies anterior to the inferior vena cava It receives an arterial supply from the subcostal artery It touches the bare area of the liver« CORRECT ANSWER.

------Dr Mohammed Shamsul Islam Khan, Medical Officer, Clinical Neuro-Surgery, National Institute of Neuro-Sciences and Hospital Sher-E-Bangla Nagar, Dhaka-1207, Bangladesh. Mobile: +880 1713 455 662, E-mail: [email protected]

MyPasTest: MRCS A Online - Jan Exam 2015 03. Anatomy; Aabdomen, Pelvis & Perineum (290Qs) ------The left suprarenal vein is longer than the right, entering the left renal vein. Both glands receive an arterial supply directly from the aorta, as well as from the renal and inferior phrenic arteries. The right suprarenal gland is pyramidal in shape, the left is crescentic in shape. The anterior surface of the right suprarenal gland is overlapped medially by the inferior vena cava.

23. You are excising the right suprarenal gland and you are having difficulty locating it. Which anatomical considerations of the right suprarenal gland would assist you? Page | It drains into the right renal vein« YOUR ANSWER 133 It extends anterior to the inferior vena cava (IVC) It lies against the caudate lobe of the liver It lies on the ninth rib It receives blood from the right inferior phrenic artery« CORRECT ANSWER.

The bare area of the liver is in direct contact with the right suprarenal gland and the diaphragm. The right suprarenal gland extends medially behind the IVC, separated from the 12th rib by the diaphragm. It typically has three arterial supplies. It receives blood from the superior suprarenal artery (via the inferior phrenic artery), the middle suprarenal artery (directly via the abdominal aorta) and the inferior suprarenal artery (via the right renal artery). The venous drainage is into the IVC by a very short vessel. The left suprarenal gland drains into its corresponding renal vein.

24. A 50-year-old roofer attends the outpatient clinic with bilateral groin lumps which you decide are bilateral direct inguinal hernia. He asks your advice about the best repair. What would you recommend? Single best answer - select one answer only. Intraperitoneal repair« YOUR ANSWER Lichtenstein tension free repair Shouldice repair Totally extra-peritoneal repair« CORRECT ANSWER Trans-abdominal peritoneal repair.

Inguinal hernia may be repaired under local anaesthetic or general anaesthetic. Local anaesthetic techniques include Lichtenstein mesh repair or Shouldice repair. A herniotomy would not be appropriate in a man but would be suitable for repair of inguinal hernia in children less than 12 years old.

Laparoscopic intraperitoneal repair is an acceptable treatment, however, the long term results are unknown. The Bassini repair has fallen out of favour. The current favoured method is totally extraperitoneal (TEP repair). The recurrence rates are equivalent to Lichtenstein repair but analgesic requirements are less, return to work is quicker and chronic pain is infrequent.

25. While performing a hysterectomy the surgeon ligated the uterine artery on either side. The uterine artery arises from the: Single best answer question – choose ONE true option only. Abdominal aorta« YOUR ANSWER External iliac artery

------Dr Mohammed Shamsul Islam Khan, Medical Officer, Clinical Neuro-Surgery, National Institute of Neuro-Sciences and Hospital Sher-E-Bangla Nagar, Dhaka-1207, Bangladesh. Mobile: +880 1713 455 662, E-mail: [email protected]

MyPasTest: MRCS A Online - Jan Exam 2015 03. Anatomy; Aabdomen, Pelvis & Perineum (290Qs) ------Inferior rectal artery Internal iliac artery« CORRECT ANSWER Ovarian artery.

The uterine artery arises from the anterior division of the internal iliac artery and runs medially Page | on the levator ani toward the uterine cervix. It crosses above and in front of the ureter, to which it supplies a small branch, about 2 cm from the cervix. Reaching the side of the uterus, it 134 ascends in a tortuous manner between the two layers of the broad ligament to the junction of the Fallopian tube and uterus.

It then runs laterally toward the hilum of the and ends by joining with the ovarian artery. It supplies branches to the uterine cervix and others that descend on the vagina. The branches descending on the vagina anastomose with branches of the vaginal arteries and form with them two median longitudinal vessels, the azygos arteries of the vagina, one of which runs down in front of and the other behind the vagina.

It supplies numerous branches to the body of the uterus and from its terminal portion branches are distributed to the Fallopian tube and the round ligament of the uterus.

26. You are setting up monopolar diathermy at the start of an operation and your consultant asks you for some facts regarding the set up and use of monopolar surgical diathermy. How might you respond? It uses an alternating current of 400 Hz« YOUR ANSWER It requires a patient plate electrode of at least 20 cm2 It produces a local heating effect up to 1000 °C« CORRECT ANSWER It cannot be used in patients with pacemakers It produces burn injuries that are usually partial thickness.

Monopolar diathermy uses an alternating current at a frequency of 400kHz to 10 MHz. Current passes down the diathermy forceps, which may be applied to surgical forceps holding tissue, causing a local heating effect up to 1000oC through the patient plate electrode. The patient plate electrode must be at least 70 cm2 in size.

Monopolar diathermy may be used in patients with a pacemaker, but should be used in short bursts of less than 2 seconds, and the diathermy circuit should be away from the site of the pacemaker. It is preferable to use bipolar diathermy in these patients. Diathermy burns are usually full thickness.

27. What is the mortality rate associated with ruptured abdominal aortic aneurysms (AAA) presenting to hospital? Single best answer - select one answer only. 30%« YOUR ANSWER 40% 50%« CORRECT ANSWER 60%

------Dr Mohammed Shamsul Islam Khan, Medical Officer, Clinical Neuro-Surgery, National Institute of Neuro-Sciences and Hospital Sher-E-Bangla Nagar, Dhaka-1207, Bangladesh. Mobile: +880 1713 455 662, E-mail: [email protected]

MyPasTest: MRCS A Online - Jan Exam 2015 03. Anatomy; Aabdomen, Pelvis & Perineum (290Qs) ------70%.

The mortality rate of ruptured abdominal aortic aneurysm (AAA) presenting to hospital is 50%. The overall mortality from ruptured AAA is 75%. Size is the most important risk factor for rupture, although rate of expansion is also significant. The patient typically presents with a hypotensive episode or collapse associated with severe central abdominal pain radiating to the Page | back and the flanks. 135

28. Concerning the uterus: Single best answer question – choose ONE true option only. The uterine wall consists of two layers: the myometrium and the endometrium« YOUR ANSWER The uterus is frequently retroverted in older women« CORRECT ANSWER The body of the uterus is enclosed between the layers of the round ligament The principal support of the uterus is the covers only the superior aspect of the uterus.

The uterus is normally bent anteriorly between the cervix and the body of the uterus, and the entire uterus is as a general rule inclined anteriorly (anteverted). However, the uterus is frequently retroverted (inclined posteriorly) in older women.

The wall of the uterus consists of three layers: the perimetrium, the myometrium, and the endometrium. The body of the uterus is enclosed between the layers of the broad ligament and is freely mobile. The uterus is covered by peritoneum anteriorly and superiorly except for the vaginal part of the cervix, and the principal support of the uterus is the .

29. Exomphalos is associated with all of the following, except: Single best answer - select one answer only. Beckwith–Wiedemann syndrome« YOUR ANSWER Cardiac malformations Edward syndrome Patau syndrome Potter syndrome« CORRECT ANSWER.

Exomphalos is the failure of closure of the abdomen at the umbilical ring. Incidence is 1 in 5,000 to 1 in 10,000 live births although the trend is decreasing (partly related to termination of pregnancy following identification of other lethal structural or chromosomal abnormalities.

The incidence of associated abnormalities is between 40% - 70%, including chromosonal (trisomies 13, 18 and 21), cardiac, genitourinary, gastrointestinal, craniofacial, and pulmonary hypoplasia. Syndromes associated include Beckwith–Wiedemann syndrome, Edward syndrome and Patau syndrome. Potter syndrome or sequence relates to renal abnormalities. Oligohydramnios as a result of renal abnormalities, urinary tract obstruction or amniotic fluid leakage may lead to fetal

------Dr Mohammed Shamsul Islam Khan, Medical Officer, Clinical Neuro-Surgery, National Institute of Neuro-Sciences and Hospital Sher-E-Bangla Nagar, Dhaka-1207, Bangladesh. Mobile: +880 1713 455 662, E-mail: [email protected]

MyPasTest: MRCS A Online - Jan Exam 2015 03. Anatomy; Aabdomen, Pelvis & Perineum (290Qs) ------compression with joint contractures (arthrogryposis), pulmonary hypoplasia and squashed facies.

30. An 18-month-old child is brought to see you in clinic as his mother has noted mucus and blood staining on nappies, and has seen a bright red mass intermittently protruding from the anus. You diagnose a partial rectal prolapse. Which of the following is true regarding partial rectal prolapses? Select one answer only. Page | It involves all layers of the rectal wall« YOUR ANSWER 136 It is associated with a significant increased risk of rectal carcinoma It is more common in adults than children It should be treated with DeLorme’s procedure The majority resolve with conservative treatment« CORRECT ANSWER.

Complete rectal prolapse involves all layers of the rectal wall and is more commonly seen in elderly women. Partial rectal prolapse is more common in children and involves the mucosal layer only. The majority of cases of partial rectal prolapse in children resolve with conservative treatment.

DeLorme‟s procedure is excision of the mucosal component of a prolapse with plication of the muscle from below, and is generally used in adults. There is an associated increased risk of rectal carcinoma in adults with complete rectal prolapse, but this risk has not been reported in children with partial rectal prolapse.

31. Which one of the following structures is not encountered during a right hemicolectomy? Single best answer question – choose ONE true option only. The right ureter« YOUR ANSWER The right gonadal artery The inferior mesenteric artery« CORRECT ANSWER The second part of the duodenum The middle colic artery.

The inferior mesenteric artery supplies the large bowel from the transverse colon distally and is not encountered in a right hemicolectomy which involves excision of the caecum, ascending colon and the middle third of the transverse colon. The transverse colon crosses the second part of the duodenum.

The right ureter and gonadal vessels although both retroperitoneal, are both posteriorly related to the ascending colon and are easily injured during dissection. The middle colic artery is the terminal branch of the superior mesenteric artery and it‟s right branch is usually included in the excision during a right hemicolectomy.

32. Which of the following best describes the abdominal aorta?

------Dr Mohammed Shamsul Islam Khan, Medical Officer, Clinical Neuro-Surgery, National Institute of Neuro-Sciences and Hospital Sher-E-Bangla Nagar, Dhaka-1207, Bangladesh. Mobile: +880 1713 455 662, E-mail: [email protected]

MyPasTest: MRCS A Online - Jan Exam 2015 03. Anatomy; Aabdomen, Pelvis & Perineum (290Qs) ------Bifurcates at the level of the second lumbar vertebra« YOUR ANSWER Gives off the gonadal arteries inferior to the origin of the inferior mesenteric artery Is crossed by the left renal vein inferior to the origin of the superior mesenteric artery« CORRECT ANSWER Pierces the diaphragm at the level of the tenth thoracic vertebra Page | Supplies midgut derivatives via the coeliac axis. 137

The abdominal aorta is a continuation of the descending thoracic aorta, passing between the crura of the diaphragm at T12 level. The oesophageal opening in the diaphragm lies at T10. Having passed through the diaphragm the aorta almost immediately gives off the coeliac axis to supply the embryological foregut derivatives.

Its next ventral branch is the superior mesenteric artery and inferior to this passes posterior to the left renal vein. The paired gonadal vessels arise superior to the origin of the inferior mesenteric artery. The aorta then bifurcates into the common iliac arteries at the level of the fourth lumbar vertebra.

33. A 17-year-old male with a six-month history of diarrhoea and weight loss presents with fever, malaise and severe anorectal pain and swelling. What is the most likely diagnosis? Single best answer - select one answer only. Intersphincteric abscess« YOUR ANSWER Ischiorectal abscess Necrotising fasciitis Perianal abscess« CORRECT ANSWER Supralevator abscess.

The commonest type of anorectal sepsis is a perianal abscess (42%). Ischiorectal and intersphincteric abscesses are the next most common (22% each). Supralevator abscesses can develop from abdominopelvic pathology, and can produce lower abdominal or rectal pain.

Crohn‟s disease is associated with an increased incidence of anorectal sepsis. If untreated, perianal sepsis can lead to necrotising fasciitis, and so prompt incision and drainage of abscesses is required.

34. A 22-year-old man is stabbed in the right side of his abdomen sustaining a laceration to the upper part of the portal vein close to the liver. He is brought to A&E in hypovolaemic shock. Where is the surgeon most likely to gain access to the portal vein initially? Anterior to the hepatic artery and common bile duct« YOUR ANSWER Behind the body of the pancreas Inside the Inside the lesser sac On the right, free edge of the lesser omentum« CORRECT ANSWER.

------Dr Mohammed Shamsul Islam Khan, Medical Officer, Clinical Neuro-Surgery, National Institute of Neuro-Sciences and Hospital Sher-E-Bangla Nagar, Dhaka-1207, Bangladesh. Mobile: +880 1713 455 662, E-mail: [email protected]

MyPasTest: MRCS A Online - Jan Exam 2015 03. Anatomy; Aabdomen, Pelvis & Perineum (290Qs) ------The portal vein is formed by the confluence of the superior mesenteric vein (lying to the right of the artery) and the splenic vein, behind the neck of the pancreas. It is about 5 cm long. The portal vein lies between the two layers of the lesser omentum, running almost vertically in the free edge where the lesser omentum forms the anterior boundary of the epiploic foramen.

The portal vein is found behind the hepatic artery and common bile duct. The terminals of the Page | portal vein and the hepatic artery form, with the hepatic ductules, the triads of the liver in the corners of the hexagonal lobules. 138

The central veins drain into the hepatic veins. In trauma scenarios, liver haemorrhages can be controlled by temporary compression of the portal vein and hepatic artery in the edge of the lesser omentum (hepato-duodenal ligament). This is known as the Pringle manoeuver.

35. A 42-year-old man is stabbed in the right upper quadrant of his abdomen. During the subsequent emergency laparotomy the surgeon clamps the free edge of the lesser omentum in order to achieve rapid haemostasis. Which artery has been occluded? Common hepatic artery« YOUR ANSWER Cystic artery Gastroduodenal artery Hepatic artery « CORRECT ANSWER Right gastric artery.

This haemostatic procedure is known as Pringle‟s manoeuvre, and is intended to arrest major haemorrhage from the liver in an emergency situation. The surgeon clamps or manually compresses the free edge of the lesser omentum, which contains the . This ligament contains the hepatic portal vein, the common bile duct and the Hepatic artery.

The Common Hepatic artery arises from the Coeliac artery and curves upwards into the free edge of the lesser omentum to become the Hepatic artery. The Hepatic artery lies in-front of the Portal vein and to the left of the bile duct. The common hepatic artery gives of the Right Gastric artery as it turns into the lesser omentum and the Right Gastroduodenal artery which, passes down behind the first part of the duodenum.

36. To achieve temporary haemostasis during a trauma laparotomy for liver injury, Pringle‟s manoeuvre can be performed by manually compressing the free edge of the lesser omentum. Which vein is compressed during this manoeuvre? Gastroduodenal vein« YOUR ANSWER Hepatic vein Portal vein « CORRECT ANSWER Splenic vein Superior mesenteric vein.

------Dr Mohammed Shamsul Islam Khan, Medical Officer, Clinical Neuro-Surgery, National Institute of Neuro-Sciences and Hospital Sher-E-Bangla Nagar, Dhaka-1207, Bangladesh. Mobile: +880 1713 455 662, E-mail: [email protected]

MyPasTest: MRCS A Online - Jan Exam 2015 03. Anatomy; Aabdomen, Pelvis & Perineum (290Qs) ------Pringle‟s manoeuvre is the compression of the hepatoduodenal ligament to achieve hameostasis in liver injury. The ligament contains the hepatic artery, the common bile duct and the hepatic portal vein, and lies in the free edge of the lesser omentum, anterior to the epiploic foramen. The portal vein is formed behind the pancreatic neck by the confluence of the splenic and superior mesenteric veins.

37. A 31-year-old intravenous drug user presents to A&E with a pseudoaneurysm of the right Page | femoral artery caused by repeated injections in the groin. He requires urgent surgical intervention. Which of the following describes the course of the femoral artery in the leg most 139 accurately? The femoral artery has no branch in the adductor canal« YOUR ANSWER The femoral artery is anterior to the vein in the adductor canal« CORRECT ANSWER The femoral artery is posterior to the adductor brevis The femoral artery is posterior to the adductor longus The femoral artery is posterior to the psoas muscle.

The femoral artery enters the thigh halfway between the anterior superior iliac spine and the pubic symphysis. Here the femoral artery lies on the psoas major tendon, which separates it from the capsule of the hip joint. The femoral artery emerges from the femoral canal and then courses downwards to disappear beneath the sartorius, entering the adductor canal.

Only the profunda femoris spirals deep to the adductor longus. At the adductor hiatus, the femoral vein is lateral but ascends posteriorly in the canal until it reaches the femoral triangle, where it lies medial to the femoral artery. The femoral artery gives off a descending genicular artery while in the adductor canal.

38. A 23–year-old male presents with severe perianal swelling and discomfort. A magnetic resonance imaging scan of his pelvis reveals an abscess in the ischiorectal fossa. Which muscle lies superior and medial to his abscess? Bulbospongiosus« YOUR ANSWER Levator ani« CORRECT ANSWER Obturator internus Sphincter ani internus Transversus perinei superficialis.

The ischiorectal fossa is a prism-shaped potential space adjacent with its inferior aspect formed by perineal skin. Levator ani lies medial and superior to the fossa, which is also bordered by gluteus maximus (posterior), ischial tuberosity and obturator internus (lateral) and transversus perinea superficialis (anterior).

39. A 35-year-old with a history of alcohol abuse presents with torrential upper gastrointestinal haemorrhage. He is resuscitated, undergoes emergency oesophagogastroduodenoscopy, banding of oesophageal varices and is transferred to the high dependency unit. Which anatomical structure lies anterior to the origin of the hepatic portal vein? ------Dr Mohammed Shamsul Islam Khan, Medical Officer, Clinical Neuro-Surgery, National Institute of Neuro-Sciences and Hospital Sher-E-Bangla Nagar, Dhaka-1207, Bangladesh. Mobile: +880 1713 455 662, E-mail: [email protected]

MyPasTest: MRCS A Online - Jan Exam 2015 03. Anatomy; Aabdomen, Pelvis & Perineum (290Qs) ------Body of the pancreas« YOUR ANSWER Fourth part of the duodenum Gastric pylorus Head of the pancreas Page | Neck of the pancreas« CORRECT ANSWER. 140 The hepatic portal vein is formed of the confluence of the splenic and superior mesenteric veins, just posterior to the neck of the pancreas. Portal hypertension arises secondary to hepatic pathology, and causes porto-sytemic venous shunts. This leads to the development of oesophageal varices which can rupture and cause torrential, often fatal, haemorrhage.

40. A 36-year-old male presents with a history of alcohol abuse and insulin-dependent diabetes mellitus. He presents with septic shock and is noted to have a huge ischiorectal abscess. He undergoes emergency incision and drainage but displays symptoms of faecal incontinence in the post-operative period. Which nerve supplies fibres to the external anal sphincter? Single best answer - select one answer only. Genitofemoral nerve« YOUR ANSWER Obturator nerve Post-ganglionic parasympathetic nerves Pudendal nerve« CORRECT ANSWER Sympathetic nerves.

The external anal sphincter is a voluntary muscle innervated by the somatic inferior rectal nerves, originating from the pudendal nerve. The internal anal sphincter is involuntary and is innervated by post-ganglionic parasympathetic fibres. Severe, untreated perianal sepsis can lead to spincter dysfunction and faecal incontinence

41. A 6-year-old boy presents with hypospadias and undergoes reconstructive surgery. The male urethra develops from which embryological structure? Single best answer - select one answer only. Gubernaculum« YOUR ANSWER Mullerian duct Persistent cloaca« CORRECT ANSWER Sinovaginal bulbs Wolffian duct.

The male and female urethras develop from the persistent cloaca. The Mullerian (or „paramesonephric‟) duct is the embryological, mesodermal precursor to the uterus, cervix, fallopian tubes and the upper two-thirds of the vagina.

It degenerates in male development in the presence of anti-Mullerian hormone produced by the testes. The Wolffian (or „mesonephric‟) duct is crucial to male urogenital development and ------Dr Mohammed Shamsul Islam Khan, Medical Officer, Clinical Neuro-Surgery, National Institute of Neuro-Sciences and Hospital Sher-E-Bangla Nagar, Dhaka-1207, Bangladesh. Mobile: +880 1713 455 662, E-mail: [email protected]

MyPasTest: MRCS A Online - Jan Exam 2015 03. Anatomy; Aabdomen, Pelvis & Perineum (290Qs) ------regresses in females. The gubernaculum retracts the testes into the scrotum during development. The sinovaginal bulbs develop to form the inferior vagina.

42. A 35-year-old athlete presents with a tender mass in the abdomen on the lateral aspects of his rectus abdominis muscle. This occurred after a session of intense exercise. He is diagnosed with a Spigelian hernia and is scheduled for surgery. How would you describe the linea semilunaris of the rectus sheath? Page | Forms the lateral side of Hesselbach’s triangle« YOUR ANSWER 141 Is crossed by the inferior epigastric vessels« CORRECT ANSWER Is crossed by the median umbilical ligament Is formed by interdigitation of the internal and external oblique aponeuroses Is one of the preferred lines of incision in abdominal surgery.

The linea semilunaris marks the lateral margin of the rectus muscle and sheath, passing from the pubic tubercle to the costal cartilage at the tip of the ninth rib, which overlies the fundus of the gallbladder on the right.

The inferior epigastric vessels and medial umbilical ligament cross the linea posteriorly below the arcuate line, before ascending in the posterior compartment of the rectus sheath. The anterior abdominal wall would be greatly weakened both physically and functionally by cutting along the semilunar line as the nerve supply to the rectus abdominis muscle and overlying skin would be interrupted.

Furthermore, Langer‟s lines on the abdomen are horizontal. Spigelian hernias occur at the edge of the linea semilunaris, typically at the level of the arcuate line. The three borders of Hesselbach‟s triangle are the inferior epigastric artery, the inguinal ligament and the lateral border of the rectus abdominis muscle (linea semilunaris).

43. A 26-year-old primigravida aborted a 4-week-old embryo. On gross examination which of the following would have become evident in the 4th week? Single best answer question – choose ONE true option only. Differentiation of the limbs into their segments has occurred« YOUR ANSWER The cerebral hemispheres appear as hollow buds« CORRECT ANSWER The cloacal tubercle is evident The embryo is less curved and the head is relatively large The nose forms a short, flattened projection.

During the fourth week the embryo is markedly curved on itself and when viewed in profile is almost circular in outline. The cerebral hemispheres appear as hollow buds and the elevations that form the rudiments of the auricula are visible. The limbs now appear as oval flattened projections. 44. A 45-year-old gentleman presents to the Acute Surgical Admissions Unit with a one-day history of worsening epigastric pain, radiating through to the back, and vomiting. He is not able to find ------Dr Mohammed Shamsul Islam Khan, Medical Officer, Clinical Neuro-Surgery, National Institute of Neuro-Sciences and Hospital Sher-E-Bangla Nagar, Dhaka-1207, Bangladesh. Mobile: +880 1713 455 662, E-mail: [email protected]

MyPasTest: MRCS A Online - Jan Exam 2015 03. Anatomy; Aabdomen, Pelvis & Perineum (290Qs) ------any comfortable position and analgesia is not helpful. Clinical examination reveals generalised abdominal guarding and rigidity. What is the most appropriate investigation to confirm diagnosis? Amylase of 450 international units is suggestive of acute pancreatitis« YOUR ANSWER CT scan of the abdomen« CORRECT ANSWER Page | Ultrasound scan 142 Normal serum amylase excludes acute pancreatitis The modified Glasgow score.

Amylase level gradually returns to normal over 3–4 days and delay in performing the test can lead to a false negative result. A raised amylase can also occur in upper gastrointestinal (GI) perforation, mesenteric infarction, small bowel obstruction, tubo-ovarian disease, renal failure, or macroamylasemia. Lipase is produced by the pancreas alone and persists for longer than amylase. It is therefore more sensitive and specific in the diagnosis of acute pancreatitis.

The pancreas is poorly visualised by ultrasonography in over 50% of cases but is useful to assess the aetiology (eg gallstones) or to detect other pathologies (eg abdominal aortic aneurysm). CT can be used to confirm the diagnosis when the biochemical tests are inconclusive or in cases when other abdominal emergencies are suspected eg bowel perforation or infarction. Amylase is often used to confirm the diagnosis of acute pancreatitis in conjunction with clinical symptoms and signs.

However, the use of arbitrary values (e.g. >1000 or 3-4 times the upper limit of normal) are dependent upon the half life of Amylase. Where available, plasma Lipase has more accuracy than Amylase. In this context, CT scan of the abdomen is the best investigative modality in confirming acute pancreatitis. It is also useful when the diagnosis is not clear and other conditions such as perforation and peritonitis are being considered.

45. A professional cyclist complains of scrotal and penile paraesthesia and an unusual sensation during ejaculation. The affected nerve arises from which roots? Single best answer - select one answer only. L2-4« YOUR ANSWER L3-5 S1-2 S2-4« CORRECT ANSWER S3-5.

Regular cyclists are at risk of pudendal nerve entrapment (also known as Alcock Canal Syndrome). The pudendal nerve arises at the S2-4 roots and supplies sensory and somatic fibres to the external genitalia and the urinary and anal sphincters. Other than the symptoms described in the scenario above, pathology of the nerve can lead to urinary and faecal incontinence. 46. In an attempt to reduce portal pressure, a 45-year-old alcoholic undergoes a porto-systemic bypass procedure. How would you describe the portal vein? Single best answer - select one answer only. ------Dr Mohammed Shamsul Islam Khan, Medical Officer, Clinical Neuro-Surgery, National Institute of Neuro-Sciences and Hospital Sher-E-Bangla Nagar, Dhaka-1207, Bangladesh. Mobile: +880 1713 455 662, E-mail: [email protected]

MyPasTest: MRCS A Online - Jan Exam 2015 03. Anatomy; Aabdomen, Pelvis & Perineum (290Qs) ------At the epiploic foramen, it is anterior to the Hepatic artery« YOUR ANSWER It is formed in the confluence of the superior mesenteric and splenic veins« CORRECT ANSWER It lies behind the epiploic foramen It lies within the lesser omentum Page | It receives tributaries from the pancreatic neck. 143 The portal vein is the confluence of the superior mesenteric vein after its union with the splenic vein. It lies in front of the inferior vena cava, and passes upwards behind the neck of the pancreas and first part of the duodenum. It runs in the free edge of the lesser omentum, forming the anterior boundary of the epiploic foramen.

The portal vein receives tributaries from the left and right gastric, cystic and superior pancreaticoduodenal veins. At the epiploic foramen, the portal vein is the most posterior structure. Anterior and to the left lies the hepatic artery and anterior and to the right lies the common bile duct.

47. A 45-year-old motorcyclist presents with hypovolaemic shock following blunt abdominal trauma. He undergoes emergency laparotomy and is noted to have a liver laceration. The surgeon clamps the hepatoduodenal ligament just anterior to the epiploic foramen. Which vein is carried within the ligament? Cystic vein« YOUR ANSWER Gastroduodenal vein Hepatic vein Hepatic portal vein« CORRECT ANSWER Right gastric vein.

Pringle‟s Manoeuvre (clamping or grapsing of the hepatoduodenal ligament) is performed to achieve rapid haemostasis in cases of severe hepatic trauma. The ligament contains the common bile duct, the hepatic portal vein and the hepatic artery proper, which continue to form the portal triad.

48. A 25-year-old primigravida aborted a 2-month-old embryo. On gross examination: Single best answer question – choose ONE true option only. The eyelids are present in the shape of folds above and below the eye« YOUR ANSWER The different parts of the auricula are not distinguishable The lower lip is completed The neck is fully developed The palate is completely developed.

During the second month of development the flexure of the head is gradually reduced and the neck is somewhat lengthened. The upper lip is completed and the nose is more prominent. The nostrils are directed forward and the palate is not completely developed.

------Dr Mohammed Shamsul Islam Khan, Medical Officer, Clinical Neuro-Surgery, National Institute of Neuro-Sciences and Hospital Sher-E-Bangla Nagar, Dhaka-1207, Bangladesh. Mobile: +880 1713 455 662, E-mail: [email protected]

MyPasTest: MRCS A Online - Jan Exam 2015 03. Anatomy; Aabdomen, Pelvis & Perineum (290Qs) ------The eyelids are present in the shape of folds above and below the eye and the different parts of the auricula are distinguishable. By the end of the second month, the fetus measures from 28 to 30 mm in length.

49. A 2-year-old boy is brought to the paediatric emergency department because he has had several episodes of rectal bleeding. On evaluation he is diagnosed to have a 3-cm ileal outpouching located 0.6 m (2 ft) from the ileocaecal valve. This structure most commonly contains which Page | type of ectopic tissue? Single best answer question – choose ONE true option only. 144 Duodenal« YOUR ANSWER Gastric« CORRECT ANSWER Hepatic Jejunal Oesophageal.

This child has a Meckel‟s diverticulum. Meckel‟s diverticulum is present in about 2-4% of the population. Typically, it is a blindly ending pouch a few centimetres long on the antimesenteric border of the ileum within 100 cm of the ileocaecal junction. It is a remnant of an embryological structure called the vitelline duct, which once connected the yolk sac with the developing mid- gut.

Usually the vitelline duct disappears completely. Meckel‟s diverticulum may produce no symptoms. However, sometimes it can become inflamed, or it might have ectopic gastric or pancreatic cells in its walls leading to ulceration. In some cases, a fibrous strand connects the diverticulum to the inner aspect of the umbilicus and a loop of small intestine can become twisted around this (volvulus) causing obstruction.

The symptoms may closely mimic those of appendicitis. It is useful to remember the „rule of twos‟ associated with Meckel‟s diverticulum: present in 2% of the population, occurs within 0.6 m (2 ft) of the ileocaecal valve, contains two types of ectopic mucosa (gastric and pancreatic) and is usually symptomatic by the age of 2 years.

50. A 45-year-old woman undergoes a right nephrectomy for a staghorn calculus. You are the surgeon performing the procedure. From an anatomical point of view, what are you likely to note? Single best answer - select one answer only. Parietal pleura does not extend below the twelfth rib« YOUR ANSWER The adrenal glands are at easy risk of injury The perinephric fat surrounds the renal fascia of Gerota The renal artery lies anterior to the renal vein The subcostal neurovascular bundle is relatively protected from risk of injury« CORRECT ANSWER.

The lower border of the parietal pleura crosses the twelfth rib, the lateral border of the erector spinae and passes in horizontally to the lower border of the twelfth thoracic vertebra. There is therefore a triangle of pleura in the costovertebral angle below the medial part of the twelfth rib,

------Dr Mohammed Shamsul Islam Khan, Medical Officer, Clinical Neuro-Surgery, National Institute of Neuro-Sciences and Hospital Sher-E-Bangla Nagar, Dhaka-1207, Bangladesh. Mobile: +880 1713 455 662, E-mail: [email protected]

MyPasTest: MRCS A Online - Jan Exam 2015 03. Anatomy; Aabdomen, Pelvis & Perineum (290Qs) ------behind the upper pole of the kidney. The subcostal neurovascular bundle is relatively protected from injury by lying in the costal groove.

The renal fascia of Gerota surrounds the perinephric fat, which lies outside the renal capsule. The renal arteries lie behind the renal veins. The adrenal glands are anatomically well protected. They lie anterosuperior to the upper part of each kidney, within their own compartment of the renal fascia. Care must be taken during a nephrectomy to avoid damage to the adrenals, but Page | these are not easily damaged by a careful surgeon using a posterior approach. 145 51. Recognised complications of abdominal aortic aneurysm repair include all of the following, except: Single best answer - select one answer only. Incisional hernia« YOUR ANSWER Ischaemic colitis Madura foot« CORRECT ANSWER Paraplegia Trash foot.

Ligation of the inferior mesenteric artery in the absence of adequate collateral circulation to the left colon may cause ischaemic colitis. Paraplegia may result from injury to the artery of Adamkiewicz. Trash foot is caused by the lodging of distal emboli, that may cause infarction of the whole foot. Madura foot is infective in origin.

52. You are examining a patient and discover a lump in the femoral triangle. Your differential diagnosis would include ALL of the following EXCEPT: Single best answer - choose ONE option only. Ectopic testis« YOUR ANSWER Psoas abscess Saphena varix Sarcoma Troisier’s node« CORRECT ANSWER.

Differential diagnoses of a femoral hernia include: inguinal hernia, saphena varix, lymphadenopathy, psoas abscess, lipoma, femoral aneurysm, sarcoma, ectopic testes and obturator hernia. Troisier's sign is supraclavicular.

53. A 74-year-old male undergoes an emergency repair of a ruptured infrarenal abdominal aortic aneurysm. Two days later he develops signs of systemic sepsis and returns to theatre, where an infracted section of ileum is excised. The origin of the superior mesenteric artery is at which vertebral level? L1« YOUR ANSWER L2 T11 T10 ------Dr Mohammed Shamsul Islam Khan, Medical Officer, Clinical Neuro-Surgery, National Institute of Neuro-Sciences and Hospital Sher-E-Bangla Nagar, Dhaka-1207, Bangladesh. Mobile: +880 1713 455 662, E-mail: [email protected]

MyPasTest: MRCS A Online - Jan Exam 2015 03. Anatomy; Aabdomen, Pelvis & Perineum (290Qs) ------T12.

The origin of the superior mesenteric artery (SMA) is at the level of the first lumbar vertebra, also the level of the transpyloric plane. This transverse line lies halfway between the suprasternal notch and the pubic symphysis, and its plane passes through the origin of the SMA, the pancreatic neck, the first part of the duodenum, the pylorus, the renal hila, the Page | duodenojejunal flexure, the gall bladder fundus, the origin of the hepatic portal vein, the transverse mesocolon and the splenic hilum. 146

54. You are seeing a patient in A&E following a blunt abdominal trauma. An IVU demonstrates unilateral non-function. What would your diffential diagnosis list include? An absent kidney and ureteric injury« YOUR ANSWER An absent kidney, massive parenchymal disruption, renal artery avulsion, Renal artery thrombosis « CORRECT ANSWER An absent kidney, massive parenchymal disruption, renal artery avulsion, renal artery thrombosis and ureteric injury An absent kidney or massive parenchymal disruption only An absent kidney or renal artery thrombosis only.

Unilateral non-function on intravenous urography (IVU) indicates an absent kidney, thrombosis, or avulsion of the renal artery, or a massive parenchymal disruption. In these cases further imaging is required in the form of a contrast-enhanced computed tomography (CT) scan or a renal arteriogram.

55. During an elective laparoscopic inguinal hernia repair you correctly identify a direct subtype, with the sac protruding through Hesselbach‟s triangle. What is the medial border of this triangle? Ilioinguinal nerve« YOUR ANSWER Inferior epigastric vessels Inguinal ligament Linea alba Linea semilunaris« CORRECT ANSWER.

Hesselbach‟s triangle is an anatomical region bordered by the inguinal ligament (inferiorly), the linea semilunaris at the lateral edge of the rectus sheath (medially) and the inferior epigastric vessles (superolaterally). Direct inguinal hernias protrude through the , contained within the borders of the triangle.

56. An 87-year-old female presents with severe right hip pain following a fall. She is diagnosed with a fractured neck of femur and undergoes hemiarthroplasty. Which of the following arteries supply the trochanteric anastomosis? Single best answer - select one answer only. Anterior spinal artery« YOUR ANSWER External iliac artery

------Dr Mohammed Shamsul Islam Khan, Medical Officer, Clinical Neuro-Surgery, National Institute of Neuro-Sciences and Hospital Sher-E-Bangla Nagar, Dhaka-1207, Bangladesh. Mobile: +880 1713 455 662, E-mail: [email protected]

MyPasTest: MRCS A Online - Jan Exam 2015 03. Anatomy; Aabdomen, Pelvis & Perineum (290Qs) ------Gluteal arteries« CORRECT ANSWER Iliolumbar arteries Posterior spinal arteries.

The superior and inferior branches of the gluteal arteries originate from the internal iliac artery; Page | their descending branches, supply the trochanteric anastomosis of the hip (together with the ascending branch of the medial circumflex artery, they form the retinacular vessels that pierce 147 the joint capsule and ascend the femoral neck, providing main supply to the femoral head).

The iliolumbar arteries supply the anterior superior iliac spine anastomosis. The external iliac artery progresses beyond the inguinal ligament as the femoral artery to supply the lower limb. The anterior and posterior spinal arteries supply the spinal cord.

57. A two-week-old boy presents with projectile vomiting, palpable epigastric mass and failure to thrive. He is diagnosed with hypertrophic pyloric stenosis and undergoes Ramstedt‟s pyloromyotomy. Which is the deepest layer of the pyloric wall to be divided? Circular muscle « YOUR ANSWER Longitudinal muscle Mucosa Parietal peritnoneum Serosal surface.

Ramstedt‟s pyloromyotmy is indicated in hypertrophic pyloric stenosis. An incision is made along the length of the hypertrophic muscle, and the layers of longitudinal and circular muscle are divided until the submucosa is exposed.

58. In the surgical anatomy of the liver which of the following is correct? Single best answer - select one answer only. Segment I lies to the left of the portal vein« YOUR ANSWER Segment II lies lateral to the porta hepatis The caudate lobe lies anterior to the portal vein The portal vein runs in the anterior border of Winslow’s foramen« CORRECT ANSWER Three hepatic arteries divide the liver into four sectors.

The caudate lobe (segment I according to Couinaud‟s classification, 1957) lies posterior to the portal vein but anterior to the inferior vena cava. Three main hepatic veins divide the liver into four sectors, each of which receives a portal pedicle, with an alternation between hepatic veins and portal pedicles. According to this functional anatomy, the liver is divided into hemilivers (right and left) by the main portal scissura called Cantlie‟s line.

59. A 55-year old gentleman with cirrhosis of the liver has fresh PR bleeding and is found to have a mass in the low rectum. What would be the next step in his management? Computerized tomography of abdomen and pelvis« YOUR ANSWER

------Dr Mohammed Shamsul Islam Khan, Medical Officer, Clinical Neuro-Surgery, National Institute of Neuro-Sciences and Hospital Sher-E-Bangla Nagar, Dhaka-1207, Bangladesh. Mobile: +880 1713 455 662, E-mail: [email protected]

MyPasTest: MRCS A Online - Jan Exam 2015 03. Anatomy; Aabdomen, Pelvis & Perineum (290Qs) ------Capsule endoscopy Colonoscopy« CORRECT ANSWER Low anterior resection MRI Pelvis. Page | In the gastro-oesophageal region, the oesophageal tributaries of the left gastric vein anastomose with the oesophageal veins, which empty into the azygos vein. In the paraumbilical 148 region the paraumbilical veins in the falciform ligament anastomose with the subcutaneous veins in the anterior abdominal wall. In the anorectal region the superior rectal vein anastomoses with the middle and inferior rectal veins, which are anastomoses of the internal iliac and internal pudendal veins.

The differential here is rectal neoplasm, polyp and varices. The best investigation would be an endoluminal one. Biopsies should only be considered if the mass is not of a vascular nature and along the BSG guidelines.

60. A full-term baby boy is born with incomplete fusion of the embryonic endocardial cushions in the heart. The baby is most likely to have: Single best answer question – choose ONE true option only. An atrioventricular septal defect« YOUR ANSWER Coarctation of the aorta Pulmonary stenosis Tetralogy of Fallot Transposition of the great arteries.

The terms endocardial cushion defect, atrioventricular septal defect and common atrioventricular canal defect, are interchangeable in describing defects in the formation of the atrioventricular valves, the anterior portion of the atrial septum and the posterior portion of the ventricular septum.

Endocardial cushions are masses of mesenchymal tissue that form components of the arteriovenous valves, atrial septum and ventricular septum. Defects range from incomplete (also called partial, such as ostium primum atrial septal defect with „cleft mitral valve‟) to transitional (large ostium primum defect and small „inlet‟ or posterior ventricular septal defect) to complete (large ostium primum atrial septal defect, large inlet ventricular septal defect, common arteriovenous valve).

61. During inguinal hernia repair, which structures would you encounter? Colles’ fascia« YOUR ANSWER Iliohypogastric nerve Ilioinguinal nerve« CORRECT ANSWER Lateral femoral cutaneous nerve Superior epigastric vein. ------Dr Mohammed Shamsul Islam Khan, Medical Officer, Clinical Neuro-Surgery, National Institute of Neuro-Sciences and Hospital Sher-E-Bangla Nagar, Dhaka-1207, Bangladesh. Mobile: +880 1713 455 662, E-mail: [email protected]

MyPasTest: MRCS A Online - Jan Exam 2015 03. Anatomy; Aabdomen, Pelvis & Perineum (290Qs) ------

Scarpa‟s fascia is incised during the surgical approach to an inguinal hernia. The iliohypogastric nerve supplies the internal oblique and transversus, and may be divided during an appendicectomy, which may lead to a direct inguinal hernia. The iliohypogastric nerve is not encountered during normal inguinal hernia repair. However, the ilioinguinal nerve is usually seen. Page | The nerve supplies cutaneous sensation to the skin at the base of the penis and part of the scrotum. Where it continues over the penis and scrotum, Scarpa‟s fascia assumes a different 149 name: superficial perineal fascia of Colles. The superior epigastric vein drains the rectus muscle and enters the internal thoracic veins. The lateral femoral cutaneous nerve is usually visualised in the anterior approach to the hip.

62. A three-day-old boy undergoes urological surgery. During antenatal imaging, there was evidence of bilateral hydronephrosis. What is the most likely diagnosis? Single best answer - select one answer only. Anterior urethral valves« YOUR ANSWER Bladder exstrophy Glandular hypospadia Posterior urethral valves« CORRECT ANSWER Vesicoureteric reflux.

The posterior urethral valves are mucosal folds attached by the verumontanum. These come together when micturition is initiated. Posterior urethral valves have a higher incidence than anterior valves. Ultrasound scan can make the diagnosis in utero and it presents in the neonatal period with a distended bladder and dilation of the posterior portion of the urethra.

Urine may be passed in dribbling fashion or there may be complete retention. Bladder hypertrophy may occur but the kidneys are usually unaffected. Later in childhood the presentation may be with incontinence with persistent passage of urine.

63. During an open inguinal hernia repair in a 76-year-old gentleman, you find that the patient has an indirect sac that you begin to dissect free. Which of the following structures travels within the inguinal canal but remains external to the spermatic cord? Select ONE answer only. The Cremasteric artery« YOUR ANSWER The Ilioinguinal nerve« CORRECT ANSWER The Genitofemoral nerves The pampiniform plexus The Vas deferens.

The external (or superficial) inguinal ring is formed by a V-shaped defect in the external oblique aponeurosis - an indirect hernia, that may emerge from this ring, is typically felt above and medial to the pubic tubercle. It transmits the spermatic cord with all its contents, plus the ilioinguinal nerve.

------Dr Mohammed Shamsul Islam Khan, Medical Officer, Clinical Neuro-Surgery, National Institute of Neuro-Sciences and Hospital Sher-E-Bangla Nagar, Dhaka-1207, Bangladesh. Mobile: +880 1713 455 662, E-mail: [email protected]

MyPasTest: MRCS A Online - Jan Exam 2015 03. Anatomy; Aabdomen, Pelvis & Perineum (290Qs) ------The following are the contents of the spermatic cord: three fascial layers (external spermatic, cremasteric, internal spermatic); three arteries (testicular, cremasteric, artery to the vas deferens); the pampiniform plexus, veins from the cremaster and vas deferens; and the genital branch of the genitofemoral nerve, sympathetic fibres, vas deferens and lymphatics.

64. A 64-year-old male smoker presents with haematuria and straining at micturition. He is diagnosed with a malignant ureteric neoplasia and treated accordingly. Which type of Page | epithelium lines the ureter? Single best answer - select one answer only. 150 Pseudostratified« YOUR ANSWER Simple columnar Simple sqaumous Stratified squamous Transitional« CORRECT ANSWER.

The ureter is lined by transitional epithelieum, consisting of multiple layers of epithelial cells that contract and expand.

65. A 60-year-old gentleman with a past history of atrial fibrillation attends the emergency department complaining of a 3-day history of progressively worsening generalised abdominal pain and bloody diarrhoea. He has had intermittent colicky postprandial abdominal pain for the last 6 months. There are no exacerbating factors and his weight has gradually declined presumably due to reduced oral intake from apprehension to eat. Clinical examination reveals a sinus tachycardia, and hypotension. He has severe generalised abdominal pain and distension, but no specific tenderness. What is the most likely diagnosis? Acute diverticulitis« YOUR ANSWER Gastrointestinal ischaemia« CORRECT ANSWER Perforated viscus Ruptured abdominal aortic aneurysm Ulcerative colitis.

This is a question about the acute abdomen. The history suggests a gastrointestinal cause for which the most likely answer is gastrointestinal ischaemia (i.e. ischaemic colitis, which frequently presents as severe abdominal pain that is out of proportion to the clinical signs; bloody diarrhoea; and a significantly raised serum lactate that is poorly responsive to fluid resuscitation.

The concurrent atrial fibrillation is also a potential thromboembolic source that may trigger such acute ischaemia. Ruptured abdominal aortic aneurysm is unlikely as this patient has no risk factors suggesting the diagnosis. Although ulcerative colitis is a possible cause, in the acute setting it is more likely in a younger individual.

66. You are performing a laparoscopic cholecystectomy. In locating the common bile duct, which of the following is true? Single best answer question – choose ONE true option only. It crosses anterior to the second part of the duodenum« YOUR ANSWER ------Dr Mohammed Shamsul Islam Khan, Medical Officer, Clinical Neuro-Surgery, National Institute of Neuro-Sciences and Hospital Sher-E-Bangla Nagar, Dhaka-1207, Bangladesh. Mobile: +880 1713 455 662, E-mail: [email protected]

MyPasTest: MRCS A Online - Jan Exam 2015 03. Anatomy; Aabdomen, Pelvis & Perineum (290Qs) ------It crosses in front of the neck of the pancreas It lies to the left of the hepatic artery in the lesser omentum It passes anterior to the right renal vein« CORRECT ANSWER It usually opens into the duodenum separately from the pancreatic duct. Page | The common bile duct lies to the right of the hepatic artery. The common bile duct joins the pancreatic duct at the ampulla of Vater. The ampulla itself usually opens into the duodenum. It 151 crosses a groove between the head of the pancreas and the second part of the duodenum, posterior to the second part of the duodenum, in front of the right renal vein.

67. In the normal ureter the diameter of its lumen narrows at: Single best answer question – choose ONE true option only. The pelviureteric junction« YOUR ANSWER The vesicoureteric junction The point at which the iliac vessels cross it All of the above« CORRECT ANSWER None of the above.

The lumen of the ureter is not of equal diameter throughout the ureters entire length. It has three distinct narrowings, these are the pelviureteric junction proximally, then the crossing of the iliac vessels, and distally the vesicoureteric junction in the pelvis. These are possible sites of ureteric calculus impaction.

68. An elderly, confused male presents with a grossly distended tympanic abdomen and absolute constipation. A plain abdominal radiograph reveals a grossly dilated large intestine but no evidence of pneumoperitoneum. What is the most appropriate initial management? Single best answer - select one answer only. Computerised tomographic scan of the abdomen and pelvis« YOUR ANSWER Laparotomy and sigmoid colectomy Laxatives Sigmoidoscopy Trephine ileostomy.

For this scenario, the differential diagnosis is pseudobstruction, sigmoid volvulus and carcinoma of the distal colon/rectum. Computerised tomography of the abdoment and pelvis is the best option as it permits diagnosis to be obtained and the correct treatment instituted.

Pseudobstruction may be associated with an underlying precipitating condition such as renal failure, electrolyte abnormalities, myxoedema, stroke, myocardial infarction, chest infection and retroperitoneal malignancy (pseudobstruction in the presence of retroperitoneal malignancy is known as Ogilvie‟s syndrome.

------Dr Mohammed Shamsul Islam Khan, Medical Officer, Clinical Neuro-Surgery, National Institute of Neuro-Sciences and Hospital Sher-E-Bangla Nagar, Dhaka-1207, Bangladesh. Mobile: +880 1713 455 662, E-mail: [email protected]

MyPasTest: MRCS A Online - Jan Exam 2015 03. Anatomy; Aabdomen, Pelvis & Perineum (290Qs) ------Medical management comprises correction of electrolyte abnormalities, treatment of the underlying cause and use of the anticholinergic Neostigmine (with heart monitor surveillance). The colon can be decompressed with colonoscopoy. Surgery is indicated for impending caecal perforation (>10cm), right iliac fossa pain, caecal distension beyond 72h despite maximal medical therapy and pneumoperitoneum.

Options include caecostomy. With perforation a right hemicolectomy with end ileostomy and Page | mucous fistula in the presence of faecal contamination and primary anastomosis when this is not the case. Treatment for Sigmoid volvulus includes decompression with a flatus tube 152 inserted with a rigid or flexible sigmoidoscope. Emergency Laparotomy is indicated for failure of non-operative treatment, ischaemia and gangrene.

Hartmann‟s procedure is the safest in such situations. Elective surgery is considered for recurrent volvulus, chronic electrolyte abnormalities caused by recurrent volvulus. Sigmoid colectomy is the surgical treatment of choice. Fixation procedures such as sigmoidopexy have been performed but are associated with high recurrence rates.

Obstructed colon caused by a distal neoplasm is open to several surgical options following adequate resuscitation. These include stenting (currently the topic of a randomised controlled trial), defunctioning stoma, primary resection and anastomosis following on-table colonic lavage or Hartmann‟s procedure.

69. A 25-year-old gentleman presents to his General Practitioner with a palpable testicular lump. He is referred to the Urologist. What will be the best course of action? Single best answer - select one answer only. Chemotherapy« YOUR ANSWER Schedule appointment for one month - wait and see Examination of the groin followed by imaging« CORRECT ANSWER Orchidectomy Tru-cut biopsy.

A teratoma is the most likely testicular tumour to occur in the 20–30 age group and seminomas are more common in the 30–40 age group. A pre-operative biopsy of a testicular tumour is not performed due to the potential risk of seeding. Testicular tumours typically spread to the para- aortic lymph nodes.

A testicular tumour is more common following a torsion on both the ipsilateral and contralateral sides. Following initial examination, imaging is the most appropriate answer as these tumours need to be staged prior to definitive treatment. Other aspects of staging include tumour markers (Alpha fetoprotein, beta human chorionic gonadotrophin).

70. Concerning the pelvis: Single best answer question – choose ONE true option only. The male pelvis has a larger superior pelvic aperture than the female pelvis « YOUR ANSWER The obturator foramina are round in the female The male sacrum is less curved than that of the female The ischial tuberosities are farther apart in the female pelvis« CORRECT ANSWER ------Dr Mohammed Shamsul Islam Khan, Medical Officer, Clinical Neuro-Surgery, National Institute of Neuro-Sciences and Hospital Sher-E-Bangla Nagar, Dhaka-1207, Bangladesh. Mobile: +880 1713 455 662, E-mail: [email protected]

MyPasTest: MRCS A Online - Jan Exam 2015 03. Anatomy; Aabdomen, Pelvis & Perineum (290Qs) ------The male pelvis is shallower that that of the female.

The general structure of the male pelvis is heavier and thicker than the female pelvis and has more prominent bone markings. The female pelvis is wider, shallower, and has both a larger superior and inferior pelvic aperture. Page | The ischial tuberosities are farther apart in the female pelvis because of the wider pubic arch, and the sacrum is less curved in the female pelvis. In addition to this the obturator foramina are 153 round in the male and oval in the female.

71. A 79-year-old male presents with difficulty in passing urine and is diagnosed with prostatic adenocarcinoma. From which histological structure is his disease most likely to have originated? Single best answer - select one answer only. Anterior fibromuscular zone« YOUR ANSWER Central zone Peripheral zone« CORRECT ANSWER Prostatic urethra Transitional zone.

The peripheral zone of the prostate contains around 75% of all the glandular tissue. Accordingly, 80% of all prostate cancers originate here. 10-20% of cancers originate within the transitional zone, and 2.5% within the central zone. The anterior fibromuscular zone contains no glandular tissue.

72. A 25-year-old man from Uganda presents with a high anal fistula. He is a non-smoker who has had a chronic cough for 3 months and has noticed weight loss of 2 stone. What is the most likely associated condition leading to this high anal fistula? Select one answer only. Carcinoma« YOUR ANSWER Crohn’s disease Diverticular disease Tuberculosis« CORRECT ANSWER Ulcerative colitis.

High fistulae are uncommon but may be due to carcinoma, diverticular disease, tuberculosis, Crohn's disease, ulcerative colitis, trauma or radiotherapy. The most likely condition that this patient is suffering from is tuberculosis. TB is endemic in Uganda, and often presents with pulmonary symptoms, and weight loss. 73. A 26-year-old primigravida aborted a 5-week-old embryo. On gross examination: Single best answer question – choose ONE true option only. The embryo is markedly curved on itself« YOUR ANSWER The elevations that form the rudiments of the auricula are visible The primitive segments number about 30

------Dr Mohammed Shamsul Islam Khan, Medical Officer, Clinical Neuro-Surgery, National Institute of Neuro-Sciences and Hospital Sher-E-Bangla Nagar, Dhaka-1207, Bangladesh. Mobile: +880 1713 455 662, E-mail: [email protected]

MyPasTest: MRCS A Online - Jan Exam 2015 03. Anatomy; Aabdomen, Pelvis & Perineum (290Qs) ------The limbs appear as oval flattened projections The cloacal tubercle is evident« CORRECT ANSWER.

During the fifth week the embryo is less curved and the head is relatively large. Differentiation of the limbs into their segments occurs. The nose forms a short, flattened projection. The cloacal tubercle is evident. Page |

74. A 2-year-old boy presents with a large mass in the left flank and proceeds to laparotomy with 154 excision of Wilms‟ tumour. During the procedure, the lienorenal ligament is divided. Which vessels are carried by this ligament? Left colic« YOUR ANSWER Left gastric Pancreaticoduodenal Renal Splenic« CORRECT ANSWER.

The lienorenal ligament connects the hilum of the spleen to the left kidney. The splenic artery and vein are carried within it, as is the pancreatic tail. Wilms‟ tumour (nephroblastoma) is the commonest type of renal tumour of childhood, representing 20% of all childhood malignancies.

75. A 38-year-old woman with an extensive malignant growth in the anterior wall of the vagina is most likely to have involvement of which adjacent structure? Single best answer question – choose ONE true option only. Anal canal« YOUR ANSWER Fundus of the bladder« CORRECT ANSWER Perineal body Rectum Rectovesical fascia.

The anterior surface of the vagina is related to the fundus of the bladder and the urethra. A growth in the anterior wall is therefore most likely to involve the fundus of the urinary bladder and/or the urethra. The posterior surface is separated from the rectum by the rectouterine pouch in its upper quarter and by the rectovesical fascia in its middle two quarters, while the lower quarter is separated from the anal canal by the perineal body.

Its sides are enclosed between the levatores ani muscles. As the terminal portions of the ureters pass forward and medially to reach the fundus of the bladder, they run close to the lateral fornices of the vagina and as they enter the bladder they are slightly anterior to the anterior fornix. 76. When making an incision for an inguinal hernia repair, the vessels which may be encountered beneath Camper‟s fascia are: Single best answer question – choose ONE true option only. The accessory obturator vein« YOUR ANSWER The gonadal vein

------Dr Mohammed Shamsul Islam Khan, Medical Officer, Clinical Neuro-Surgery, National Institute of Neuro-Sciences and Hospital Sher-E-Bangla Nagar, Dhaka-1207, Bangladesh. Mobile: +880 1713 455 662, E-mail: [email protected]

MyPasTest: MRCS A Online - Jan Exam 2015 03. Anatomy; Aabdomen, Pelvis & Perineum (290Qs) ------The superficial inferior epigastric artery and vein« CORRECT ANSWER The superficial circumflex iliac artery and vein None of the above.

The superficial fascia just superior to the inguinal ligament can be divided into two layers: i. a Page | superficial fatty layer (Camper‟s fascia) and ii. a membranous deep layer (Scarpa‟s fascia). The superficial vessels and nerves are located between these two layers of superficial fascia. 155

One should be mindful of the superficial inferior epigastric vessels when cutting through Camper‟s as failure to do so can lead to troublesome bleeding.

77. A 46-year-old male presents with an acutely painful left groin. He is tender over the affected area and you notice that his swelling originates inferior and lateral to the pubic tubercle. What is the most likely diagnosis? Single best answer - select one answer only. Direct inguinal hernia« YOUR ANSWER Femoral hernia« CORRECT ANSWER Indirect inguinal hernia Obturator hernia Spigelian hernia.

Femoral hernias always arise inferior and lateral to the pubic tubercle. In contrast, inguinal hernias originate superior and medial to the tubercle, and can only be classified into „direct‟ or „indirect‟ at operation, when their relation to the inferior epigastric artery can be observed. Spigelian hernias originate at the linea semilunaris of the abdominal wall, and obturator hernias are rare, and usually found in females.

78. A 42-year-old male presents with abnormal penile curvature, erectile dysfunction and pain. He is diagnosed with Peyronie‟s disease. In which penile structure is the pathology? Single best answer - select one answer only. Corpora cavernosa« YOUR ANSWER Corpus spongiosum Penile bulb Perineal raphe Tunica albuginea« CORRECT ANSWER.

Peyronie‟s disease is a chronic inflammation of the tunica albuginea, which surrounds the corpora cavernosa of the penis. With time, fibrous tissue develops which causes the abnormal curvature and the other symptoms as described.

79. A 49-year-old woman is diagnosed with a malignancy of the upper rectum. From which artery is the lesion‟s blood supply most likely to be derived? Inferior mesenteric artery« YOUR ANSWER External pudendal artery ------Dr Mohammed Shamsul Islam Khan, Medical Officer, Clinical Neuro-Surgery, National Institute of Neuro-Sciences and Hospital Sher-E-Bangla Nagar, Dhaka-1207, Bangladesh. Mobile: +880 1713 455 662, E-mail: [email protected]

MyPasTest: MRCS A Online - Jan Exam 2015 03. Anatomy; Aabdomen, Pelvis & Perineum (290Qs) ------Left external iliac artery Obturator artery Superior mesenteric artery.

The arterial supply of the rectum is principally from the superior rectal artery (i.e. a continuation Page | of the inferior mesenteric artery) with assistance from the middle rectal arteries (i.e. from internal iliac arteries) to the middle and inferior parts. The inferior rectal arteries supply the 156 anorectal junctions and anal canal. Superior, middle and inferior rectal arteries anastomose with each other.

The rectum contains no by definition, although parts are covered by peritoneum. The pararectal lymph nodes drain the upper rectum into the inferior mesenteric and lumbar lymph nodes. Lymph from the lower rectum is drained to the internal iliac and inguinal lymph nodes.

The superior rectal vein drains into the inferior mesenteric vein (portal circulation). The middle rectal vein drains into the internal iliac vein. The inferior rectal vein drains into the internal pudendal vein (both systemic circulation).

80. A 19-year-old man undergoes an open repair of an indirect right inguinal hernia. Which of the following structures is least likely to be encountered inside the spermatic cord? Single best answer - select one answer only. Genital branch of the genitofemoral nerve« YOUR ANSWER Ilioinguinal nerve« CORRECT ANSWER Pampiniform plexus Testicular artery Vas deferens.

The spermatic cord contains three fascial layers (external spermatic, cremasteric, internal spermatic) and three arteries (testicular, cremasteric, artery to the vas deferens). Other structures include: pampiniform plexus, vas deferens, the genital branch of the genitofemoral nerve, sympathetic fibres and lymphatics. The ilioinguinal nerve enters the inguinal canal but not the spermatic cord.

81. A 67-year-old man presents with a long history of vague upper gastrointestinal symptoms including dysphagia and heartburn. An OGD reveals the presence of a para-oesophageal hernia. Which of these statements most accurately describes this condition? Single best answer - select one answer only. Can cause palpitations due to a relative increase in pressure on the underside of the thoracic diaphragm« YOUR ANSWER Are more common than sliding herniae Occurs in a younger age group than sliding herniae Occurs when a part of the stomach herniates through the oesophageal hiatus with subsequent herniation of the gastro oesophageal junction Occurs more commonly in female patients« CORRECT ANSWER. ------Dr Mohammed Shamsul Islam Khan, Medical Officer, Clinical Neuro-Surgery, National Institute of Neuro-Sciences and Hospital Sher-E-Bangla Nagar, Dhaka-1207, Bangladesh. Mobile: +880 1713 455 662, E-mail: [email protected]

MyPasTest: MRCS A Online - Jan Exam 2015 03. Anatomy; Aabdomen, Pelvis & Perineum (290Qs) ------

Para-oesophageal hiatus herniae occur mainly in the elderly population, and there is an established preponderance of women affected. Symptoms usually relate to the mechanical effects, and patients commonly complain of dysphagia (more than one-third of sliding herniae may be asymptomatic). Gastric volvulus is a recognised complication, but because of the elderly age group, surgery is usually risky. Page | Sliding hiatus herniae are the commonest type (95%) and occur when the oesophagogastric junction herniates through the hiatus. Para-oesophagealhernia occur when a part of the 157 stomach herniates through the oesophageal hiatus to lie next to the OG junction without herniation of the junction. Palpitations can occur but this is though pressure on the vagus nerve.

82. The spleen normally does not descend below the costal margin. However, it pushes downward and medially when pathologically enlarged. What structure limits the straight-vertical-downward movement? Single best answer question – choose ONE true option only. Left colic flexure« YOUR ANSWER Left suprarenal gland Ligament of Treitz Pancreas Stomach.

The left colic flexure, also called the splenic flexure, is the point where the colon takes a sharp downward turn. This flexure is the point where the transverse colon ends and the descending colon begins. It is located immediately inferior to the spleen, so an enlarged spleen must move medially to avoid this colic flexure.

The left suprarenal gland is a retroperitoneal structure, which sits superior to the kidney. The suspensory muscle of the duodenum or ligament of Treitz is a thin sheet of muscle derived from the right crus of the diaphragm - it suspends the fourth part of the duodenum from the posterior abdominal wall. Both the pancreas and stomach lie medial to the spleen. These organs would not prevent the spleen from descending inferiorly.

83. A 23-year-old male presents with severe perianal discomfort and fever. A magnetic resonance imaging scan of the pelvis is performed and the radiologist reports a collection of pus inferior and lateral to levator ani. What is the most likely diagnosis? Single best answer - select one answer only. Horseshoe abscess« YOUR ANSWER Intersphincteric abscess Ischiorectal abscess« CORRECT ANSWER Perianal abscess Supralevator abscess.

The ischiorectal (or ischioanal) fossae are prism-shaped fascia-lined potential spaces adjacent to the rectum. They are bordered by the levator ani muscle (superiorly and medially), the anal

------Dr Mohammed Shamsul Islam Khan, Medical Officer, Clinical Neuro-Surgery, National Institute of Neuro-Sciences and Hospital Sher-E-Bangla Nagar, Dhaka-1207, Bangladesh. Mobile: +880 1713 455 662, E-mail: [email protected]

MyPasTest: MRCS A Online - Jan Exam 2015 03. Anatomy; Aabdomen, Pelvis & Perineum (290Qs) ------canal (medially), the perineal skin (inferiorly), and the ischial tuberosity (laterally). Ischiorectal abscesses are the second most common type of anorectal sepsis. Perianal abscesses are the most common.

84. A 17-year-old boy with chronic diarrhoea, bloody stools and weight loss also reports left hip pain. Clinically, he is pyrexial with a temperature of 38 °C and his left leg is flexed. Straightening the leg causes pain. A CT scan of the abdomen and pelvis reveals a psoas abscess. What is the Page | insertion of the psoas muscle? Single best answer - select one answer only. 158 Distal femoral shaft« YOUR ANSWER Femoral neck Greater trochanter Lesser trochanter« CORRECT ANSWER Proximal femoral shaft.

The psoas muscle originates from the lateral surfaces of the T12-L5 vertebrae, and inserts alongside the iliacus muscle at the lesser trochanter of the femur. It acts to flex the hip joint. Psoas abscess is a recognised complication of Crohn‟s Disease and can also develop secondary to appendicitis, renal infection and spinal tuberculosis

85. A 3-year-old Ugandan child presents with anterior abdominal wall swelling and pain. The admitting consultant suggests a Mayo repair is the most suitable surgical intervention. What is the diagnosis? Single best answer - select one answer only. Epigastric hernia« YOUR ANSWER Inguinal hernia Paraumbilical hernia Spigelian hernia Umbilical hernia« CORRECT ANSWER.

An umbilical hernia protrudes through the umbilical cicatrix to lie in the subcutaneous tissues. They will often resolve as the child grows and few will require surgical treatment. Those that do can be repaired with the Mayo „vest-over-pants‟ approach. There is a higher incidence in black children than white.

86. The transpyloric plane: Single best answer question – choose ONE true option only. Is situated midway between the jugular notch and the pubic tubercle« YOUR ANSWER Passes through the superior border of L1 Passes through the tail of the pancreas Passes through the neck of the pancreas« CORRECT ANSWER Passes through the second part of the duodenum.

The transpyloric plane is situated midway between the jugular notch in the manubrium of the sternum and the pubic symphysis. It passes through the inferior border of L1, the pylorus of the

------Dr Mohammed Shamsul Islam Khan, Medical Officer, Clinical Neuro-Surgery, National Institute of Neuro-Sciences and Hospital Sher-E-Bangla Nagar, Dhaka-1207, Bangladesh. Mobile: +880 1713 455 662, E-mail: [email protected]

MyPasTest: MRCS A Online - Jan Exam 2015 03. Anatomy; Aabdomen, Pelvis & Perineum (290Qs) ------stomach, the anterior parts of the 9th costal cartilages, the duodenojejunal junction, the head, neck and body of the pancreas, and the renal hila.

87. A 70-year-old female presents to the clinic with a lump in the groin. The lump is reducible and has a cough impulse. It is located above and medial to the pubic tubercle. Which of the following is true about this type of hernia? Page | In women a direct hernia is more likely than an indirect one« YOUR ANSWER Repair requires fixation of mesh to the reflected edge of the Inguinal ligament« CORRECT 159 ANSWER Repair requires preservation of the round ligament The diagnosis is that of an femoral hernia which is more common than an inguinal hernia in women The hernia can be repaired with a local anaesthetic block to L2 and L3.

An inguinal hernia emerges above and medial to the pubic tubercle. The inguinal canal in women contains the round ligament of the uterus and a remnant of the processus vaginalis. A persistent processus vaginalis in women forms the canal of Nuck. The floor of the inguinal canal is formed by the inguinal ligament.

During repair the round ligament can be divided. Inguinal hernia can be repaired by a local anaesthetic block that requires infiltration of the Iliohypogastric and Ilioinguinal nerves (T12 and L1), 2 cm medial from the anterior superior iliac spine, Genitofemoral nerve (L1 and L2), 1-2cm lateral to the pubic tubercle and local infiltration of the subcutaneous tissues along the line of incision.

Femoral hernia accounts for 20% and 5% of hernias in women and men respectively. The male to female ratio of inguinal hernia is 10 to 1. Direct hernias are extremely rare in females.

88. The ejaculatory duct opens into the: Single best answer question – choose ONE true option only. Membranous urethra« YOUR ANSWER Prostatic urethra« CORRECT ANSWER Spongy urethra Lateral lobes of the prostate gland Duct of the bulbourethral glands. The ejaculatory ducts are two in number, one on either side of the middle line. Each is formed by the union of the duct from the seminal vesicles with the ductus deferens and is about 2 cm long.They commence at the base of the prostate and run forward and downward between its middle and lateral lobes and along the sides of the prostatic utricle, to end by separate slit-like orifices close to or just within the margins of the utricle in the prostatic urethra. The ducts diminish in size and also converge, toward their terminations.

89. When performing digital rectal examination on a female patient, what should you normally be able to palpate? Single best answer - select one answer only. Cervix« YOUR ANSWER ------Dr Mohammed Shamsul Islam Khan, Medical Officer, Clinical Neuro-Surgery, National Institute of Neuro-Sciences and Hospital Sher-E-Bangla Nagar, Dhaka-1207, Bangladesh. Mobile: +880 1713 455 662, E-mail: [email protected]

MyPasTest: MRCS A Online - Jan Exam 2015 03. Anatomy; Aabdomen, Pelvis & Perineum (290Qs) ------Ischial tuberosity Rectovesical pouch Seminal vesicles Ureter. Page | On digital rectal examination, the structures that can be palpated in either sex are: the coccyx and lower sacrum posteriorly; the ischial spines and ischiorectal fossae at the laterally; and the 160 anorectal ring at the anorectal junction. In men, the prostate can be felt (but normal seminal vesicles are not usually palpable). In women, the cervix can be felt through the vaginal wall, with the uterosacral ligaments laterally, and sometimes the .

90. A 45-year-old man is involved in a road traffic accident where as the driver he is hit from behind by a car travelling at 50 mph. He was wearing his seatbelt. At the scene, the car is overturned and airbags deployed. He is taken to the emergency department intubated and in cervical spine immobilisation. Blood is noticed at the urethral meatus and the prostate gland is high riding. What is the most likely pattern of his injuries? Deceleration, compression, restraint injuries« YOUR ANSWER Fracture dislocation of acetabulum and head of femur which may present with blood at the external urinary meatus Myocardial contusion, pneumothorax, vertebral fracture, rupture spleen, pancreas, liver and kidneys which lead to impotence in approximately 50% of cases Rupture of membranous urethra that requires urethral catheterization and is best investigated at the time of multiple injury Unstable vertebral fracture, intra-abdominal and pelvic organ injuries.

With this type of scenario, the likely injuries are multifactorial and may include fracture dislocation of the femur and acetabulum, and additionally where there is membranous injury to the urethra this should NOT be catheterized per urethrum.

The most likely pattern of injuries in this particular scenario are deceleration, compression, and restraint injuries as this encompasses the likely injury mechanisms. Myocardial contusion, pneumothorax, vertebral fracture, rupture spleen, pancreas, liver and kidneys are also possible with this scenario but less likely to occur.

91. During a difficult vaginal delivery, the SHO on call was asked to come and make an episiotomy. She made a median episiotomy and in doing so cut too far through the perineal body into the structure immediately posterior. Which structure did she cut? Single best answer question – choose ONE true option only. External anal sphincter« YOUR ANSWER Bulbospongiosus muscle Ischiocavernosus muscle Sphincter urethrae

------Dr Mohammed Shamsul Islam Khan, Medical Officer, Clinical Neuro-Surgery, National Institute of Neuro-Sciences and Hospital Sher-E-Bangla Nagar, Dhaka-1207, Bangladesh. Mobile: +880 1713 455 662, E-mail: [email protected]

MyPasTest: MRCS A Online - Jan Exam 2015 03. Anatomy; Aabdomen, Pelvis & Perineum (290Qs) ------Sacrospinous ligament.

An episiotomy is an incision made in the perineum to enlarge the distal end of the birth canal and to prevent serious damage to the perineal structures. This procedure is often performed when there is a risk of tearing the birth canal during a breech or forceps delivery. When performing a median episiotomy, a cut is made immediately posterior to the vagina, through the Page | perineal body. 161 If this cut went too far, the physician might cut through the external anal sphincter or the rectum. It is important to remember that episiotomies are usually made in the posterolateral direction, not on the midline. If the incision tears further during the delivery, a median incision is more likely than a posterolateral incision to extend posteriorly through the external anal sphincter and the rectum. A posterolateral incision is much safer! The bulbospongiosus muscle, ischiocavernosus muscle and sphincter urethrae are anterior to the area that is cut during an episiotomy. The sacrospinous ligament extends from the sacrum to the ischial spine - it is deep to the perineum and should not be involved in this procedure.

92. Which of the following is a correct description of the hepatorenal pouch? Single best answer - select one answer only. It communicates freely with the greater sac« YOUR ANSWER It communicates freely with the right paracolic gutter« CORRECT ANSWER It communicates freely with the right subphrenic space It is closed inferiorly by the inferior It lies posterior to the right kidney.

The hepatorenal pouch lies between the right lobe of the liver and right kidney. It forms the lowest part of the peritoneal cavity, apart from the pelvis, in the supine abdomen and communicates freely with the lesser sac and the right paracolic gutter. So intraperitoneal fluid or pus can easily accumulate in the hepatorenal pouch. The space is closed superiorly by the inferior coronary ligament and so does not directly communicate with the right subphrenic space.

93. Inflammatory aortic aneurysms are possibly a more severe form of degenerative AAA. Which of the following may lead you to confirm or suspect inflammatory aortic aneurysm? Single best answer - select one answer only. They are always associated with an elevated erythrocyte sedimentation rate (ESR)« YOUR ANSWER They can present with renal failure« CORRECT ANSWER They are more common in females They are responsible for 20% of all abdominal aortic aneurysms in smokers They are usually already known to be inflammatory, pre-operatively.

------Dr Mohammed Shamsul Islam Khan, Medical Officer, Clinical Neuro-Surgery, National Institute of Neuro-Sciences and Hospital Sher-E-Bangla Nagar, Dhaka-1207, Bangladesh. Mobile: +880 1713 455 662, E-mail: [email protected]

MyPasTest: MRCS A Online - Jan Exam 2015 03. Anatomy; Aabdomen, Pelvis & Perineum (290Qs) ------Inflammatory aortic aneurysm account for 5-10% of all AAAs. They should be suspected in younger patients with little other vascular disease and are more common in males who smoke. They may be accompanied by an elevated ESR, but these aneurysms are often small.

Computerised tomography (CT) or ultrasound may be required. Most are however diagnosed at the time of operation. The inflammatory process may be extensive and involve the 4th part of Page | the duodenum, ureters, mesentery, small bowel, sigmoid etc. If the aneurysm compresses the ureter the patient can present with renal failure. 162

94. While reviewing the abdominal aortogram of a 67-year-old man with an abdominal aortic aneurysm, a radiologist noticed an occluded inferior mesenteric artery. However, on enquiry, the patient denied having any abdominal symptoms. Occlusion of the inferior mesenteric artery is seldom symptomatic because its territory may be supplied by branches of the: Single best answer question – choose ONE true option only. Gastroduodenal artery« YOUR ANSWER Ileocolic artery Middle colic artery« CORRECT ANSWER Right colic artery Splenic artery.

The middle colic artery is the branch from the superior mesenteric artery that supplies the transverse colon. This is the most distal part of the colon that receives blood from the SMA. Branches from the middle colic go to the marginal artery, which would be able to supply the descending colon, sigmoid colon and rectum if the inferior mesenteric artery was occluded.

The ileocolic and right colic arteries are also branches of the SMA that supply the colon (and contribute to the marginal artery), but the middle colic, which serves a more distal part of the colon, is a better answer.

The gastroduodenal artery is a branch off the common hepatic artery, which supplies parts of the duodenum, pancreas and stomach. The splenic artery is one of the three branches of the coeliac trunk. It supplies the spleen, pancreas and curvature of the stomach.

95. During an abdominal aortic aneurysm repair your consultant points to the epiploic foramen and asks what forms its inferior border. What would be the appropriate response? Aorta« YOUR ANSWER Caudate lobe of the liver First part of the duodenum« CORRECT ANSWER Portal vein Second part of the duodenum.

The epiploic foramen of Winslow lies behind the free edge of the lesser omentum. The posterior wall of the epiploic foramen is formed by the peritoneum of the posterior abdominal wall, which passes as a smooth layer from the hepatorenal pouch to the duodenum. ------Dr Mohammed Shamsul Islam Khan, Medical Officer, Clinical Neuro-Surgery, National Institute of Neuro-Sciences and Hospital Sher-E-Bangla Nagar, Dhaka-1207, Bangladesh. Mobile: +880 1713 455 662, E-mail: [email protected]

MyPasTest: MRCS A Online - Jan Exam 2015 03. Anatomy; Aabdomen, Pelvis & Perineum (290Qs) ------

Behind this peritoneum lies the inferior vena cava. The superior border of the epiploic foramen is formed by the inferior surface of the liver (caudate lobe). The first part of the duodenum forms part of the inferior border. The portal vein lies anteriorly in the free edge of the opening.

96. You suspect a patient you are treating has ischaemic bowel secondary to a thrombosis of the superior mesenteric artery. In reviewing the scans which of the following would allow you to Page | locate the superior mesenteric artery? Single best answer question – choose ONE true option only. 163 It crosses posterior to the third part of the duodenum« YOUR ANSWER It lies to the left of the inferior mesenteric artery It lies to the right of the superior mesenteric vein It passes posterior to the left renal vein It passes posterior to the splenic vein« CORRECT ANSWER.

The superior mesenteric artery supplies the gut from the mid-second part of the duodenum to a level just short of the splenic flexure of the colon. It is directed downwards anterior to the left renal vein and posterior to the splenic vein and pancreas. The superior mesenteric vein lies on its right side. It lies anterior to third part of the duodenum.

97. A 58-year-old female presents with a pulsatile abdominal swelling and undergoes duplex imaging. This reveals a 5.1cm abdominal aortic aneurysm that appears to be infrarenal. Her abdomen is soft and non-tender. Hypertension is her only co-morbidity. What is the most appropriate management? Single best answer - select one answer only. Anticoagulation« YOUR ANSWER Discharge to primary care for blood pressure control Endovascular aneurysm repair Open aneurysm repair Surveillance« CORRECT ANSWER.

Infrarenal aortic aneurysms are usually amenable to endovascular repair, but surgery of any type is only indicated if the diameter is greater than 5.4cm, or if the aneurysm is symptomatic, or if it has grown by more than 1cm in the previous year. Patients with small aneurysms should be offered surveillance imaging and clinical review to ensure that none of these criteria are satisfied.

98. Which of the following best describes the autonomic nervous supply to the abdomen? Consists of only parasympathetic fibres« YOUR ANSWER Fibres from the greater and lesser splanchnic nerves synapse in the coeliac ganglion« CORRECT ANSWER Preganglionic parasympathetic fibres run in the greater, lesser and least splanchnic nerves The inferior hypogastric plexus is a midline structure

------Dr Mohammed Shamsul Islam Khan, Medical Officer, Clinical Neuro-Surgery, National Institute of Neuro-Sciences and Hospital Sher-E-Bangla Nagar, Dhaka-1207, Bangladesh. Mobile: +880 1713 455 662, E-mail: [email protected]

MyPasTest: MRCS A Online - Jan Exam 2015 03. Anatomy; Aabdomen, Pelvis & Perineum (290Qs) ------The vagus nerve supplies foregut, but not mid-gut or hindgut derivatives.

The abdomen has both sympathetic and parasympathetic innervation. Visceral sympathetic fibres from the thoracolumbar sympathetic outflow pass through the sympathetic chain without synapsing and form the greater (T5–9), lesser (T10–11) and least (T12) splanchnic nerves. The fibres from the greater and lesser splanchnic nerves synapse at the coeliac ganglion. Page |

The inferior hypogastric plexi receive both sympathetic and parasympathetic fibres. They are 164 bilateral structures located on the lateral pelvic wall. The vagus nerve supplies parasympathetic fibres to the fore- and mid-gut. The pelvic splanchnic nerves supply hindgut derivatives.

99. During a hemihepatectomy your consultant points to an H-shaped arrangement of structures on the visceral surface between the limbs off which lie the caudate and quadrate lobes. Which structures form this H shape? Aorta, IVC, Ligamentum teres« YOUR ANSWER Aorta, Gall bladder, IVC, Ligamentum teres, Porta hepatis Aorta, Gall bladder, IVC, Ligamentum teres Gall bladder, IVC, Ligamentum teres, Porta hepatis« CORRECT ANSWER IVC, Ligamentum teres, Porta hepatis.

The four structures comprising the H-shape dividing the visceral surface of the liver are: 1. The gall bladder 2. The IVC 3. The ligamentum teres 4. The porta hepatis.

100. A three-week-old boy presents with non-bilious vomiting, hypokalaemia and hypochloraemic alkalosis. On examination he has an epigastric mass. He is diagnosed with pyloric stenosis and is taken to theatre to undergo Ramstedt‟s pyloromyotomy. To which level should the incision be made? Single best answer - select one answer only. Circular muscle layer« YOUR ANSWER Longitudinal muscle layer Mucosa« CORRECT ANSWER Serosa Submucosa.

For Ramstedt‟s procedure to be effective, a longitudinal incision down to the level of the mucosa should be performed.

101. A 45-year-old male presents with a swelling of the right groin. He is diagnosed with an inguinal hernia and proceeds to surgery. During the procedure, you notice a direct hernia through the posterior wall of the inguinal canal. Which structure forms the lateral one-third of the posterior wall of the inguinal canal? Aponeurosis of external oblique« YOUR ANSWER ------Dr Mohammed Shamsul Islam Khan, Medical Officer, Clinical Neuro-Surgery, National Institute of Neuro-Sciences and Hospital Sher-E-Bangla Nagar, Dhaka-1207, Bangladesh. Mobile: +880 1713 455 662, E-mail: [email protected]

MyPasTest: MRCS A Online - Jan Exam 2015 03. Anatomy; Aabdomen, Pelvis & Perineum (290Qs) ------Transversus abdominis Inguinal ligament Internal oblique Transversalis fascia « CORRECT ANSWER. Page | The inguinal canal is an oblique intermuscular slit about 4 cm long lying above the medial half of the inguinal ligament. It commences at the deep inguinal ring, ends at the superficial inguinal 165 ring, and transmits the spermatic cord and ilioinguinal nerve in the male and the round ligament of the uterus and ilioinguinal nerve in the female. Its anterior wall is formed by the external oblique aponeurosis, assisted laterally by the internal oblique muscle. Its floor is the inrolled lower edge of the inguinal ligament, reinforced medially by the lacunar ligament.

Its roof is formed by the lower edges of the internal oblique and transversus muscles, which arch over from in front of the cord laterally to behind the cord medially, where their conjoined aponeuroses, constituting the conjoint tendon, are inserted into the pubic crest and the pectineal line of the pubic bone. The posterior wall of the canal is formed by the strong conjoint tendon medially and the weak transversalis fascia throughout.

102. A 21-year-old man is stabbed in the abdomen with a long knife. Which part of the abdomen must be lacerated to result in an inferior vena cava (IVC) injury? The central tendon of the diaphragm at T12 level« YOUR ANSWER The diaphragm at T10 level The free edge of the lesser omentum The left paravertebral space The posterior wall of the epiploic foramen« CORRECT ANSWER.

The IVC commences opposite the L5 vertebra. It runs on the right side of the aorta, upwards beyond the aortic opening of the diaphragm and extends to the central tendon of the diaphragm, which it pierces at the level of T8.

The IVC lies behind the portal vein near the pancreas and bile duct, and forms the posterior wall of the epiploic foramen of Winslow. The right vein of the suprarenal gland is usually only a few millimetres long and enters the IVC directly. The right suprarenal vein drains directly in the IVC. The left suprarenal vein is longer and enters the left renal vein.

103. A 55-year old gentleman with severe epigastric pain radiating through to the back has an Amylase of 500 International Units, a total Bilirubin of 50 mmol/l, Alkaline Phosphatase of 250 mmol/l, an Aspartate and Alanine Transaminase of 50 and 95 mmol/l. Severity can be predicted with which one of the following? APACHE I score« YOUR ANSWER C-reactive protein Glasgow Score« CORRECT ANSWER Ranson’s criteria ------Dr Mohammed Shamsul Islam Khan, Medical Officer, Clinical Neuro-Surgery, National Institute of Neuro-Sciences and Hospital Sher-E-Bangla Nagar, Dhaka-1207, Bangladesh. Mobile: +880 1713 455 662, E-mail: [email protected]

MyPasTest: MRCS A Online - Jan Exam 2015 03. Anatomy; Aabdomen, Pelvis & Perineum (290Qs) ------Serum amylase.

Levels of serum amylase and lipase are not predictive of severity. CRP > 150 mg/l more than 48 hours after the onset of symptoms, pleural effusion on chest X-ray, and an APACHE II score > 8 are predictive of severe acute pancreatitis. CT can be used to assess prognosis by grading the extent of pancreatic necrosis (Balthazar score). Page |

Other CT features such as necrosis of the head of pancreas, mesenteric oedema and intra- 166 peritoneal fluid are also associated with a poorer outcome. The British Society of Gastroenterology recommends that the Glasgow score and APACHE II score be used to predict severity from 24h after admission. Ranson‟s criteria are only applicable to acute pancreatitis caused by alcohol consumption.

104. A 45-year-old male presents with rectal bleeding, weight loss and deep pelvic pain. Subsequent CT imaging of the abdomen and pelvis reveals a large rectal tumour with hepatic metastases. By which anatomical route are metastatic cells likely to „settle‟ in the liver? Inferior mesenteric vein to superior mesenteric vein« YOUR ANSWER Inferior mesenteric artery to hepatic artery Middle rectal vein to internal rectal plexus Inferior mesenteric vein to Splenic vein« CORRECT ANSWER Superior mesenteric vein to portal vein.

The inferior mesenteric vein (IMV) drains abdominal viscera originating from the embryological hindgut. It is a tributary of the splenic vein, which confluences with the superior mesenteric vein (SMV) to form the hepatic portal vein (HPV). The rectal veins are themselves tributaries of the inferior mesenteric vein. Anatomical variation does exist, where the IMV drains into the SMV or HPV directly, but this is not usual.

105. A 16-year-old male presents with right iiac fossa pain and vomiting. He is taken to theatre with a diagnosis of suspected acute appendicitis. During the procedure his appendix is noted to be normal, but there is evidence of an inflamed Meckel‟s diverticulum on the antimesenteric ileal border. Such diverticulae are remnants of which embryological structure? Cloaca« YOUR ANSWER Gubernaculum Mesonephric duct Paramesonephric duct Vitelline duct« CORRECT ANSWER.

The vitelline duct joins the yolk sac to the midgut in the primitive embryo. 2% of the population have a remnant of this duct approximately 2 feet from the ileocaecal junction on the antimesenteric border of the ileum. If these Meckel‟s diverticulae become inflamed, they can give symptoms and signs consistent with a diagnosis of acute appendicitis. The gubernaculum contracts to draw the testes into the scrotal sac, and the cloaca, meso- and paramesonephric ducts are the embryological precursors of many urogenital structures.

------Dr Mohammed Shamsul Islam Khan, Medical Officer, Clinical Neuro-Surgery, National Institute of Neuro-Sciences and Hospital Sher-E-Bangla Nagar, Dhaka-1207, Bangladesh. Mobile: +880 1713 455 662, E-mail: [email protected]

MyPasTest: MRCS A Online - Jan Exam 2015 03. Anatomy; Aabdomen, Pelvis & Perineum (290Qs) ------106. A 48-year-old female with metastatic breast cancer presents with a painful, erythematous swelling of the right calf. She undergoes a procedure to insert a caval filter, intended to reduce the risk of pulmonary embolus. At which vertebral level does the inferior vena cava pierce the diaphragm? T6« YOUR ANSWER T8« CORRECT ANSWER Page | T10 167 T12 L1.

The inferior vena cava pierces the diaphragm at the level of the 8th thoracic vertebra. The caval opening also admits the right phrenic nerve. The aorta traverses the diaphragm at the level of the twefth thoracic vertebra, with the oesophageal hiatus at the level of the tenth thoracic vertebra.

107. A specialist registrar while performing inguinal hernia repair made an incision parallel to and 5 cm (2 inches) above the inguinal ligament. The supervising consultant warned him to look out for the inferior epigastric vessels. The specialist registrar is most likely to find the inferior epigastric vessels between which layers of the abdominal wall? Single best answer question – choose ONE true option only. Camper’s fascia and Scarpa’s fascia« YOUR ANSWER External abdominal oblique and internal abdominal oblique muscles Internal abdominal oblique and transversus abdominis muscles Skin and deep fascia of the abdominal wall Transversus abdominis muscle and peritoneum« CORRECT ANSWER.

The inferior epigastric vessels lie on the inner surface of the transversus abdominis and are covered by parietal peritoneum. Remember: the peritoneum lies over the inferior epigastric vessels to make the . Camper‟s fascia and Scarpa‟s fascia are two layers of the superficial fascia - Camper‟s is the fatty layer and Scarpa‟s is the membranous layer.

108. The descending duodenum: Single best answer question – choose ONE true option only. Is a retroperitoneal structure« YOUR ANSWER Receives the common bile duct Lies lateral to the head of the pancreas Lies anterior to the hilum of the right kidney All of the above« CORRECT ANSWER.

The descending part of the duodenum (the second part) descends vertically, directly anterior to the hilum of the right kidney, and as a result is intimately related on its posterior aspect to the medial margin of the right kidney, renal pelvis, pelviureteric junction, and often the right proximal ureter. The common bile duct also lies posterior and drains into the descending ------Dr Mohammed Shamsul Islam Khan, Medical Officer, Clinical Neuro-Surgery, National Institute of Neuro-Sciences and Hospital Sher-E-Bangla Nagar, Dhaka-1207, Bangladesh. Mobile: +880 1713 455 662, E-mail: [email protected]

MyPasTest: MRCS A Online - Jan Exam 2015 03. Anatomy; Aabdomen, Pelvis & Perineum (290Qs) ------duodenum. Directly medial and intimately related to the descending duodenum lies the pancreatic head.

109. A builder falls from the second storey of a building under construction. The patient is conscious but complains of not feeling his „legs‟. Neurological assessment in the Emergency Department suggests that he has no cutaneous sensation from his umbilicus to his toes. The patient has sustained a spinal cord injury at which spinal cord level? Single best answer Page | question – choose ONE true option only. 168 T6« YOUR ANSWER T8 T10« CORRECT ANSWER L2 L4.

The umbilicus is an important landmark on the abdomen, because its position is relatively consistent among humans. The skin around the waist at the level of the umbilicus is supported by the tenth thoracic spinal nerve (T10 dermatome). The umbilicus is at the level of the fibrocartilage between the third and fourth lumbar vertebrae.

110. A 40-year-old man presents to the Emergency Department with a penile fracture. If the Buck‟s fascia remains intact, where will the haematoma be visible? Single best answer question – choose ONE true option only. The perineum in a butterfly pattern« YOUR ANSWER The penis, scrotum, and perineum tracking up the anterior abdominal wall The penis and scrotum only The penile shaft only« CORRECT ANSWER The shaft and the glans penis only.

A penile fracture is a result of a tear in the tunica albuginea, which is a dense white fibrous capsule enclosing the corporal bodies. Such a tear leads to bleeding which is usually contained by Buck‟s fascia. Given the fact that Buck‟s fascia (the deep fascia of the penis) remains intact the haematoma is therefore limited to the shaft of the penis only.

111. A 36-year-old lady presents to casualty with profuse hematemesis. She is a heavy drinker and has been for several years. On examination she appears confused and has obvious ascites. Which of the following represents the first step in her management? Administration of vasoactive drugs such as somatostatin or octreotide« YOUR ANSWER Balloon tamponade using a Sengstaken-Blakemore tube Performing a TIPS procedure as an emergency Performing an urgent OGD and endoscopic band ligation Securing iv access« CORRECT ANSWER.

------Dr Mohammed Shamsul Islam Khan, Medical Officer, Clinical Neuro-Surgery, National Institute of Neuro-Sciences and Hospital Sher-E-Bangla Nagar, Dhaka-1207, Bangladesh. Mobile: +880 1713 455 662, E-mail: [email protected]

MyPasTest: MRCS A Online - Jan Exam 2015 03. Anatomy; Aabdomen, Pelvis & Perineum (290Qs) ------Chronic alcoholism is one cause of intra-hepatic portal hypertention, the consequences of which are caused by blood being forced down alternate channels due to the increased portal pressure. These include hepatic encephalopathy, ascites, hepatorenal syndrome, splenomegaly and portocaval anastomosis.

Oesophageal varices may present with profuse haematemesis as in this case and the most appropriate first step in management is to establish iv access and crossmatch blood. The other Page | treatment modalities should then be considered. 169 112. You are consenting a 46-year-old female for laparoscopic cholecystectomy. Which of the following would you mention? Select one answer only. Liver cirrhosis alone is an absolute contraindication« YOUR ANSWER Operative cholangiography is mandatory during laparoscopic cholecystectomy Previous abdominal surgery is an absolute contraindication The average in-patient stay is 1 day« CORRECT ANSWER 20% require conversion to open cholecystectomy.

Operative cholangiography does not prevent bile duct injury and ultrasound MRCP before surgery can determine the presence or absence of common bile duct stones. Conversion rates of less than 5% should be achieved. Absolute contraindications include liver cirrhosis with portal hypertension (i.e. due to low peripheral resistance and increased risk of bleeding), inability to tolerate general anaesthesia and severe coagulopathy; relative contraindications include liver cirrhosis without portal hypertension, mild coagulopathy, pregnancy (second trimester and beyond) and severe cardiorespiratory insufficiency.

113. A 55-year-old male executive who had a history of a chronic duodenal ulcer was admitted to the emergency department exhibiting signs of a severe internal haemorrhage. He was quickly diagnosed with perforation of the posterior wall of the first part of the duodenum and erosion of an artery behind it by the gastric expellant. What is the most likely artery to be involved? Single best answer question – choose ONE true option only. Common hepatic« YOUR ANSWER Gastroduodenal« CORRECT ANSWER Left gastric Proper hepatic Superior mesenteric. The gastroduodenal artery is a branch of the common hepatic artery; it descends behind the first part of the duodenum. Therefore, if an ulcer destroyed the posterior wall of the duodenum, gastric juices could escape and destroy the gastroduodenal artery. The common hepatic artery is a branch of the coeliac trunk found superior to the duodenum.

The left gastric artery is a branch of the coeliac trunk, which supplies the left side of the lesser curvature of the stomach. The proper hepatic artery is a branch of the common hepatic artery; it travels superiorly from the common hepatic artery to give off the right, middle and left hepatic arteries. Finally, the superior mesenteric artery originates from the aorta at the bottom of the L1 ------Dr Mohammed Shamsul Islam Khan, Medical Officer, Clinical Neuro-Surgery, National Institute of Neuro-Sciences and Hospital Sher-E-Bangla Nagar, Dhaka-1207, Bangladesh. Mobile: +880 1713 455 662, E-mail: [email protected]

MyPasTest: MRCS A Online - Jan Exam 2015 03. Anatomy; Aabdomen, Pelvis & Perineum (290Qs) ------level, posterior to the pancreas. It travels over the third part of the duodenum and supplies the intestines, up to the last third of the transverse colon.

114. A 68-year-old woman presents with symptoms of gastro-oesophageal reflux disease. She has tested positive for Helicobacter pylori infection and been started on a PPI and eradication therapy. Which of the following tests, in common usage, has the highest sensitivity/specificity when used to detect Helicobacter pylori infection? Single best answer - select one answer only. Page | Culture of biopsy material« YOUR ANSWER 170 McNemar’s test The biopsy urease test The urea breath test« CORRECT ANSWER Serology/stool testing.

H.pylori can be initially detected using either carbon-13 urea breath test, a stool antigen test, laboratory based serology or the invasive “clo” test. Biopsy of the stomach wall is the basis of the urease test („Clo-test‟), which takes up to 30 minutes to get a result, and depends on the presence of urease in the bacterium to alter the substrate from orange to pink.

Culture of biopsies may yield the presence of H. pylori, but this is not routinely used. The urea breath test detects radiolabelled carbon dioxide in the breath after ingesting radiolabelled urea. McNemar‟s test is a non-parametric statistical test and nothing to do with H.Pylori ! Serological testing (sensitivity 92%, specificity 83%) performs less well when compared to breath testing (sensitivity 95%, specificity 96%) and stool testing (sensitivity 95%, specificity 94%). Ref NICE CG17.

115. You are the surgical ST3 taking your CT1 trainee through his first open inguinal hernia repair and are testing your trainees knowledge regarding intraoperative anatomy. Which of the following is correct regarding the mid-point of the inguinal ligament? It forms the surface marking of the femoral pulse« YOUR ANSWER IT lies at a point halfway between the anterior superior iliac spine and the symphysis pubis It lies immediately superior to the deep inguinal ring It lies closer to the midline on the left than the right It lies laterally to the mid-inguinal point« CORRECT ANSWER.

The mid-point of the inguinal ligament lies at a point halfway between the anterior superior iliac spine and the pubic tubercle, and is the surface landmark of the deep inguinal ring. It is distinct from and lies laterally to, the mid-inguinal point, which lies halfway between the anterior superior iliac spine and the symphysis pubis and is the surface marking of the femoral pulse. There should be no difference in location between the left and right sides.

116. You are incising and draining a perianal abscess and the consultant asks you about the anal canal and its anatomical relations. How would you best describe the canal? The canal lies above the levator ani muscle« YOUR ANSWER The canal has a longitudinal muscular coat

------Dr Mohammed Shamsul Islam Khan, Medical Officer, Clinical Neuro-Surgery, National Institute of Neuro-Sciences and Hospital Sher-E-Bangla Nagar, Dhaka-1207, Bangladesh. Mobile: +880 1713 455 662, E-mail: [email protected]

MyPasTest: MRCS A Online - Jan Exam 2015 03. Anatomy; Aabdomen, Pelvis & Perineum (290Qs) ------The canal has a lymphatic drainage via the inguinal lymph nodes« CORRECT ANSWER The canal has an external sphincter innervated by sympathetic fibres from the pelvic plexus The canal is lined by columnar epithelium below the dentate line.

The levator ani forms part of the deep external anal sphincter and the canal lies below this muscle. The Page | anal canal has no longitudinal muscular coat. Lymph from the lower anal canal drains via the superficial inguinal nodes. The external anal sphincter is innervated by the inferior rectal branch of the pudendal 171 nerve (S2–S4), the internal sphincter receives a sympathetic supply from the pelvic plexus. The upper anal canal is thrown into vertical folds called anal columns. Above the dentate line the canal is lined by columnar epithelium, below this level this changes to squamous epithelium.

117. The first arterial branch or branches arising from the abdominal aorta is (are)? Single best answer question – choose ONE true option only. The renal arteries« YOUR ANSWER The right adrenal artery The left adrenal artery The inferior phrenic arteries« CORRECT ANSWER The superior mesenteric artery.

The abdominal aorta is a continuation of the thoracic aorta, it begins at the aortic hiatus in the diaphragm between T12/L1 and ends at L4 by dividing into two common iliac arteries. The first arteries to come off the aorta are the inferior phrenic arteries which arise just inferior to the diaphragm and pass superolaterally over the crura of the diaphragm.

118. You are treating a patient with portal hypertension. The most likely location of the anastamosis responsible for variceal bleeding is? Single best answer - select ONE true answer only. Between the left gastric veins and the azygos system« YOUR ANSWER (Correct Answer) Between portal system and anterior abdominal wall veins Between the superior rectal vein and the internal pudendal vein At the bare area of the liver In the falciform ligament. All of the above are portosystemic anastamoses. In the gastrooesophageal region, the oesophageal tributaries of the left gastric vein anastomose with the oesophageal vein, which empties into the azygos vein. This is the most likely site of variceal bleeding.

In the paraumbilical region the paraumbilical veins in the falciform ligament anastomose with the subcutaneous veins in the anterior abdominal wall. In the anorectal region, the superior rectal vein anastomoses with the middle and inferior rectal veins, which are anastomoses of the internal iliac and internal pudendal veins. In the bare area of the liver anastomosis are via tributaries of the phrenic vein.

119. A 75-year-old gentleman undergoes an elective open abdominal aortic aneurysm repair. The aneurysm is infra-renal. In order to expose the aorta and the aneurysm, the vascular ------Dr Mohammed Shamsul Islam Khan, Medical Officer, Clinical Neuro-Surgery, National Institute of Neuro-Sciences and Hospital Sher-E-Bangla Nagar, Dhaka-1207, Bangladesh. Mobile: +880 1713 455 662, E-mail: [email protected]

MyPasTest: MRCS A Online - Jan Exam 2015 03. Anatomy; Aabdomen, Pelvis & Perineum (290Qs) ------surgeon needs to mobilize the third part of the duodenum. Which of the following is the anterior relation of the third part of the duodenum? Single best answer - select one answer only. Coeliac artery« YOUR ANSWER Head of the Pancreas Right Poses muscle Page | Right renal artery 172 Superior mesenteric vessels« CORRECT ANSWER.

The third part of the duodenum lies in the subcostal plane (L3) and follows the inferior margin of the pancreatic head. The third part of the duodenum lies posterior to the superior mesenteric vessels. It overlies the aorta and the beginning of the inferior mesenteric artery.

The root of the small bowel mesentery is attached near its termination on the left. The second part of the duodenum overlies the right ureter and renal vessels in the hilum of the right kidney, separated from the renal artery by the vein. The transpyloric plane is a guide to the surface marking of the duodenum.

120. During a 5-yearly follow-up for colorectal cancer, a CT scan of the abdomen in a 50-year-old woman reveals an 3.5 cm incidentaloma. On further assessment in the MDT, the incidentaloma represents a right adrenal gland mass. What is the best next step in management? Assessment of function« YOUR ANSWER Fine needle aspiration Laparoscopic adrenalectomy MRI« CORRECT ANSWER Yearly follow-up.

The left suprarenal vein is longer than the right, entering the left renal vein. Both glands receive an arterial supply directly from the aorta, as well as from the renal and inferior phrenic arteries. The right suprarenal gland is pyramidal in shape; the left is crescentic. The anterior surface of the right suprarenal gland is overlapped medially by the inferior vena cava.

Incidentalomas of the adrenal gland are diagnosed in up to 4.5% of patients undergoing imaging for other reasons. The first step is to determine whether the lesion is a metastasis. Once excluded it is necessary to assess the functional status of the lesion. This requires the measurement of 24-h urinary catecholamine and metanephrine levels, 9.00am and midnight serum cortisol and aldosterone levels. The incidence of primary malignancy increases with size. Surgery is now recommended for lesions > 3cm in size in patients under 50 years of age due to the development of laparoscopic adrenalectomy. MRI is useful in distinguishing benign or secondary malignant deposits. Fine needle aspiration is of limited value. It is extremely important to assess the lesion for phaeochromocytoma as this has bearing on preoperative preparation such that hypertensive crisis can be prevented.

121. A three-week-old boy presents with non-bilious vomiting, hypochloraemic metabolic alkalosis and hypokalaemia. On examination there is an epigastric mass. What is the most likely diagnosis?

------Dr Mohammed Shamsul Islam Khan, Medical Officer, Clinical Neuro-Surgery, National Institute of Neuro-Sciences and Hospital Sher-E-Bangla Nagar, Dhaka-1207, Bangladesh. Mobile: +880 1713 455 662, E-mail: [email protected]

MyPasTest: MRCS A Online - Jan Exam 2015 03. Anatomy; Aabdomen, Pelvis & Perineum (290Qs) ------Bacterial gastroenteritis« YOUR ANSWER Gastro-oesophageal reflux disease Obstructed umbilical hernia Pyloric stenosis« CORRECT ANSWER Page | Wilms’ tumour. 173 The obstruction in pyloric stenosis is proximal to the sphincter of Oddi, hence the vomitus is not bile stained. With persistent vomiting, metabolic alkalosis results from loss of hydrochloric acid. Hypokalaemia can result as the renal compensation to preserve sodium gives rise to increased potassium loss in the urine as well as loss of potassium-rich fluid in the vomitus.

The diagnosis can be made by a test feed – an ultrasound scan or barium meal will confirm the gastric outlet obstruction. Pyloric stenosis requires a pyloromyotomy, Ramstedt's operation; it has a polygenic mode of inheritance and occurs more commonly in males. Wilms‟ tumour is a nephroblastoma of the kidney.

122. You review a 54-year-old man with a history of alcoholism; you are concerned that there may be evidence of portal hypertension. Which of the following stems best describes blood flow to the liver? Single best answer question – choose ONE true option only. The hepatic artery supplies 75% of the total liver blood flow« YOUR ANSWER The normal portal vein pressure is 8–12 mmHg The portal vein supplies 75% of liver blood flow« CORRECT ANSWER The caudate lobe of the liver does not have it’s own branch of the hepatic vein Only the portal vein enters the liver via the porta hepatis.

The blood supply of the liver is around 25% of the resting cardiac output and arises from two main vessels. The hepatic artery, a branch of the coeliac axis supplies 25% of the total liver blood flow. Autoregulation of blood flow by the hepatic artery ensures constant liver blood flow.

The portal vein drains most of the gastrointestinal (GI) tract and the spleen and constitutes 75% of liver blood flow. Normal portal pressure is 5–8 mmHg, but blood flow increases after meals. Both vessels enter the liver via the porta hepatis. The caudate lobe receives an independent blood supply from the hepatic portal vein and artery and its branch of the hepatic vein drains directly into the inferior vena cava. 123. A 38-year-old female with a one year history of intermittent epigastric pain presents with vomiting and abdominal distension. A plain abdominal radiograph reveals dilated loops of small bowel and aerobilia. What is the most appropriate management? Emergency laparotomy« YOUR ANSWER (Correct) Endoscopic retrograde cholangiopancreatography Laparoscopic cholecystectomy Percutaneous drainage of the gall bladder

------Dr Mohammed Shamsul Islam Khan, Medical Officer, Clinical Neuro-Surgery, National Institute of Neuro-Sciences and Hospital Sher-E-Bangla Nagar, Dhaka-1207, Bangladesh. Mobile: +880 1713 455 662, E-mail: [email protected]

MyPasTest: MRCS A Online - Jan Exam 2015 03. Anatomy; Aabdomen, Pelvis & Perineum (290Qs) ------Surgical closure of a cholceystoduodenal fistula.

This patient has a small bowel obstruction secondary to gallstone ileus. This is a surgical emergency requiring urgent laparotomy. The gallstone usually impacts proximal to the ileocaecal valve, and so it should be massaged through into the large intestine or extracted via an enterotomy, thus relieving the obstruction. Page |

The gallstone will have passed into the small bowel through a biliary-enteric fistula, but this 174 does not usually require surgical repair. Such fistulae often lead to aerobilia, which can be seen on plain abdominal radiographs.

124. You are examining a patient for an inguinal hernia. Which of the following is correct regarding the inguinal rings? Single best answer - choose ONE true answer only. The deep inguinal ring lies 5 cm above the midinguinal point« YOUR ANSWER The deep inguinal ring lies 1.5 cm above the midpoint of the inguinal ligament« CORRECT ANSWER The deep inguinal ring lies 5 cm above the midpoint of the inguinal ligament The superficial inguinal ring lies 1.5cm above the midpoint of the inguinal ligament The superficial inguinal ring lies 5cm above the midpoint of the inguinal ligament.

The inguinal canal is about 4 cm long. The posterior wall of the canal has the conjoint tendon medially and the transversalis fascia throughout. The deep inguinal ring lies about 1.5 cm above the mid-point of the inguinal ligament.

The floor of the inguinal canal is the unrolled lower edge of the inguinal ligament, re-inforced medially by the lacunar ligament. The midinguinal point is midway between the anterior superior iliac spine and the symphysis pubis.

125. The transpyloric plane: Crosses the right costal margin at the tip of the tenth costal cartilage, the surface marking of the gall-bladder fundus« YOUR ANSWER Defines the level at which the coeliac axis leaves the aorta Is the plane where the portal vein is formed« CORRECT ANSWER Lies at the level of the second lumbar vertebra Lies mid-way between the manubrium sterni and the pubic symphysis. The transpyloric plane is a convenient way to relate anatomical structures. It is an imaginary transverse plane with a surface marking mid-way between the jugular notch and the pubic symphysis. This corresponds to the level of the first lumbar vertebra.

The surface marking of the fundus of the gall-bladder is at its junction with the right ninth costal cartilage. It represents the point at which the superior mesenteric artery leaves the aorta and where the splenic and superior mesenteric veins join to form the portal vein.

------Dr Mohammed Shamsul Islam Khan, Medical Officer, Clinical Neuro-Surgery, National Institute of Neuro-Sciences and Hospital Sher-E-Bangla Nagar, Dhaka-1207, Bangladesh. Mobile: +880 1713 455 662, E-mail: [email protected]

MyPasTest: MRCS A Online - Jan Exam 2015 03. Anatomy; Aabdomen, Pelvis & Perineum (290Qs) ------126. A 14-year-old male presents with a one hour history of severe testicular pain and is rushed to theatre for an emergency scrotal exploration. The testis appears to be torted and is repaired in the usual fashion. At closure, the surgeon is satisfied that normal perfusion has been restored. The patient has an increased risk of which condition? Single best answer - select one answer only. Chronic groin pain« YOUR ANSWER Page | Increased risk of cancer in the contralateral testis« CORRECT ANSWER 175 Inguino-scrotal hernia Infertility Varicocoele.

Testicular torsion is a urological emergency that can occur at any age, although it most commonly occurs during adolescence. It is a clinical diagnosis, and a low threshold for scrotal exploration should be used. Evidence exists to indicate that, following a torsion, there is an increased risk of testicular cancer in both the ipsilateral and contralateral testis.

127. A 76-year-old lifelong smoker has presented to the Emergency Department with haematemesis. Following stabilisation, an OGD has been performed with the presumed diagnosis of a gastric ulcer. Unfortunately a large polypoid mass was found in the distal oesophagus adjacent to the gastrooesophageal junction with evidence of a co-existent hiatus hernia. Biopsy confirmed the diagnosis of oesophageal carcinoma. Which of the following confers the highest relative risk of developing this condition? Single best answer - select one answer only. Barrett's oesophagus« YOUR ANSWER (Correct) Diverticular disease of the oesophagus Zinc deficiency Achalasia Helicobacter pylori infection.

Barrett's oesophagus is associated with a risk of 13% of developing oesophageal adenocarcinoma and requires regular endoscopic surveillance. Other risk factors include corrosive strictures, HPV infection, Tyelosis, Coeliac disease (squamous cell), Plummer–Vinson syndrome, tobacco smoking, alcohol and vitamin C deficiency. Helicobacter pylori infection is associated with gastric cancer. 128. A 30-year-old gentleman with ulcerative colitis attends the coloproctology clinic following a Gastroenterology referral for restorative proctolcolectomy. Which of the following is most accurate regarding restorative proctolcolectomy? Pouch failure is likely in patients with primary sclerosing cholangitis« YOUR ANSWER Pouch surveillance is mandatory in all cases of ulcerative colitis Pouchitis is common with restorative procotocolectomy Restorative proctolcolectomy is a three-stage procedure

------Dr Mohammed Shamsul Islam Khan, Medical Officer, Clinical Neuro-Surgery, National Institute of Neuro-Sciences and Hospital Sher-E-Bangla Nagar, Dhaka-1207, Bangladesh. Mobile: +880 1713 455 662, E-mail: [email protected]

MyPasTest: MRCS A Online - Jan Exam 2015 03. Anatomy; Aabdomen, Pelvis & Perineum (290Qs) ------Restorative proctolcolectomy is contra-indicated in patients who have an internal anal sphincter defect.

Incompetent ileocaecal valves are not uncommon and can lead to a „backwash‟ ileitis, making distinction from Crohn‟s or a mixed colitis difficult. Remember, ulcerative colitis (UC) starts as a proctitis and ascends in a continuous fashion. UC has many extra-intestinal manifestations, of Page | which uveitis is one. It is associated to the HLA-B27 phenotype. UC by definition is generally restricted to the colon and so sparing the small bowel. 176

Malabsorption is a definite problem with Crohn‟s disease. Restorative proctolcolectomy is a procedure by which a pouch from terminal ileum is fashioned and anastomosed to the anus, 1- 2cm above the dentate line. The pouch is usually J in shape and measures 15cm in length. RPC is recommended in ulcerative colitis and FAP where the colon needs to be removed to eliminate the risk of colon cancer.

The incidence of pouch failure is 5-15 % per year and reasons include pelvic sepsis, poor function and pouchitis. Pouch surveillance is recommended for those who have colonic dysplasia or neoplasm, as there are rare case reports of neoplasm developing within the pouch. Ideal pouch function is approximately 4-6 times per day and two times at night.

The benefit of the pouch is to eliminate urgency and avoid a permanent ileostomy. RPC should be a one stage or two stage operation (if a defunctioning stome is employed).

129. During an inguinal hernia repair in a 54-year-old man, the ilioinguinal nerve is injured in the inguinal canal. This will most likely result in which of the following? Single best answer question – choose ONE true option only. Paraesthesia over the dorsum of the penis« YOUR ANSWER Paraesthesia over the pubis and scrotum, and loss of cremasteric contraction Paraesthesia over the pubis and anterior scrotum only« CORRECT ANSWER Paraesthesia over the anterior and medial thigh Paraesthesia over the pubis only.

An injury to the ilioinguinal nerve within the inguinal canal may result in parasthesia over the pubis and anterior scrotum. The ilioinginal nerve (L1) passes through the internal oblique muscle to enter the inguinal canal laterally. It passes anterior to the cord and exits the superficial ring to provide sensation to the mons pubis and anterior scrotum in the male and labia majora in the female. 130. A 26-year-old man is admitted with a painful right shoulder following a high speed RTA. Plain radiographs reveal a fractured scapula. He appears systemically well but undergoes a full body CT scan as the casualty registrar quite correctly recognises that this fracture carries a high risk of concomitant soft tissue injury. In which of the following bones does a fracture indicate high energy trauma which could be associated with hidden severe injuries? Femur« YOUR ANSWER Clavicle Tibia

------Dr Mohammed Shamsul Islam Khan, Medical Officer, Clinical Neuro-Surgery, National Institute of Neuro-Sciences and Hospital Sher-E-Bangla Nagar, Dhaka-1207, Bangladesh. Mobile: +880 1713 455 662, E-mail: [email protected]

MyPasTest: MRCS A Online - Jan Exam 2015 03. Anatomy; Aabdomen, Pelvis & Perineum (290Qs) ------10th rib 1st rib« CORRECT ANSWER.

The 1st rib, sternum and scapula are both notoriously difficult to break but also act as guardians to the underlying soft tissue structures which are otherwise relatively well protected. If these bony structures are breached, it is assumed that there may be underlying damage to the Page | cardiopulmonary apparatus or great vessels. The other bones mentioned also require force to break them but tend not to be associated with underlying injury. 177

131. A 69-year-old man with recent onset fast atrial fibrillation presents to A&E with acute, severe periumbilical abdominal pain. Arterial blood gas reveals a raised lactate. CT-Scanning confirms extensive ischaemic small bowel. The patient is taken to theatre for a laparotomy and small bowel resection. Which feature best differentiates the upper jejunum from the lower ileum? Single best answer - select one answer only. A thinner wall« YOUR ANSWER Fewer circular folds Less fat at the mesenteric border« CORRECT ANSWER More aggregated lymphatic follicles (Peyer’s patches) More arterial arcades.

The jejunum has a thicker wall, less mesenteric fat, more plicae circulares, a wider lumen, fewer Peyer‟s patches and fewer arterial arcades than the ileum.

132. Intussusception is most common in which of the following? Single best answer - select one answer only. Females in the summer months« YOUR ANSWER Females in the winter months Females regardless of the time of year Males in the summer months Males in the winter months« CORRECT ANSWER.

Intussusception is the telescoping of one part of the intestine into the other and is most common in infants aged 4 monts to 1 year, with peak incidence between 6-9 months. It is more common in the winter months and in males. 133. A 72-year-old male presents with macroscopic haematuria and is found to have a transitional cell carcinoma in the posterior wall of the bladder. Staging investigations suggest tumour invasion into a retrocystic structure. Which structure has been invaded? Single best answer - select one answer only. Anterior portion of prostate gland« YOUR ANSWER Bulbospongiosus Corpus spongiosum Perineal body ------Dr Mohammed Shamsul Islam Khan, Medical Officer, Clinical Neuro-Surgery, National Institute of Neuro-Sciences and Hospital Sher-E-Bangla Nagar, Dhaka-1207, Bangladesh. Mobile: +880 1713 455 662, E-mail: [email protected]

MyPasTest: MRCS A Online - Jan Exam 2015 03. Anatomy; Aabdomen, Pelvis & Perineum (290Qs) ------Seminal vesicles« CORRECT ANSWER.

In males, the seminal vesicles lie in the rectovesical pouch, posterior to the posterior wall of the bladder. They are at risk of local invasion from posterior wall bladder tumours.

134. A 38-year-old woman was seen in the out-patient clinic complaining of a boil located on her Page | labia majora. Lymphatic spread of the infection would most likely enlarge which nodes? Single best answer question – choose ONE true option only. 178 Lumbar« YOUR ANSWER Sacral External iliac Superficial inguinal« CORRECT ANSWER Internal iliac.

The perineum and the external genitalia, including the labia majora and scrotum, drain to the superficial inguinal lymph nodes. However, in the male, remember that the testes do not drain to the superficial inguinal lymph nodes! The lymphatic vessels for testes travel in the spermatic cord and drain the testes into the lumbar nodes (ovaries also drain to lumbar nodes). The lumbar nodes drain the internal pelvic organs; the sacral nodes drain the prostate gland, uterus, vagina, rectum and posterior pelvic wall; the external iliac nodes drain the lower limb; the internal iliac nodes drain the pelvis and gluteal region.

135. You are performing a testicular examination on a 27-year-old, white patient whose father was previously diagnosed with a teratoma and you discover a lump. Given this family history how much more likely is it that this patient has a testicular tumour than a patient without this history? Single best answer - select one answer only. Family history is not a relevant factor« YOUR ANSWER 4 times more likely« CORRECT ANSWER 5 times more likely 9 times more likely 10 x more likely. A teratoma is the most likely testicular tumour to occur in the 20–30 age group and seminomas are more common in the 30–40 age group. A testicular tumour is more common following torsion on both the ipsilateral and contralateral sides.

It is also more common with undescended testes. Testicular tumours are 5 times more common in white men than black men. Having a first degree relative who has had a teratoma increases the risk by 4 times for a father and 9 times for a brother.

136. A 49-year old man develops an abscess in the lesser sac following an episode of severe pancreatitis. He undergoes laparotomy for washout of the abscess. The lesser omentum is incised to access the lesser sac. Which of the following best describes the anatomy of the lesser omentum? Encloses the right gastric vessels« YOUR ANSWER (Correct)

------Dr Mohammed Shamsul Islam Khan, Medical Officer, Clinical Neuro-Surgery, National Institute of Neuro-Sciences and Hospital Sher-E-Bangla Nagar, Dhaka-1207, Bangladesh. Mobile: +880 1713 455 662, E-mail: [email protected]

MyPasTest: MRCS A Online - Jan Exam 2015 03. Anatomy; Aabdomen, Pelvis & Perineum (290Qs) ------Has the left hepatic duct in its free edge Is attached to the liver in the fissure of the ligamentum teres Is attached to the third part of the duodenum Is supplied by gastroepiploic arteries. Page | The right and left gastric arteries supply the lesser omentum as they lie between its two peritoneal layers. The free edge of the lesser omentum is attached to the first 2 cm of the first 179 part of the duodenum below and the fissure of the above. The common hepatic duct is joined by the cystic duct to form the common bile duct in the free edge of the lesser omentum.

137. A 70-year-old woman attends the clinic with a history of right upper quadrant pain. She has an ultrasound scan of the abdomen that reveals a normal liver, a common bile duct diameter of 7mm and multiple gallstones within a contracted thick-walled gallbladder. Which of the following statements is true regarding this patient? A laparoscopic cholecystectomy should be recommended« YOUR ANSWER (Correct) Gallbladder carcinoma can be excluded She has acute cholecystitis She has Mirizzi’s syndrome which is a gallstone that is eroding through the gallbladder wall into the ileum The common bile duct is dilated.

Gallbladder contraction results from the effects of the neurohormone cholecystokinin (CCK). Six millimetres is the upper limit of normal for common bile duct diameter, and allowances of a few millimetres more may be made in elderly patients. Mirizzi‟s syndrome is obstructive jaundice caused by a large gallstone in Hartmann‟s pouch compressing the common bile duct.

Mirizzi‟s syndrome is one of the indications to convert from laparoscopic cholecystectomy to an open procedure, often leaving Hartmann‟s pouch behind to fibrose. Iatrogenic causes of gas in the biliary tract may often be overlooked; endoscopic retrograde cholangiopancreatography (ERCP) is a common cause.

138. The cremasteric muscle is an extension of the: Single best answer question – choose ONE true option only. External abdominal oblique muscle« YOUR ANSWER Transverse abdominal muscle Internal abdominal oblique muscle« CORRECT ANSWER Pyramidalis muscle Dartos muscle.

The cremaster is a thin muscular layer that is composed of a number of fasciculi, which take origin from the middle of the inguinal ligament. At its origin its fibres are continuous with those of the internal oblique and also occasionally with the transverses abdominis. It passes along the ------Dr Mohammed Shamsul Islam Khan, Medical Officer, Clinical Neuro-Surgery, National Institute of Neuro-Sciences and Hospital Sher-E-Bangla Nagar, Dhaka-1207, Bangladesh. Mobile: +880 1713 455 662, E-mail: [email protected]

MyPasTest: MRCS A Online - Jan Exam 2015 03. Anatomy; Aabdomen, Pelvis & Perineum (290Qs) ------lateral side of the spermatic cord and descends with it through the superficial inguinal ring on the front and sides of the cord.

It forms a series of loops that differ in thickness and length in different people. At the upper part of the cord the loops are short, but they become successively longer and longer, the longest reaching down as low as the testis, where a few are inserted into the tunica vaginalis. These loops are united together by areolar tissue and form a thin covering over the cord and testis Page | called the cremasteric fascia. 180 The fibres ascend along the medial side of the cord and are inserted by a small pointed tendon into the pubic tubercle and crest, as well as into the front of the sheath of the rectus abdominis.

139. A 47-year-old male motorcyclist is involved in a road traffic accident and sustains direct blunt trauma to his pelvis, causing a fracture of the ischiopubic ramus. Which layer of the abdominal wall is contiguous with Colles‟ fascia? Single best answer - select one answer only. Anterior rectus sheath« YOUR ANSWER Camper’s fascia Posterior rectus sheath Scarpa’s fascia« CORRECT ANSWER Transversalis fascia.

Colles‟ fascia forms the superficial fascial boundary of the superficial perineal space and attaches to the margin of the ischiopubic ramus. The fascia is continuous with the dartos fascia of the scrotum, with the superficial penile fascia (that covers the clitoris and penis), and with Scarpa‟s fascia of the abdominal wall.

140. A 67-year-old male presents with painless obstructive jaundice, weight loss and general malaise. He undergoes computerised tomography which confirms the presence of a pancreatic mass. He proceeds to surgery. Which portion of the pancreas lies between the abdominal aorta and the superior mesentric artery? Single best answer - select one answer only. Body« YOUR ANSWER Head Neck Tail Uncinate process« CORRECT ANSWER.

The uncinate process is a curved extension of the pancreatic head, and is bound by the superior mesenteric vessels anteriorly and the abdominal aorta posteriorly. The superior mesenteric artery passes posterior to the pancreatic neck as it arises from the abdominal aorta.

141. A 32-year-old man has been brought in following a high speed RTA. He was the passenger having been shunted from behind. The airbag failed to deploy and he has sustained a blunt injury to the right hypochondrium from the steering column. He is haemodynamically stable with bruising and tenderness to the right upper quadrant of his abdomen. Which of the following is true regarding blunt liver injury? Single best answer - select one answer only. ------Dr Mohammed Shamsul Islam Khan, Medical Officer, Clinical Neuro-Surgery, National Institute of Neuro-Sciences and Hospital Sher-E-Bangla Nagar, Dhaka-1207, Bangladesh. Mobile: +880 1713 455 662, E-mail: [email protected]

MyPasTest: MRCS A Online - Jan Exam 2015 03. Anatomy; Aabdomen, Pelvis & Perineum (290Qs) ------Must always be suspected if there is a blunt or penetrating injury to the lower chest« YOUR ANSWER (Correct) Should be initially investigated by MRI scanning Is rarely associated with shoulder tip pain With haemorrhage can be temporarily controlled by Kocher’s manoeuvre Page | Is associated with 10% mortality when major liver resection is required. 181

The liver is in danger of injury by virtue of its anatomical location, and when injured can lead to irritation of the diaphragm with referred pain to the shoulder. During torrential haemorrhage at laparotomy, pinching of the free edge of the lesser sac containing the hepatic artery and portal vein can temporarily slow down bleeding.

This is Pringle‟s manoeuvre. Kocher‟s manoeuvre is used to expose retroperitoneal structures. In the vast majority of cases conservative treatment is used for liver injuries; however, major liver fractures will require resection of liver and should normally be performed in centres with experience; mortality however is high and in the region of 60%.

142. Concerning the portal vein: Single best answer question – choose ONE true option only. It carries blood from three major veins, the superior and inferior mesenteric vein and the left gastric vein« YOUR ANSWER It is formed posterior to the head of the pancreas It lies posterior to the bile duct in the free edge of the lesser omentum« CORRECT ANSWER At the porta hepatis the portal vein divides into four branches It lies anterior to the hepatic artery in the free edge of the lesser omentum.

The portal vein is the main channel of the portal system of veins. It carries blood from three major veins: the superior and inferior mesenteric veins and the splenic veins. The portal vein is formed posterior to the neck of the pancreas.

The portal vein ascends in the free edge of the lesser omentum to the liver (posterior to the hepatic artery and bile duct). At the porta hepatis the portal vein divides into right and left branches, which empty their blood into the hepatic sinusoids. 143. An 18-year-old Afro-Caribbean man presents to the emergency department with a painful erection which has been present for 8 hours. Which of the following statements best represents this condition? Single best answer - select one answer only. Is probably caused by idiopathic sacral nerve root damage« YOUR ANSWER Is treated with sildenafil Direct pressure and ice is the usual treatment May be treated with vasodilators Occurs in sickle-cell crisis« CORRECT ANSWER.

------Dr Mohammed Shamsul Islam Khan, Medical Officer, Clinical Neuro-Surgery, National Institute of Neuro-Sciences and Hospital Sher-E-Bangla Nagar, Dhaka-1207, Bangladesh. Mobile: +880 1713 455 662, E-mail: [email protected]

MyPasTest: MRCS A Online - Jan Exam 2015 03. Anatomy; Aabdomen, Pelvis & Perineum (290Qs) ------A priapism is a prolonged painful erection which has been present for over 4 hours in the absence of physical of psychological stimulation. It can occur in the penis or clitoris.

Prolonged, painful erection may be physiological or occur in leukaemia, sickle-cell disease, pelvic malignancy, spinal cord injury, haemodialysis and after injection of vasodilators for impotence. Relating to cord injury, it is caused by damage to the cervival cord (eg death erection in hanging victims). Page |

Treatment includes vasoconstrictors and aspiration. Direct pressure and ice may help reduce 182 the swelling but is more frequently used as a treatment for paraphimosis. Sildenafil is used for impotence.

144. A 50-year-old woman has a suspected right renal artery stenosis. A transfemoral aortogram has been performed. Which one of the following is likely to be correct? Single best answer question – choose ONE true option only. The femoral artery at the groin is situated halfway between the anterior superior iliac spine and the pubic tubercle« YOUR ANSWER The catheter passes through the common femoral artery into first the external iliac artery and then the aorta at its bifurcation The right renal artery also gives off the right ovarian and suprarenal arteries The right and left renal arteries originate from the abdominal aorta between the levels of the first and second lumbar vertebrae« CORRECT ANSWER The aorta passes through the diaphragm at the level of the tenth thoracic vertebra.

The femoral artery at the groin constantly lies halfway between the anterior superior iliac spine and the midline (the pubic symphysis). The catheter continues proximally through the external and then the common iliac artery to reach the aorta.

The aorta gives off the suprarenal and the gonadal (ovarian or testicular) arteries as separate branches, respectively above and below the origins of the renal arteries on each side. The aorta gives off its renal branches between L1 and L2, and passes through the diaphragm at T12.

145. A one-week-old male presents with non-bilious vomiting, palpable abdominal mass and irritability. Which metabolic disturbance is most likely to be found at presentation? Single best answer - select one answer only. Hyperkalaemia« YOUR ANSWER Hypocalcaemia Hypomagnasaemia Metabolic acidosis Metabolic alkalosis« CORRECT ANSWER.

The obstruction in pyloric stenosis is proximal to the sphincter of Oddi, hence the vomitus is not bile stained. With persistent vomiting, metabolic alkalosis results from loss of hydrochloric acid. Hypokalaemia can result as the renal compensation to preserve sodium gives rise to increased potassium loss in the urine as well as loss of potassium-rich fluid in the vomitus.

------Dr Mohammed Shamsul Islam Khan, Medical Officer, Clinical Neuro-Surgery, National Institute of Neuro-Sciences and Hospital Sher-E-Bangla Nagar, Dhaka-1207, Bangladesh. Mobile: +880 1713 455 662, E-mail: [email protected]

MyPasTest: MRCS A Online - Jan Exam 2015 03. Anatomy; Aabdomen, Pelvis & Perineum (290Qs) ------

The diagnosis can be made by a test feed – an ultrasound scan or barium meal will confirm the gastric outlet obstruction. Pyloric stenosis requires a pyloromyotomy, Ramstedt's operation; it has a polygenic mode of inheritance and occurs more commonly in males.

146. A 75-year-old gentleman undergoes a transurethral resection for benign prostatic hyperplasia. The procedure takes over two hours as he has a large prostate. The surgeon uses Page | Glycine as the irrigating fluid. In the postoperative period, he becomes confused. What is the most likely reason for this? Single best answer - select one answer only. 183 Hyponatraemia« YOUR ANSWER (Correct) Hypotension Pulmonary oedema Septicaemia Transient ischaemic attack.

Ejaculatory ducts enter the upper posterior part of the prostate gland to open into the urethra. Laterally lies the levator ani. The arterial supply to the prostate is derived from the inferior vesical artery, with some small branches from the middle rectal and internal pundendal.

The prostate is surrounded by a venous plexus, which drains into the internal iliac vein and subsequently into the internal vertebral venous plexus. Hyponatraemia can occur due to the irrigation fluid used during TURP. It is a cause of postoperative confusion following TURP. Hypertonic Saline should be administered to correct the hyponatraemia.

The irrigating fluid causes an osmotic diuresis and also diffuses into cells where it is metabolized and causes intracellular oedema. Other complications from TURP include septicaemia in 1.5%, mortality in 0.5%, blood loss needing transfusion in 0.4-6.4%, erectile dysfunction in 10%, retrograde ejaculation in 70%, incontinence in 1%, bladder neck stenosis in 2% and urethral stricture in1%. Up to 2% of patients per year require re-operation.

147. Which of the following is correct concerning the blood supply to the foregut? Single best answer - select one answer only. The gastroduodenal artery passes anterior to the first part of the duodenum« YOUR ANSWER The hepatic artery usually divides into its left and right branches in the free edge of the lesser omentum The main blood supply to the pancreas is the common hepatic artery The short gastric arteries are found within the gastrosplenic ligament« CORRECT ANSWER The superior and inferior pancreaticoduodenal arteries are branches of the gastroduodenal artery.

The coeliac axis has three main branches: the left gastric, the splenic and the common hepatic arteries. The splenic artery is the main blood supply to the pancreas. Its short gastric branches are found within the gastrosplenic ligament. The common hepatic gives off the gastroduodenal artery, which passes behind the first part of the duodenum (and so may be eroded into by a duodenal ulcer).

------Dr Mohammed Shamsul Islam Khan, Medical Officer, Clinical Neuro-Surgery, National Institute of Neuro-Sciences and Hospital Sher-E-Bangla Nagar, Dhaka-1207, Bangladesh. Mobile: +880 1713 455 662, E-mail: [email protected]

MyPasTest: MRCS A Online - Jan Exam 2015 03. Anatomy; Aabdomen, Pelvis & Perineum (290Qs) ------The common hepatic artery then continues in the free edge of the lesser omentum as the hepatic artery and passes up towards the porta hepatis. Here is it divides into its left and right branches. The inferior pancreaticoduodenal artery is a branch of the superior mesenteric system.

148. A 36-year-old female is struck by a car and is brought to the emergency department complaining of abdominal pain. On examination, there is tenderness and brusing and so Page | diagnostic peritoneal lavage is performed. During the procedure, 15 millilitres of blood is aspirated from the peritoneal cavity. What is the most appropriate management? 184 Computerised tomography of the abdomen and pelvis« YOUR ANSWER Emergency laparotomy« CORRECT ANSWER Focussed assessment with sonography for trauma (FAST) Emergency arteriography Repeat diagnostic peritoneal lavage within 24 hours.

Diagnostic peritoneal lavage has largely been superceded by FAST. However, it is occasionally used when such technology is unavailable. Positive indications for laparotomy are more than 10mls blood aspirated, aspiration of bile, stool or enteric content and leak of fluid into chest drains.

149. Which of the following is derived from the mesonephric ducts? Single best answer - select one answer only. Prostate« YOUR ANSWER Falloian tubes Uterus Vagina Vas deferens« CORRECT ANSWER.

The embryological derivatives of the mesonephric duct and paramesonephric duct are listed as follows: MESONEPHRIC DUCT (Wolffian Duct): Male structures formed from mesonephric duct: Epididymis Vas deferens Ureter Renal pelvis, calyces and collecting tubules Ejaculatory duct Seminal vesicle

Female structures formed from the mesonephric duct: Ureter Renal pelvis, calyces, and collecting tubules

PARAMESONEPHRIC DUCT (Mullerian Duct): Male:

------Dr Mohammed Shamsul Islam Khan, Medical Officer, Clinical Neuro-Surgery, National Institute of Neuro-Sciences and Hospital Sher-E-Bangla Nagar, Dhaka-1207, Bangladesh. Mobile: +880 1713 455 662, E-mail: [email protected]

MyPasTest: MRCS A Online - Jan Exam 2015 03. Anatomy; Aabdomen, Pelvis & Perineum (290Qs) ------No major structures in the male

Structures formed in the female: Fallopian tubes Uterus Upper portion of vagina Vaginal wall Page |

150. A 56-year-old nervous, obese lady presents to your accident and emergency with mild 185 upper abdominal pain and symptoms of an acidic taste in the mouth occurring several times per day over the past 2 years. She states her pain is worse after eating. She is haemodynamically stable and has normal observations. Which of the following most accurately describes this lady‟s probable condition? Single best answer - select one answer only. It is usually treated by highly selective vagotomy in cases where trials of medication have failed« YOUR ANSWER It is usually worsened by smoking which acts to contract the lower oesophageal sphincter and increase oesophageal acidity In cases such as this lifestyle modifying advice often has the desired effect« CORRECT ANSWER Prokinetic drugs such as metoclopramide should be used as the treatment of choice in this condition It leads to the development of Barrett’s metaplasia in 30% of cases.

Insufficient clearance of acid can contribute to the symptoms of gastro-oesophageal reflux disease (GORD), but this is not the most common problem which is incompetence of the lower oesophageal sphincter plus failure of the anatomical valve at the angle of His. Symptoms of water brash are classically worse at night and with obesity and smoking which reduces the competence of the lower oesophageal sphincter.

In patients with mild symptoms, the first option should be lifestyle modifications like losing weight, giving up smoking, not eating within 2 hours of bed and raising the head of the bed by 6-8 inches. If these do not work then proton pump inhibitors and H2 blockers should be tried before prokinetics. If surgical intervention is needed then the treatment of choice is a Nissen‟s Fundoplication. Vagotomy has largely been superceded by proton pump inhibitors. Barrett‟s oesophagus is seen in approximately 10% of patients undergoing endoscopy for investigation of GORD.

151. A 45-year-old female undergoes ileocaecectomy through a midline laparotomy incision. You are asked to close the abdominal wall. Which is the most appropriate suture for this task? Single best answer - select one answer only. Catgut« YOUR ANSWER Poliglecaprone monofilament Polydioxanone monofilament« CORRECT ANSWER Polyglactin braided Polypropylene monofilament.

------Dr Mohammed Shamsul Islam Khan, Medical Officer, Clinical Neuro-Surgery, National Institute of Neuro-Sciences and Hospital Sher-E-Bangla Nagar, Dhaka-1207, Bangladesh. Mobile: +880 1713 455 662, E-mail: [email protected]

MyPasTest: MRCS A Online - Jan Exam 2015 03. Anatomy; Aabdomen, Pelvis & Perineum (290Qs) ------Polydiaxonone is commonly used in loop form for closure of the abdominal wall. Polglecaprone is used for subcutaneous skin closure, polyglactin for soft tissue approximation and ligation, and polypropylene for vascular and other anastomoses. Catgut is rarely used in current practice, due to higher rates of post-operative infection.

152. When obtaining consent for transurethral resection of the prostate from a patient what are the procedure specific complications you would mention? Page | Bladder perforation, erectile dysfunction, incontinence, Haematuria, Retrograde ejaculation, UTI, 186 Urethral stricture« YOUR ANSWER Bladder perforation, erectile dysfunction, incontinence, Priapism, Retrograde ejaculation Bladder perforation, Priapism, deep vein thrombosis, Urethral stricture Deep vein thrombosis, pulmonary embolism, death Bladder perforation, erectile dysfunction, pulmonary embolism, retrograde ejaculation, death.

Procedure-specific complications for a TURP include: retrograde ejaculation, erectile dysfunction, incontinence, haematuria, UTI, urethral stricture and bladder perforation. When counselling a patient about a TURP other complications should also be discussed, such as deep vein thrombosis (DVT), pulmonary embolism (PE) and death (which occurs in 0.5–1% of cases).

153. An 18-year-old boy presents to the emergency department with priapism. It is extremely painful and the emergency doctor is not familiar with the condition and its treatment. He contacts you and enquires about his patient‟s problem and best treatment. Which of the following is most applicable to priapism? Likely to result in permanent erectile failure if present for >5 hours« YOUR ANSWER Priapism can follow injection of vasoactive drugs Priapism is associated with Thalassaemia Priapism may be treated by aspirating blood from the Corpora with a 21 gauge cannula« CORRECT ANSWER The corpus spongiosum is often involved.

Priapism, sustained painful penile erection, is commonly iatrogenic, occurring after the administration of intracavernosal agents used to treat erectile dysfunction. It may also occur in sickle cell disease and affects only the corpora cavernosa. Non-operative treatment includes aspiration, irrigation with heparanised saline and/or administration of intracavernosal metaraminol or phenylephrine. Surgical techniques include the formation of a caverno- spongiosal shunt. Impotence may result but is less likely if the condition is effectively treated.

154. The anal canal: Demonstrates an abrupt change in epithelial structure at the pectinate (dentate) line« YOUR ANSWER Has anal cushions lying distal to the pectinate (dentate) line Has circular and longitudinal muscle fibres ------Dr Mohammed Shamsul Islam Khan, Medical Officer, Clinical Neuro-Surgery, National Institute of Neuro-Sciences and Hospital Sher-E-Bangla Nagar, Dhaka-1207, Bangladesh. Mobile: +880 1713 455 662, E-mail: [email protected]

MyPasTest: MRCS A Online - Jan Exam 2015 03. Anatomy; Aabdomen, Pelvis & Perineum (290Qs) ------Has fibres from the sacral sympathetic outflow to cause internal sphincter contraction Is supplied by branches of the inferior mesenteric and internal iliac arteries« CORRECT ANSWER.

The anal canal has only circular muscle fibres (unlike the rest of the gastrointestinal (GI) tract which has both circular and longitudinal fibres), consisting of the internal and external anal sphincters. The mucosa in the upper anal canal consists predominately of columnar cells until Page | is reaches the pectinate line. 187 There is then a gradual change to the multi-layered squamous epithelium of the pecten. Anal cushions lie proximal to the pectinate line. The anal canal has a mixed arterial supply – the superior rectal artery to the upper third and the inferior rectal vessels to the lower third. Sympathetic fibres originating from the first two lumbar segments of the cord supply the internal sphincter – sacral sympathetic outflow does not exist.

155. A 40-year-old woman is involved in a head-on collision with a vehicle travelling at 40 miles per hour. She is a pedestrian and is thrown backwards by the car. On arrival at the scene, the emergency team intubate and ventilate her as well as placing her in cervical spine immobilization. In the resuscitation room, free fluid is noticed on focused abdominal sonography. The trauma team decide to do a diagnostic peritoneal lavage (DPL). Which of the following is true about DPL? A nasogastric (NG) tube is required« YOUR ANSWER (Correct) DPL is mandatory in traffic incidents such as described It is contraindicated in pregnancy Laparotomy is not necessary with a negative result The incision should be made a finger breadth above the pubic symphysis.

Diagnostic peritoneal lavage (DPL) is performed under local anaesthetic. The patient requires a nasogastric tube and catheter to deflate the stomach and bladder respectively. A vertical sub umbilical incision is performed and the linea alba is divided.

After incising the peritoneum, a peritoneal dialysis catheter is inserted. If no blood or gastric content is aspirated, 1 l of normal saline is introduced and allowed to equilibriate for 3 min.

Positive DPL is: 1. Rred cell count more than 100000/mm3 2. Wwhite cell count more than 500/mm3 3. Presence of bile, bacteria or faecal material.

Retroperitoneal injuries are not detected by DPL. Further a false positive result can occur if the DPL fluid becomes contaminated by bleeding from the incisions made for the procedure. For DPL, the incision needs to be made 1/3 of the way up a line drawn from the pubic symphysis to the umbilicus.

156. The ovum is the largest cell in the human body, typically visible to the naked eye without the aid of a microscope or other magnification device. The ova: Single best answer question – choose ONE true option only. ------Dr Mohammed Shamsul Islam Khan, Medical Officer, Clinical Neuro-Surgery, National Institute of Neuro-Sciences and Hospital Sher-E-Bangla Nagar, Dhaka-1207, Bangladesh. Mobile: +880 1713 455 662, E-mail: [email protected]

MyPasTest: MRCS A Online - Jan Exam 2015 03. Anatomy; Aabdomen, Pelvis & Perineum (290Qs) ------Are developed from the primitive germ cells, which are embedded in the substance of the ovaries« YOUR ANSWER (Correct) Are released by the rupture of the corona radiata at the time of ovulation Give rise to oogonia Measure about 2 mm in diameter Page | Undergo routine further development after liberation. 188

The ova are developed from the primitive germ cells, which are embedded in the substance of the ovaries. Each primitive germ cell gives rise, by repeated divisions, to a number of smaller cells called oogonia, from which the ova or primary oocytes are developed.

Human ova are extremely small, measuring about 0.2 mm in diameter and are enclosed within the egg follicles of the ovaries. By the enlargement and subsequent rupture of a follicle at the surface of the ovary, an ovum is liberated and conveyed by the uterine tube to the cavity of the uterus.

Unless it is fertilised it undergoes no further development and is discharged from the uterus, but if fertilisation take place it is retained within the uterus and develops into a new being.

157. A 55-year-old female presents with severe burns following a house fire. During clinical assessment you observe that her back, right arm and perineum have been affected, and you calculate the total body surface area (TBSA) burned to optimise management. What is the TBSA in this case? Single best answer - select one answer only. 18%« YOUR ANSWER 19% 27% 28%« CORRECT ANSWER 36%. Wallace‟s „rule of nines‟ states that TBSA can be divided into 9% for the head, 9% for each arm, 18% for each leg and 36% for the trunk (18% for the posterior and anterior trunk respectively), and 1% for the perineum. This is important as appropriate fluid management is based on the Parkland formula (Amount of fluid in first 24 hours in mls = 4 x body mass in kg x TBSA, with 50% given in the first 8 hours).

A more accurate estimation of TBSA burned can be obtrained using Lund and Browder charts which are available in both adult and paediatric versions. The paediatric charts allow adjustment for the age of the child.

158. A 60-year-old woman who had undergone a successful left renal transplant one week earlier, presents with fever, oliguria and rising serum creatinine. Which is the best management option in such a scenario? It is a picture of acute rejection characterized by preformed antibodies to donor HLA antigen and should be treated with intravenous Hydrocortisone« YOUR ANSWER Tacromlimus should be the first-line treatment ------Dr Mohammed Shamsul Islam Khan, Medical Officer, Clinical Neuro-Surgery, National Institute of Neuro-Sciences and Hospital Sher-E-Bangla Nagar, Dhaka-1207, Bangladesh. Mobile: +880 1713 455 662, E-mail: [email protected]

MyPasTest: MRCS A Online - Jan Exam 2015 03. Anatomy; Aabdomen, Pelvis & Perineum (290Qs) ------The woman has chronic graft rejection Treatment is with Prednisolone Urinary obstruction, infection, reduced blood supply and drug toxicity should be considered before treating for acute graft rejection« CORRECT ANSWER. Page | Graft rejection following kidney transplantation can be hyperacute, early and late acute, and chronic. Hyperacute rejection is irreversible and due to preformed antibodies to HLA class I 189 antigen. It occurs within a few minutes of revascularisation. Risk is increased from previous blood transfusion, pregnancy and previous failed transplant. Preoperative ABO compatibility is necessary.

Early acute graft rejection is likely to occur 1 week to 10 days post renal transplantation. It is the result of a T-helper cell mediated response or vascular response mediated by antibody. Recipient T cells react to antigen present on donor vascular endothelium and dendritic cells with the release of Il-2.

The vascular response is noted by IgG and IgM antibodies bound to donor endothelium. Acute late rejection is seen in patients immunosuppressed by agents such as Prednisolone or Azathioprine. It occurs 11 days to 6 weeks post transplantation and is characterised by florid antibody and complement mediated vascular damage.

159. A 67-year-old female presents with rectal bleeding, tenesmus and weight loss. She is diagnosed with high rectal adenocarcinoma and proceeds to surgery. The proximal rectum has primary lymphatic drainage to which nodes? Deep inguinal lymph nodes« YOUR ANSWER External iliac lymph nodes Inferior mesenteric lymph nodes« CORRECT ANSWER Superficial inguinal lymph nodes Superior mesenteric lymph nodes.

The rectum has no true mesentery, but is enveloped by the visceral pelvic mesentery, referred to as the mesorectum. This tissue contains the pararectal lymph nodes which are the primary draining lymph nodes of the rectum. Lymphatic vessels from the superior half of the rectum drain to para-rectal nodes and from there to inferior mesenteric and lumbar nodes. In contrast, lymphatic vessels from the inferior half of the rectum travel with the middle rectal vessels to the internal iliac nodes.

160. A 38-year-old man is involved in a high speed RTA where he sustains trauma to his right hemi pelvis. He is left with long term sensory deficit affecting the majority of his right leg. An MRI confirms the clinical findings that damage has been sustained to the sacral plexus. With regards to the sacral plexus, which one of the following is true? Single best answer - select one answer only. Emerges through the anterior sacral foramina« YOUR ANSWER (Correct) Is joined by the lumbosacral trunk posterior to piriformis

------Dr Mohammed Shamsul Islam Khan, Medical Officer, Clinical Neuro-Surgery, National Institute of Neuro-Sciences and Hospital Sher-E-Bangla Nagar, Dhaka-1207, Bangladesh. Mobile: +880 1713 455 662, E-mail: [email protected]

MyPasTest: MRCS A Online - Jan Exam 2015 03. Anatomy; Aabdomen, Pelvis & Perineum (290Qs) ------It is formed from the anterior division of the 1st sacral nerve and posterior divisions of the 2nd and 3rd sacral nerves. Lateral cutaneous nerve of the thigh is a branch The inferior gluteal nerve passes through the greater sciatic foramen above piriformis. Page | The sacral plexus is formed by the lumbosacral trunk and the anterior division of the 1st sacral nerve plus portions of the anterior divisions of the 2nd and 3rd sacral nerves. 190

The sacral nerves are joined by the lumbosacral trunk anterior to piriformis. The superior gluteal nerve passes through the greater sciatic foramen above piriformis. The inferior gluteal nerve, posterior cutaneous nerve of the thigh and pudendal nerves pass through the greater sciatic foramen below piriformis.

The lateral cutaneous nerve of the thigh is a branch of the lumbar plexus arising from the dorsal branches of the second and third lumbar nerves.

161. Whilst performing a cholecystectomy your consultant points out Calot‟s triangle and asks about its boundaries. Which of the following is the correct description? Single best answer - select ONE true answer only. It lies between the right and left hepatic arteries and inferior border of the liver« YOUR ANSWER It lies between the left hepatic duct, left hepatic artery and inferior border of the liver It lies between the common hepatic duct, common hepatic artery and inferior border of the liver It lies between the left hepatic artery, right hepatic artery and inferior border of the liver It lies between the cystic duct, common hepatic duct and inferior border of the liver« CORRECT ANSWER.

Calot‟s triangle lies between the inferior border of the liver, cystic duct and common hepatic duct and contains the cystic artery usually. 162. A 23-year-old male presents with a painful, tender swelling inferior and lateral to the pubic tubercle. What is the most likely diagnosis? Single best answer - select one answer only. Amyand’s hernia« YOUR ANSWER Femoral hernia« CORRECT ANSWER Inguinal hernia Obturator hernia Spigelian hernia.

Although they are more common in females, males can also develop femoral hernias. The key to diagnosis is the relation of the neck of the hernia to the pubic tubercle: in femoral hernias this lies inferior and lateral to the tubercle, whereas in inguinal hernias the neck is superior and medial to the tubercle.

------Dr Mohammed Shamsul Islam Khan, Medical Officer, Clinical Neuro-Surgery, National Institute of Neuro-Sciences and Hospital Sher-E-Bangla Nagar, Dhaka-1207, Bangladesh. Mobile: +880 1713 455 662, E-mail: [email protected]

MyPasTest: MRCS A Online - Jan Exam 2015 03. Anatomy; Aabdomen, Pelvis & Perineum (290Qs) ------Amyand‟s hernia is an inguinal hernia involving an inflamed appendix. An obturator hernia is more common in females, and can be considered if the pain increases with hip extension, medial rotation and abduction. This hernia is very rare.

A Spigelian hernia, another uncommon hernia, occurs through a defect in the synonymous fascia, at the lateral edge of the rectus abdominus muscle. Page |

163. You are performing an appendicectomy and find a Meckel‟s diverticulum. Your consultant 191 asks you what this is, and what causes it. How do you respond? Select one answer only. It is a false diverticulum and a remnant of the vitello-intestinal duct« YOUR ANSWER It is a false diverticulum and a urachal remnant It is a true diverticulum and remnant of the allantois It is a true diverticulum and a remnant of the vitello-intestinal duct« CORRECT ANSWER It is a true diverticulum and a urachal remnant.

It is a true diverticulum, (all layers involved), on the antimesenteric border of the ileum, which is a remnant of the vitello-intestinal duct and occurs in 2% of the population. It may contain heterotropic gastric (producing dark red rectal bleeding almost like melaena) and pancreatic mucosa. It should be removed if it contains abnormal mucosa, is long, has a narrow mouth or if the patient is less than 40.

164. A 68-year-old female undergoes superficial femoral artery angioplasty after developing critical lower limb ischaemia. Two days later she complains of pain and you observe a pulsatile swelling in the left groin. What is the most likely diagnosis? Single best answer - select one answer only. Abscess« YOUR ANSWER Aneurysm Arteriovenous fistula Dissection Pseudo-aneurysm« CORRECT ANSWER.

Pseudo-aneurysms are a common complication of arterial puncture during interventional procedures. They are defined as a haematoma developing outside an arterial wall secondary to that wall‟s puncture. Clinically they are pulsatile and painful, and can rupture, leading to significant haemorrhage. Dissections and fistulae can also complicate arterial puncture.

165. A 23-year-old aid worker, recently returned from Uganda, presents with a two week history of diarrhoea, severe abdominal pain and fever. He is noted to have generalised peritonitis and is taken to theatre for an emergency laparotomy. During the procedure he is noted to have several intestinal perforations and undergoes bowel resection and peritoneal lavage. The subsequent pathology report confirms the presence of Salmonella typhi within Peyer‟s patches. Where is the most likely site of his perforation? Duodenum« YOUR ANSWER ------Dr Mohammed Shamsul Islam Khan, Medical Officer, Clinical Neuro-Surgery, National Institute of Neuro-Sciences and Hospital Sher-E-Bangla Nagar, Dhaka-1207, Bangladesh. Mobile: +880 1713 455 662, E-mail: [email protected]

MyPasTest: MRCS A Online - Jan Exam 2015 03. Anatomy; Aabdomen, Pelvis & Perineum (290Qs) ------Gastric body Ileum« CORRECT ANSWER Jejunum Sigmoid colon. Page | This patient has typhoid fever, caused by Salmonella typhi. The bacteria accumulates within Peyer‟s patches and causes mucosal uleration. Left untreated, these ulcers can perforate and 192 cause secondary peritonitis. The ileum has the highest concentration of Peyer‟s patches within the gastrointestinal tract.

166. A 32-year-old male presents with unilateral testicular swelling. He undergoes clinical examination and investigation and is diagnosed with a testicular seminoma. To which group of lymph nodes is his tumour most likely to metastasize? Single best answer - select one answer only. Axillary« YOUR ANSWER Deep inguinal Para-aortic« CORRECT ANSWER Superficial inguinal Supraclavicular.

In males in their fourth decade, seminomas are the commonest tumour type, in comparison to third-decade males where teratomas predominate. All testicular tumours metastasize to the para-aortic nodes, reflecting the abdominal embryological development of the testes.

167. A 45-year-old lady presents to casualty with right sided abdominal pain and vomiting. A presumed diagnosis of appendicitis was made and she was prepared for a diagnostic laparoscopy which revealed an inflamed Meckel‟s diverticulum. Which of the following most correctly describes a Meckel‟s diverticulum? Single best answer - select one answer only. May contain heterotropic gastric mucosa« YOUR ANSWER (Correct) Classically presents in middle age with rectal bleeding Is equally distributed between males and females May be a false diverticulum When present within a hernia is termed a Richter’s hernia.

Meckel‟s diverticulum is an example of a true diverticulum. It many contain heterotropic gastric mucosa that can ulcerate and bleed. Intestinal obstruction can occur due to congenital bands between the apex of the diverticulum and the umbilicus.

This can lead ultimately to occurrence of volvulus and/or internal herniation. When present within a hernia it is termed a Littre‟s hernia (rare). Occurs twice as frequently in men. The majority are asymptomatic but if symptoms do occur they are usually before 2 years of age with rectal bleeding and intussusception.

------Dr Mohammed Shamsul Islam Khan, Medical Officer, Clinical Neuro-Surgery, National Institute of Neuro-Sciences and Hospital Sher-E-Bangla Nagar, Dhaka-1207, Bangladesh. Mobile: +880 1713 455 662, E-mail: [email protected]

MyPasTest: MRCS A Online - Jan Exam 2015 03. Anatomy; Aabdomen, Pelvis & Perineum (290Qs) ------168. A 72-year-old female is admitted for an elective right hemicolectomy, following a diagnosis of colonic adenocarcinoma. During the procedure, you are advised to avoid injury to a vessel arising from the abdominal aorta at the level of the second lumbar vertebra, and passing anterior to the inferior vena cava. Which vessel is at risk? Common iliac artery« YOUR ANSWER Inferior mesenteric artery Page | Middle suprarenal artery 193 Right ovarian artery« CORRECT ANSWER Right renal artery.

The gonadal vessels originate from the abdominal aorta at the level of the L2 vertbrae and descend into the pelvis anterior to the inferior vena cava (IVC). The right renal artery travels posterior to the IVC. The gonadal vessels are usually encountered during the dissection stage of a right hemicolectomy.

169. A 58-year-old female presents with persistent upper abdominal pain and low-grade fever, but no jaundice. On examination there is an ill-defined fullness in the epigastrium. She is currently awaiting laparoscopic cholecystectomy following an episode of acute pancreatitis two months earlier. What is the most appropriate investigation? Single best answer - select one answer only. Computerised tomography« YOUR ANSWER (Correct) Magnetic resonance cholangiopancreatography Serum alpha fetoprotein Upper GI endoscopy Ultrasound scan.

Pancreatic pseudocysts can develop following attacks of acute pancreatitis. They take 3–6 weeks to mature and usually present with low-grade fever, leucocytosis, chronic abdominal pain and/or persistent rise in serum amylase.

The investigation of choice is CT abdomen, which enables the size and location of the collection to be determined. They are treated initially with percutaneous (with imaging guidance), internal or external drainage, but if recurrent may require formal surgical drainage via cystogastrostomy or cystenterostomy.

170. A 37-year-old female who is post-partum, presents with right upper quadrant pain, nausea and vomiting, hepatosplenomegaly and ascites. What is the most likely cause for this? Select one answer only. Alcoholic cirrhosis« YOUR ANSWER Budd–Chiari syndrome« CORRECT ANSWER Pylephlebitis after acute appendicitis Splenectomy ------Dr Mohammed Shamsul Islam Khan, Medical Officer, Clinical Neuro-Surgery, National Institute of Neuro-Sciences and Hospital Sher-E-Bangla Nagar, Dhaka-1207, Bangladesh. Mobile: +880 1713 455 662, E-mail: [email protected]

MyPasTest: MRCS A Online - Jan Exam 2015 03. Anatomy; Aabdomen, Pelvis & Perineum (290Qs) ------Tricuspid valve incompetence.

Portal hypertension with a pressure of over 20 mmHg is commonly caused by prehepatic problems such as portal vein thrombosis, hepatic disease such as cirrhosis and post hepatic problems such as tricuspid valve incompetence and Budd–Chiari syndrome of hepatic vein thrombosis. Budd chiari syndrome commonly results in hepatosplenomegaly and ascities. It is Page | associated with pregnancy and being post partum. 194

171. An elderly lady presents to the Emergency Department with severe dysphagia. Her symptoms have been progressing and there is a vague suggestion of a gastric bubble above the diaphragm on the chest radiograph, raising the possibility of a hiatus hernia. Which of the following is correct regarding hiatus herniae? Single best answer - select one answer only. The majority of patients with hiatus herniae are symptomatic, and usually experience acid reflux« YOUR ANSWER As with gastro-oesophageal reflux disease, the most appropriate initial diagnostic modality is ambulatory pH monitoring When GORD is associated with a proven sliding hiatus hernia, surgical management in the form of a Nissen fundoplication is the treatment of choice Despite the high fibre diet, paradoxically hiatus herniae are found more commonly in rural African communities Sliding herniae are primarily associated with symptoms of GORD, whilst rolling herniae can cause gastric strangulation and necrosis and gangrene« CORRECT ANSWER.

The majority of hiatus herniae are sliding or axial in nature, are usually asymptomatic but can be associated with oesophagitis, stricture formation, dysphagia, chronic anaemia and inhalational pneumonitis. Rolling herniae or para-oesophageal hiatal herniae usually affect elderly patients who present with intermittent dysphagia, pain after eating due to distension of the intrathoracic part of the stomach, cardiac symptoms due to pressure effects on the heart, and hiccough due to Phrenic nerve irritation.

Complications include incarceration, gangrene and gastric volvulus. Oesophagitis is more commonly associated with sliding hiatus hernias. The most appropriate investigation is upper GI endoscopy, upper GI contrast series or upper GI manometry.

In severe cases of Hiatus Herniae, surgery in the form of Nissen fundoplication may be necessary but in the vast majority symptoms can be managed by lifestyle modification and medications such as proton pump inhibitors and H2 blockers. The low fibre diet of Westernised society, and also the prolonged sitting position for defecation, results in a higher incidence of hiatus herniae.

172. A 35-year-old gentleman with familial adenomatous polyposis (FAP) undergoes a Whipples procedure for a malignant duodenal polyp. During the dissection, the Superior mesenteric artery is identified. What is the closest anatomical relation to its origin? Single best answer - select one answer only. Neck of the pancreas« YOUR ANSWER (Correct)

------Dr Mohammed Shamsul Islam Khan, Medical Officer, Clinical Neuro-Surgery, National Institute of Neuro-Sciences and Hospital Sher-E-Bangla Nagar, Dhaka-1207, Bangladesh. Mobile: +880 1713 455 662, E-mail: [email protected]

MyPasTest: MRCS A Online - Jan Exam 2015 03. Anatomy; Aabdomen, Pelvis & Perineum (290Qs) ------The hepatic flexure of the colon The splenic vein as it passes below Tail of the pancreas The third part of the duodenum. Page | The superior mesenteric artery (SMA) supplies the entire small intestine except the proximal duodenum. The SMA also supplies the caecum, ascending colon and half the transverse colon. 195 The origin of the SMA lies behind the neck of the pancreas (at L1 level). The SMA passes forwards and downwards in front of the uncinate process and the third part of the duodenum. Throughout, the superior mesenteric vein lies to its right. The splenic vein grooves the posterosuperior aspect of the pancreas and passes above the SMA.

173. You are reviewing a one-day-old with abdominal distension and bile stained vomiting. He has not yet passed meconium. You suspect Hirschsprung‟s disease. Which of the following is true? Select one answer only. There is a congenital ganglion absence in the rectum« YOUR ANSWER (Correct) There will be dilation of the affected segment This affects both sexes equally This is diagnosed by cold-cup rectal biopsy This usually presents in patients greater than 10-years of age.

Hirschsprung‟s disease occurs in 1 in 5000 births and has an M : F ratio of 5 : 1. Dilation occurs proximal to the affected segment. Full-thickness rectal biopsy is required. It presents in the neonate with abdominal distension and bile-stained vomiting. 174. A 76-year-old lady presents to the A & E department with profuse fresh rectal bleeding with clots. Currently she has a pulse rate of 90 bpm and a BP of 140/86. She had a similar episode 2 years ago, has not lost any weight and has normal bowel habits. She is also on omeprazole for long standing gastro-oesophageal reflux disease. Which of the following statements best describes this clinical scenario? Single best answer - select one answer only. The commonest cause of massive rectal bleeding is diverticular disease and is usually associated with lower abdominal pain« YOUR ANSWER The passage of bright red blood is never due to duodenal ulcer It is usually controlled by catheter-directed infusion of vasopressin Following resuscitation an urgent OGD is an diagnostic option« CORRECT ANSWER It may be localised by angiography if <1 ml/min.

Diverticular disease with erosion of small vessels in the bowel wall is the most common form of significant lower gastrointestinal haemorrhage and is usually painless. Diverticulitis however often presents with LIF pain and a temperature but is rarely associated with bleeding. However, if bleeding is brisk, it may come from as high up as a peptic or duodenal ulcer, with the presence of blood promoting a fast transit time.

------Dr Mohammed Shamsul Islam Khan, Medical Officer, Clinical Neuro-Surgery, National Institute of Neuro-Sciences and Hospital Sher-E-Bangla Nagar, Dhaka-1207, Bangladesh. Mobile: +880 1713 455 662, E-mail: [email protected]

MyPasTest: MRCS A Online - Jan Exam 2015 03. Anatomy; Aabdomen, Pelvis & Perineum (290Qs) ------In a stable patient an urgent OGD is a relatively simple diagnostic test which can also be therapeutic. Vasopressin has little role in the management of such patients – if they do not settle conservatively, then patients will need surgery. Angiographic techniques can pick up bleeding at the rate of 1 ml/min in experienced hands.

175. A 46-year-old female presents with massive haematemesis and upper abdominal pain. She is resuscitated and proceeds to emergency gastroscopy, which reveals a deep ulcer in the Page | posterior wall of the gastric body. The patient eventually requires an emergency laparotomy to achieve haemostasis. Which vessel is likely to be the source of the haemorrhage? Single best 196 answer - select one answer only. Common hepatic artery« YOUR ANSWER Gastroduodenal artery Splenic artery« CORRECT ANSWER Superficial oesophageal veins Superior mesenteric artery.

Peptic ulcers of the posterior gastric body can erode through the splenic artery, causing massive haemorrhage. A similar complication can occur in duodenal peptic ulcers when the gastroduodenal artery is eroded. The superficial oesophageal veins become oesophageal varices in cases of portal hypertension, occasionally causing massive upper gastrointestinal haemorrhage themselves.

176. A 37-year-old male presents with colicky right loin pain and is noted to have microscopic haematuria. A plain kidney-ureter-bladder radiograph reveals a dense opacity at the level of the pelvic brim. Which structure lies posterior to the ureter at this point? Single best answer - select one answer only. Appendix« YOUR ANSWER Bifurcation of the common iliac artery« CORRECT ANSWER Testicular artery Vas deferens Vesicoureteric junction.

The ureter descends towards the pelvis anterior to the psoas major muscle but posterior to the gonadal artery. It then crosses anterior to the bifurcation of the common iliac artery at the level of the pelvic brim before descending posteroinferiorly along the lateral pelvic side wall. It courses anteromedially towards the bladder, passing beneath the vas deferens or the uterine vessels, and enters the urinary bladder at the vesicoureteric junction.

177. A junior doctor is asked to insert a urethral catheter in a 57-year-old gentleman who has developed acute urinary retention following an elective inguinal hernia repair. He uses a 14 French Foley catheter and an aseptic technique. It is difficult but he manages. A few hours later, you are called as the gentleman‟s scrotum and perineum begins to become swollen. Blood is seen at the meatus and in the catheter bag. What will be the most appropriate management step? Single best answer - select one answer only.

------Dr Mohammed Shamsul Islam Khan, Medical Officer, Clinical Neuro-Surgery, National Institute of Neuro-Sciences and Hospital Sher-E-Bangla Nagar, Dhaka-1207, Bangladesh. Mobile: +880 1713 455 662, E-mail: [email protected]

MyPasTest: MRCS A Online - Jan Exam 2015 03. Anatomy; Aabdomen, Pelvis & Perineum (290Qs) ------Cystoscopy« YOUR ANSWER Cystourethrogram Immediate repair Repair within 7 days Page | Suprapubic catheter« CORRECT ANSWER. 197 Traumatic catheter insertion may rupture the posterior and anterior urethra. Rupture of the spongy (penile) part of the urethra leaks urine into the superficial perineal fascia (of Colles), which is continuous with the membranous fascia (of Scarpa) in the anterior abdominal wall – such urine leakage may result in swelling of the perineum and scrotum.

Blood may also be seen at the urethral meatus. A suprapubic cathether may be used to temporarily divert urine out of the bladder and away from the injured urethra. Investigation with a retrograde urethrogram and repair should be considered 6-12 weeks post injury.

It is important to note that the penile urethra takes a 90º angle through the bulbar part. The membranous urethra is the narrowest and least dilatable of all the parts of the male urethra. The corpus spongiosum invests the penile urethra. The prostatic part is bounded by both the internal and external urethral sphincters.

178. A 10-month-old boy presents with colicky abdominal pain, vomiting and redcurrant stools. On examination there is a sausage-shaped mass palpable within an otherwise soft abdomen. What is the most appropriate initial management? Emergency laparotomy « YOUR ANSWER Hydrostatic gastrografin enema« CORRECT ANSWER Lower GI endoscopy Rigid sigmoidoscopy Stool softeners.

Intussusception occurs in 0.4% of children. It is more common in boys, usually younger than 1 year. Most frequently the intussusception is ileocolic. Blood and mucus or „redcurrant jelly‟ stool may be passed after the first 24 hours as a late manifestation. Sausage-shaped mass may be felt in upper abdomen. Rectal examination is very important as occasionally the apex of intussusception is palpable. The majority of cases are successfully treated conservatively with hydrostatic Gastrografin enema. Operative management is indicated in established peritonitis, perforation or failure of hydrostatic reduction.

179. The narrowest point of the urethra is? Single best answer – choose ONE true option only. The navicular fossa« YOUR ANSWER At the site of seminal colliculus At the membranous urethra The external urethral orifice« CORRECT ANSWER The bulbous urethra. ------Dr Mohammed Shamsul Islam Khan, Medical Officer, Clinical Neuro-Surgery, National Institute of Neuro-Sciences and Hospital Sher-E-Bangla Nagar, Dhaka-1207, Bangladesh. Mobile: +880 1713 455 662, E-mail: [email protected]

MyPasTest: MRCS A Online - Jan Exam 2015 03. Anatomy; Aabdomen, Pelvis & Perineum (290Qs) ------

The male urethra is approximately 20 cm long and the narrowest points are the external urethral orifice, bladder neck & just proximal to navicular fossa. The seminal colliculus are present in the prostatic urethra at the verumontanum. The prostatic urethra is the widest and most dilatable part of the entire male urethra. Page | 180. You are managing the rupture of the urinary bladder. How would you go about this? Single best answer - select one answer only. 198 Bladder lacerations are repaired en masse with absorbable sutures« YOUR ANSWER Bladder lacerations are repaired en masse with non-absorbable sutures Bladder lacerations are repaired in layers with absorbable sutures« CORRECT ANSWER Bladder lacerations are repaired in layers with non-absorbable sutures Intraperitoneal lacerations can be treated by catheter drainage.

Bladder injuries are commonly associated with pelvic fractures. Intraperitoneal rupture is repaired via a transperitoneal approach in layers with absorbable sutures. Extraperitoneal ruptures can be treated with catheterisation.

181. A 45-year-old female presents with colicky right upper quadrant pain which is exacerbated by eating. She is diagnosed with gallstones and undergoes laparoscopic cholecytstectomy. From which branch of the abdominal aorta does the gall bladder‟s blood supply arise? Coeliac artery« YOUR ANSWER (Correct) Hepatic artery proper Inferior mesenteric artery Right gastric artery Superior mesenteric artery.

Branches of the coeliac artery supply the gastrointestinal (GI) structures in the foregut, including the gall bladder. The inferior pancreaticoduodenal artery, a branch of the superior mesenteric artery supplies both the duodenum and the head of the pancreas, which are both foregut structures. Branches of the superior mesenteric artery supply the GI structures in the midgut. Branches of the inferior mesenteric artery supply the GI structures in the hindgut. The hepatic artery proper and the right gastric artery are not branches of the abdominal aorta.

182. Which of the following is correct regarding the urinary bladder? Has a peritoneal covering on all surfaces« YOUR ANSWER Has smooth muscle fibres at the bladder neck to maintain urinary continence Has a sympathetic nerve supply to the detrusor muscle Is attached to the anterior abdominal wall by the median umbilical ligament« CORRECT ANSWER Receives its main arterial supply from the obturator and inferior gluteal arteries.

------Dr Mohammed Shamsul Islam Khan, Medical Officer, Clinical Neuro-Surgery, National Institute of Neuro-Sciences and Hospital Sher-E-Bangla Nagar, Dhaka-1207, Bangladesh. Mobile: +880 1713 455 662, E-mail: [email protected]

MyPasTest: MRCS A Online - Jan Exam 2015 03. Anatomy; Aabdomen, Pelvis & Perineum (290Qs) ------Only the superior surface and some of the base of the urinary bladder are covered by peritoneum. The ureters pierce that bladder wall at the superolateral angles of the trigone. The median umbilical ligament – the remains of the urachus – extends from the apex to the anterior abdominal wall. The detrusor muscle has parasympathetic innervation. The circular smooth muscle fibres at the bladder neck do not maintain urinary continence. The main arterial supply to the bladder is from the superior and inferior vesical arteries, with smaller branches from the obturator and inferior gluteal arteries. Page |

183. A 56-year-old woman presents to casualty with severe upper abdominal pain. On 199 questioning she admits to several similar episodes of upper abdominal pain radiating to her back over the past 4 years which have all required admission. She has previously lived in Africa as a child but currently lives in the UK. On examination she is in in severe pain and looks thin and unkempt. A diagnosis of chronic pancreatitis is made. Which of the following is the most likely aetiology in this lady? Single best answer - select one answer only. Alcohol« YOUR ANSWER (Correct) Cholelithiasis Hypocalcaemia Hypothermia Mumps.

Cholelithiasis (gallstones) and choledocholithiasis (gallstones in the common bile duct) are common causes of acute pancreatitis which with alcohol accounts for approximately 80% of cases. Hypothermia and viral infections such as mumps are also well-recognised causes of this condition. Hypocalcaemia is a complication of acute pancreatitis, as opposed to hypercalcaemia which is a cause. Scorpion bites are also a cause in those with appropriate exposure. The most common cause of chronic pancreatitis however is chronic excess alcohol as is the case in this thin unkempt patient.

184. A 45-year-old man presents with severe upper abdominal pain following a prolonged course of non-steroidal anti-inflammatory drugs for chronic back pain. He is peritonitic and an erect chest radiograph reveals free intra-abdominal air. He consents for an upper midline laparotomy to deal with a presumed perforated gastroduodenal ulcer. Which of the following statements is true regarding this condition? Single best answer - select one answer only. A pre-operative OGD is beneficial in locating the site of pathology« YOUR ANSWER An effective repair for small perforations involves primary repair and a free omental patch The surgical approach involves incising rectus abdominis The transversalis fascia is incised in this surgical approach« CORRECT ANSWER The visceral peritoneum is broached during this approach.

A midline laparotomy is not a muscle cutting incision. The incision passes through the skin, subcutaneous flat, the linea alba, extraperitoneal fat, transversalis fascia and parietal peritoneum. Once the diagnosis of a gastric perforation is made then laparotomy is the next step. An OGD adds little to the diagnosis and may make matters worse by increasing the perforation. At operation the defect is closed over an omental patch which is attached to its blood supply rather than being a free flap.

------Dr Mohammed Shamsul Islam Khan, Medical Officer, Clinical Neuro-Surgery, National Institute of Neuro-Sciences and Hospital Sher-E-Bangla Nagar, Dhaka-1207, Bangladesh. Mobile: +880 1713 455 662, E-mail: [email protected]

MyPasTest: MRCS A Online - Jan Exam 2015 03. Anatomy; Aabdomen, Pelvis & Perineum (290Qs) ------

185. Which of the following is the most common pathological lead point in intussusception? Single best answer - select one answer only. Dance’s sign« YOUR ANSWER Intraluminal haematoma Page | Meckel’s diverticulum Peyer’s patch« CORRECT ANSWER 200 Polyps.

Dances sign, (a feeling of emptiness on palpation), with viscera absent in the right lower quadrant on abdominal X-ray is seen in 15% of cases. An intraluminal haematoma in Henoch– Schönlein purpura may trigger intussusception. Meckel‟s diverticulum and polyps are identified as a pathological lead point in 10% of cases. Peyer‟s patches in the ileum are the pathological lead patch in up to 90% of cases.

186. You are treating a patient with a rectal carcinoma and you consultant asks you about the about the blood supply and lymphatic drainage of the rectum. Which one of the following is true? Single best answer - Choose ONE correct answer. Lymph from the proximal rectum drains into the inferior mesenteric nodes « YOUR ANSWER (Correct) It has lymphatic drainage to the superficial inguinal nodes It has venous drainage to the portal system alone It has a venous drainage into the superior mesenteric vein It receives its main arterial blood supply from the middle rectal artery.

Veins from the rectum anastomose freely forming an internal and an external rectal plexus. The superior and inferior rectal veins closely follow their arteries so their drainage is to both the portal and systemic systems. Lymph from the superior half of the rectum drains to para-rectal nodes and from there to inferior mesenteric and lumbar nodes. In contrast, lymphatic vessels from the inferior half of the rectum travel with the middle rectal vessels to the internal iliac nodes.

The rectum has no true mesentery; the visceral pelvic fascia around the rectum is referred to by surgeons as the mesorectum. The main blood supply to the rectum is from the superior rectal artery with some contribution from the middle and inferior rectal and median sacral vessels. The pararectal lymph nodes are found in the mesorectum, which is removed together with the rectum during rectal excision for carcinoma.

187. A 70-year-old gentleman undergoes a difficult laparoscopic cholecystectomy where the cystic duct is found to be wide. Prior to this procedure he had undergone an ERCP and sphincterotomy for common bile duct stones. In the postoperative period he develops a subdiaphragmatic collection which is drained. The fluid is bile and so a stent is placed within the common bile duct by ERCP. Within 24 hours he becomes tachycardic, hypotensive, passes

------Dr Mohammed Shamsul Islam Khan, Medical Officer, Clinical Neuro-Surgery, National Institute of Neuro-Sciences and Hospital Sher-E-Bangla Nagar, Dhaka-1207, Bangladesh. Mobile: +880 1713 455 662, E-mail: [email protected]

MyPasTest: MRCS A Online - Jan Exam 2015 03. Anatomy; Aabdomen, Pelvis & Perineum (290Qs) ------black stools and develops severe abdominal pain with guarding. What is the most likely diagnosis? Single best answer - select one answer only. Acute pancreatitis« YOUR ANSWER Enterocutaneous fistula Gastrointestinal haemorrhage « CORRECT ANSWER Page | Small bowel obstruction 201 Small bowel perforation.

Division of the sphincter of Oddi with the sphincterotome may cause pancreatitis, duodenal perforation or bleeding. Many patients have a transiently increased serum amylase but a few develop true acute pancreatitis with pain and ultrasound evidence of pancreatitis. The black stools suggest the passage of melaena and therefore upper GI bleeding.

188. A two-day-old child is diagnosed with cryptorchidism. Which embryological structure is associated with this diagnosis? Allantois« YOUR ANSWER Genital tubercle Gubernaculum« CORRECT ANSWER Mullerian duct Wolffian duct. The gubernaculum (or caudal genital ligament) is a ligament that connects the scrotal base with the foetal testis. Failure of the gubernaculums to shorten sufficiently leads to cryptorchidism. The allantois forms the umbilical vasculature, the genital tubercle is the rudimentary penis, the Mullerian duct forms the vagina, cervix, uterus and oviducts (and is inhibited in males) and the Wolffian (or mesonephric duct) duct forms the epididymis, seminal vesicles and vas deferens (and is inhibited in females).

189. Whilst performing an exploratory laparotomy for small bowel obstruction you identify a polypoid mass within the small bowel. You resect the mass and having examined the small bowel around the mass from within and without the lumen, you perform a hand sown end to end anastomosis. Which of the following anatomical differences would enable you best to operatively distinguish the jejunum from the ileum? Single best answer - select one answer only. Mesenteric vessels form multiple jejunal arcades with short branches to the intestine« YOUR ANSWER The jejunum has a narrower lumen The jejunum has a thicker wall« CORRECT ANSWER There is an absence of Peyer’s patches in the jejunal wall There is an absence of plicaecirculares in the jejunum.

The jejunum makes up the proximal two-fifths of the small intestine and the ileum the distal three-fifths. There is no macroscopic demarcation between the two. The jejunum has a wider lumen, thicker wall and plicaecirculares. Peyer‟s patches are present in the mucous membrane ------Dr Mohammed Shamsul Islam Khan, Medical Officer, Clinical Neuro-Surgery, National Institute of Neuro-Sciences and Hospital Sher-E-Bangla Nagar, Dhaka-1207, Bangladesh. Mobile: +880 1713 455 662, E-mail: [email protected]

MyPasTest: MRCS A Online - Jan Exam 2015 03. Anatomy; Aabdomen, Pelvis & Perineum (290Qs) ------of the distal ileum but would not aid you to differentiate between the two with the naked eye. The jejunal mesenteric vessels form only one or two arcades with long and infrequent branches stretching to the intestinal wall.

190. You are assisting in an open prostatectomy. Which of the following is true regarding the prostate gland? Single best answer - select ONE correct answer. Page | Lymph drainage is to the deep inguinal lymph nodes« YOUR ANSWER It develops carcinoma most commonly in its central zone 202 It provides about 60% of the volume of seminal fluid It is supplied by a branch of the external iliac artery It is surrounded by a venous plexus« CORRECT ANSWER.

Each ejaculatory duct passes obliquely through the prostate gland to open on the veromontanum. The prostate is mainly supplied from the prostatic branch of the inferior vesical artery (a branch of the internal iliac), with small branches from the middle rectal and internal pudendal vessels. It drains to the lateral pelvic lymph nodes. Most carcinomas develop in the peripheral zone. The prostate provides about 30% of the volume of seminal fluid, most of which comes from the seminal vesicles.

191. A 67-year-old lady presenting with symptoms of severe gastro-oesophageal reflux disease has recently had an upper GI endoscopy. She has now been given a diagnosis of Barrett‟s oesophagus. Which of the following statements most accurately describes this condition? Single best answer - select one answer only. Occurs in approximately 40% of patients with gastro-oesophageal reflux disease (GORD) in the long term« YOUR ANSWER Is caused by chronic reflux of gastric acid into the lower oesophagus causing metaplasia of the normal squamous epithelium of the lower oesophagus to columnar and goblet cell epithelium.« CORRECT ANSWER Presents with malignancy in one-third of cases Is diagnosed by the position of the z line which rises from its normal position Histamine H2 blockers are the treatment of choice.

5-15% of patients with gastro-oesophageal reflux disease (GORD) will develop Barrett‟s oesophagus which is associated with an increased risk of oesophageal malignancy. The chronic reflux of gastric acid into the lower oesophagus causes metaplasia from squamous epithelium to columnar epithelium with goblet cells and together with elevation of the Z line on OGD is diagnostic.

The Z line usually lies 3cm from the lower oesophageal sphincter and is identified by a change in colour from salmon pink to a deeper red. The treatment of choice is with proton pump inhibitors and close follow up. The risk of esophageal cancer in people with Barrett's was 0.13 percent.About 1 in 860 patients with Barrett's go on to develop cancer (Cancer research UK).

------Dr Mohammed Shamsul Islam Khan, Medical Officer, Clinical Neuro-Surgery, National Institute of Neuro-Sciences and Hospital Sher-E-Bangla Nagar, Dhaka-1207, Bangladesh. Mobile: +880 1713 455 662, E-mail: [email protected]

MyPasTest: MRCS A Online - Jan Exam 2015 03. Anatomy; Aabdomen, Pelvis & Perineum (290Qs) ------192. A 47-year-old man undergoes an open left inguinal hernia repair. During the operation, the surgeon transects the ilioinguinal nerve. What is the most likely immediate consequence for the patient? Single best answer - select one answer only. Delayed wound healing« YOUR ANSWER Erectile dysfunction Page | Increased risk of hernia recurrence Loss of sensation over the scrotum « CORRECT ANSWER 203 Paralysis of the cremasteric muscle.

The ilioinguinal nerve supplies sensation to the skin over the medial aspect of the thigh, upper part of the scrotum and penile root, as well as motor innervation to the internal abdominal oblique & transversus abdominis muscles. It is often encountered during open hernia repair and may interfere with placement of the mesh.

It may be inadvertently damaged during surgery. Nerve entrapment is believed to be a cause of chronic pain following hernia repair. The ilioinguinal nerve is a branch of the first lumbar nerve (L1) and emerges laterally to the psoas, descends posterior to the kidney and enters the inguinal canal by piercing the transversus abdominis and internal oblique muscles (not by passing through the deep ring).

It then leaves the inguinal canal via the superficial inguinal ring. It is sensory to the skin of the lower abdomen and some of the genital area. The ilioinguinal (and iliohypogastric) nerves carry motor fibres to the external and internal obliques and the transversus abdominis. They travel in the anterior abdominal wall between the transversus and internal oblique muscles close to McBurney‟s point – so if they are damaged after a grid-iron incision (eg for appendicectomy), the muscle fibres in the conjoint tendon will be paralysed and this may give rise to an indirect inguinal hernia.

Damage of the ilioinguinal nerve at the time of hernia repair does not increase the risk of hernia recurrence. It is sometimes transected on purpose to avoid its entrapment in the hernia repair which would cause chronic groin pain. The cremasteric muscle is supplied by the genital branch of the genitofemoral nerve.

193. A 36-year-old female presents with obstructive jaundice, upper abdominal pain and fever. She is diagnosed with ascending cholangitis secondary to gallstones. Endoscopic retrograde pancreatography confirms the presence of gallstones within the common bile duct. Which ligament contains the duct as it passes inferiorly from the liver? Single best answer - select one answer only. Coronary ligament« YOUR ANSWER Hepatoduodenal ligament« CORRECT ANSWER Hepatophrenic ligament

------Dr Mohammed Shamsul Islam Khan, Medical Officer, Clinical Neuro-Surgery, National Institute of Neuro-Sciences and Hospital Sher-E-Bangla Nagar, Dhaka-1207, Bangladesh. Mobile: +880 1713 455 662, E-mail: [email protected]

MyPasTest: MRCS A Online - Jan Exam 2015 03. Anatomy; Aabdomen, Pelvis & Perineum (290Qs) ------Triangular ligament.

The common bile duct is formed of the confluence of the hepatic bile duct and the cystic duct. It runs with the hepatic portal vein and hepatic artery proper within the hepatoduodenal ligament in the free edge of the lesser omentum, and enters the gastrointestinal tract at the second part of the duodenum. Page |

194. Which organ lies anterior in direct contact with the left kidney without separation by 204 visceral peritoneum? Single best answer question – choose ONE true option only. Spleen« YOUR ANSWER Left suprarenal Tail of the pancreas« CORRECT ANSWER Left psoas muscle Splenic flexure. A small area along the upper part of the medial border of the left kidney is in relation with the left suprarenal gland, and close to the lateral border is a long strip in contact with the renal impression on the spleen. A somewhat quadrilateral field, about the middle of the anterior surface, marks the site of contact with the body of the pancreas, on the deep surface of which are the lienal vessels.

Above this is a small triangular portion, between the suprarenal and splenic areas, in contact with the postero-inferior surface of the stomach. Below the pancreatic area, the lateral part is in relation with the left colic flexure, the medial with the small intestine.

The areas in contact with the stomach and spleen are covered by the peritoneum of the omental bursa, while that in relation to the small intestine is covered by the peritoneum of the general cavity; behind the latter are some branches of the left colic vessels. The suprarenal, pancreatic, and colic areas are devoid of peritoneum.

195. You are seeing a patient in clinic who has been recently diagnosed with muscle-invasive bladder cancer. He asks you what is the best treatment option. What is your response? Single best answer - select one answer only. BCG immunotherapy« YOUR ANSWER Hormonal manipulation Cystectomy and ileal conduit urinary diversion« CORRECT ANSWER Mitomycin C chemotherapy Transurtethral resection.

A number of treatment options exist for patients with muscle-invasive bladder cancer. These options consist of radiotherapy, radical surgery, or preoperative radiotherapy with subsequent cystectomy. Radical surgery consists of cystectomy with ileal conduit, cystectomy with orthotopic neobladder reconstruction, or cystectomy with continent urinary diversion.

------Dr Mohammed Shamsul Islam Khan, Medical Officer, Clinical Neuro-Surgery, National Institute of Neuro-Sciences and Hospital Sher-E-Bangla Nagar, Dhaka-1207, Bangladesh. Mobile: +880 1713 455 662, E-mail: [email protected]

MyPasTest: MRCS A Online - Jan Exam 2015 03. Anatomy; Aabdomen, Pelvis & Perineum (290Qs) ------Intravesical mitomycin C and BCG immunotherapy and transurthethral resection are used in the treatment of superficial disease. Hormonal manipulation is used in the treatment of other cancers.

196. Which of the following arteries branches directly from the coeliac trunk? Single best answer question – choose ONE true option only. Page | The right gastric artery« YOUR ANSWER The left gastric artery« CORRECT ANSWER 205 The superior pancreaticoduodenal artery The superior mesenteric artery The inferior mesenteric artery.

The short coeliac trunk arises from the aorta at T12. It trifurcates into the common hepatic, left gastric, and splenic arteries. The right gastric artery branches from the common hepatic artery. The superior pancreaticoduodenal artery is a branch of the gastroduodenal artery. The superior and inferior mesenteric arteries are branches of the aorta arising from the aorta at L1 and L3 respectively.

197. A 37-year-old female presents for an elective laparoscopic cholecystectomy following multiple attacks of acute cholecystitis. Laparoscopy reveals significant, dense adhesions, not amenable to safe adhesiolysis. The decision is made to convert the procedure to a laparotomy, performed through Kocher‟s incision. Which fascia will be divided first? Camper’s fascia« YOUR ANSWER (Correct) Colles’ fascia Denonvillier’s fascia Scarpa’s fascia Transversus abdominis fascia.

Camper‟s fascia is the first fascial layer encountered during laparotomy, and lies superior to the more robust Scarpa‟s fascia. The anterior rectus sheath covers the full length of the rectus abdominis muscle, but its posterior counterpart is absent below the arcuate line of the anterior abdominal wall. Colles‟ fascia is a continuation of Scarpa‟s fascia into the perineal tissues, and so is not encountered during upper abdominal laparotomy. Denonvillier‟s fascia is also referred to as the rectoprostatic fascia.

198. Radical nephrectomy is indicated for localised renal carcinoma. When performing a radical nephrectomy for localised renal carcinoma: Single best answer - select one answer only. Routine ipsilateral adrenalectomy is not indicated « YOUR ANSWER (Correct) The flank incision is particularly useful for tumours involving the upper pole The thoracolumbar approach is particularly useful for large lesions involving the lower pole The renal vein and artery are ligated simultaneously The renal vein must be ligated and divided prior to ligating the renal artery.

------Dr Mohammed Shamsul Islam Khan, Medical Officer, Clinical Neuro-Surgery, National Institute of Neuro-Sciences and Hospital Sher-E-Bangla Nagar, Dhaka-1207, Bangladesh. Mobile: +880 1713 455 662, E-mail: [email protected]

MyPasTest: MRCS A Online - Jan Exam 2015 03. Anatomy; Aabdomen, Pelvis & Perineum (290Qs) ------Routine ipsilateral adrenalectomy for renal tumours is not recommended where the adrenal gland appears normal on pre-operative tumour staging (CT, MRI) and intra-operatively where there is no intra-operative suspicion of involvemen. The thoracolumbar approach is particularly useful for lesions involving the upper pole and the flank from those involving the lower pole. The renal vein is ligated and divided after ligating the renal artery.

Please see latest BAUS guidelines for further information: Page | http://www.baus.org.uk/Resources/BAUS/Documents/PDF%20Documents/BAUS%20in%20gener al/MDT%20Renal%20Cancer%20Guidance.pdf 206

199. An 88-year-old female presents to the Emergency Department with recurrent painless abdominal distension and vomiting. She has not opened her bowels for 5 days. Examination reveals a distended, soft, mildly tender abdomen with reduced bowel sounds. Plain radiographs demonstrate distended loops of large bowel that appear unchanged from those on previous admissions for similar presentations. A diagnosis of colonic pseudo-obstruction is made. Which of the following is true? Single best answer - Select one answer only. A barium enema is likely to demonstrate a mechanical cause« YOUR ANSWER The condition most commonly occurs after physical trauma The condition is rarely associated with electrolyte disturbances Institutionalised and immobile elderly patients are at increased risk « CORRECT ANSWER The condition may successfully be treated by gastroscopy .

Chronic colonic pseudo-obstruction is characterised by symptoms of recurrent bowel obstruction, abdominal bloating and/or pain, evidence of bowel obstruction on radiographic imaging, and no evidence of anatomic/structural abnormality. Risk factors include cerebral palsy or neurological disorders; chronic renal, pulmonary or cardiac disease; prolonged immobility; and prolonged used of narcotic drugs and other anticholinergic drugs that reduce bowel motility.

Possible complications include diarrhoea (and associated electrolyte disturbances), intestinal perforation, nutritional deficiencies and weight loss. Colonoscopy is often successful in decompressing the colon, although symptomatic relief from this is often temporary.

200. A 26-year-old motorist is brought to the emergency department following a high speed head-on collision. He is resuscitated and proceeds to undergo pelvic radiography. His radiograph reveals a comminuted fracture of the left acetabulum. Which bones form the acetabulum? Single best answer - select one answer only. Ilium and pubic bone« YOUR ANSWER Ilium and ischium Ilium, ischium and pubic bone« CORRECT ANSWER Ilium, ischium and sacrum Ischium and pubic bone.

The acetabulum articulates with the femoral head to form the hip joint. It is formed of three bones, the ilium, ischium and the pubic bone. It is often fractured in road traffic accidents where

------Dr Mohammed Shamsul Islam Khan, Medical Officer, Clinical Neuro-Surgery, National Institute of Neuro-Sciences and Hospital Sher-E-Bangla Nagar, Dhaka-1207, Bangladesh. Mobile: +880 1713 455 662, E-mail: [email protected]

MyPasTest: MRCS A Online - Jan Exam 2015 03. Anatomy; Aabdomen, Pelvis & Perineum (290Qs) ------vehicles collide head on, driving the engine and steering column into the lower limbs, which drives the femur posteriorly into the acetabulum with force, causing a fracture.

201. A 73-year-old male presents with painless haematuria and weight loss. He undergoes diagnostic cystoscopy and is diagnosed with transitional cell carcinoma of the bladder. Which occupational carcinogen increases the risk of this disease? Single best answer - select one answer only. Page | Arsenic« YOUR ANSWER 207 Asbestos Kerosene Coal dust Rubber« CORRECT ANSWER.

People working in the rubber and dye industries are known to have a higher incidence of bladder cancer. Arsenic exposure increases the risk of lung carcinomas, as does coal dust. Mesotheliomas are known to be caused by asbestos exposure, while benzene increases the risk of some haematological cancers.

202. While performing a laparoscopic inguinal hernia repair in a 68-year-old man with a direct inguinal hernia, the surgeon asked the specialist registrar to look at the medial inguinal fossa to identify the direct inguinal hernia. To do so, the specialist registrar would have to look at the area between the: Single best answer question – choose ONE true option only. Inferior epigastric artery and urachus« YOUR ANSWER Medial umbilical ligament and urachus Inferior epigastric artery and lateral umbilical fold Linea semilunaris, inguinal ligament and inferior epigastric artery« CORRECT ANSWER Median umbilical ligament and medial umbilical ligament.

Remember: the medial umbilical fold is made by the medial umbilical ligament (the obliterated portion of the umbilical artery), while the lateral umbilical fold is a fold of peritoneum over the inferior epigastric vessels.

The median umbilical fold is a midline structure made by the median umbilical ligament (obliterated urachus). The medial inguinal fossa is the space on the inner abdominal wall between the medial umbilical fold and the lateral umbilical fold.

This is the place in the abdominal wall where there is an area of weak fascia called the inguinal triangle (i.e. “Hesselbach's triangle ”, bound medially by the linea semilunaris, superiorly by the inferior epigastric vessels, and inferiorly by the inguinal ligament)- direct inguinal hernias can break through this space.

The lateral inguinal fossa is a space lateral to the lateral umbilical fold - indirect inguinal hernias push through the deep inguinal ring in this space.

203. How would you describe the mesentery of the small bowel?

------Dr Mohammed Shamsul Islam Khan, Medical Officer, Clinical Neuro-Surgery, National Institute of Neuro-Sciences and Hospital Sher-E-Bangla Nagar, Dhaka-1207, Bangladesh. Mobile: +880 1713 455 662, E-mail: [email protected]

MyPasTest: MRCS A Online - Jan Exam 2015 03. Anatomy; Aabdomen, Pelvis & Perineum (290Qs) ------Contains arteries that supply the large bowel« YOUR ANSWER (Correct) Contains autonomic fibres of the phrenic nerve Contains veins which drain directly into the inferior vena cava Has a root overlying the left sacroiliac joint Page | Overlies the transverse mesocolon. 208 The veins in the mesentery are all tributaries of the portal system. The root of the small bowel mesentery extends from a point to the left of the L2 vertebra, at the duodenojejunal junction, down to the right sacroiliac joint.

The nerves are postganglionic sympathetic and preganglionic parasympathetic fibres (vagus nerve) from the superior mesenteric plexus. The superior mesenteric artery supplies the mid- gut, which includes the large bowel as far as the distal transverse colon. The transverse mesocolon lies anterior to the small bowel mesentery.

204. A 35-year-old builder has a left inguinal hernia repair. During the operation, the external oblique is opened and you commence lifting the spermatic cord. Which structure lies within? Single best answer - select one answer only. Cremasteric artery« YOUR ANSWER (Correct) External spermatic fascia Femoral branch of the genitofemoral nerve lioinguinal nerve Lipoma of the cord.

Nine structures lie within the spermatic cord: three arteries (testicular, cremasteric, artery to the vas deferens); Pampiniform plexus; vas deferens; the genital branch of the genitofemoral nerve; sympathetic fibres; lymphatics and the processus vaginalis. The ilioinguinal nerve enters the inguinal canal but not the spermatic cord.

205. A 78-year-old male with a history of atrial fibrillation presents with severe abdominal pain, leucocytosis and fever. At laparotomy, there is noted to be infarction of the ascending colon. Which branch of the aorta has an embolus passed through? Coeliac plexus« YOUR ANSWER Ileocolic artery Inferior mesenteric artery Superior mesenteric artery« CORRECT ANSWER Lumbar artery.

The superior mesenteric artery supplies all the abdominal structures arising from the embryological midgut, specifically the distal duodenum, the head of the pancreas, the jejunum, the ileum, the caecum, the ascending colon, and two thirds of the transverse colon. The ileocolic artery is a branch of the superior mesenteric artery.

------Dr Mohammed Shamsul Islam Khan, Medical Officer, Clinical Neuro-Surgery, National Institute of Neuro-Sciences and Hospital Sher-E-Bangla Nagar, Dhaka-1207, Bangladesh. Mobile: +880 1713 455 662, E-mail: [email protected]

MyPasTest: MRCS A Online - Jan Exam 2015 03. Anatomy; Aabdomen, Pelvis & Perineum (290Qs) ------206. A 58-year-old male undergoes abdominoperineal resection after developing a malignant melanoma of the anus. Colles‟ fascia is contiguous with which layer of the anterior abdominal wall? Aponeurosis of external oblique« YOUR ANSWER Camper’s fascia Page | Posterior rectus sheath Scarpa’s fascia« CORRECT ANSWER 209 Transversalis fascia.

Colles‟ fascia is perineal fascia that is contiguous with Scarpa‟s fascia of the abdominal wall. Scarpa‟s fascia is a superficial fascial layer that lies between Camper‟s fascia and the external oblique muscle. Transversalis fascia continues to become the internal spermatic fascia.

207. During an elective repair of an aortic aneurysm you are asked to identify a vessel that passes behind the inferior vena cava, the head of the pancreas and the duodenum. What is the correct answer? Single best answer - select one answer only. Common hepatic artery« YOUR ANSWER Left renal artery Right renal artery« CORRECT ANSWER Superior mesenteric artery Testicular artery. The right renal artery arises from the abdominal aorta at the level of the L1-2 vertebrae. It passes behind the inferior vena cava, the right renal vein, the head of the pancreas and the descending part of the duodenum before dividing proximal to the renal hilum. Branches also supply the right adrenal gland and the proximal right ureter.

208. A 67-year-old male presents with rectal bleeding, tenesmus and loose stools. Digital rectal examination reveals the presence of a fixed mass, and biopsy confirms a rectal adenocarcinoma. He proceeds to a low-anterior resection of the rectum. Into which vessel does the inferior mesenteric vein usually drain? Single best answer - select one answer only. Hepatic portal vein« YOUR ANSWER Inferior vena cava Left gastric veins Splenic vein« CORRECT ANSWER Superior mesenteric vein.

The inferior mesenteric vein (IMV) usually drains into the splenic vein, which itself proceeds to form the hepatic portal vein (HPV) in confluence with the superior mesenteric vein (SMV). Anatomical variation is more common in the venous system than the arterial system, and occasionally the IMV drains directly into either the SMV or the HPV.

209. A 34-year-old gentleman with Crohn's disease is admitted with vomiting, colicky peri- umbilical pain and abdominal distension. He has had a previous ileocolic resection. A CT scan ------Dr Mohammed Shamsul Islam Khan, Medical Officer, Clinical Neuro-Surgery, National Institute of Neuro-Sciences and Hospital Sher-E-Bangla Nagar, Dhaka-1207, Bangladesh. Mobile: +880 1713 455 662, E-mail: [email protected]

MyPasTest: MRCS A Online - Jan Exam 2015 03. Anatomy; Aabdomen, Pelvis & Perineum (290Qs) ------of the abdomen reveals multiple small bowel strictures. What should you be aware of regarding the management of this man? Single best answer - Select one answer only. Intravenous fluids do not have a role in managing this patient « YOUR ANSWER Intravenous hydrocortisone and parenteral nutrition are required« CORRECT ANSWER Stricturoplasty is immediately required Page | The patient needs a small bowel resection 210 The symptoms are definitely caused by adhesions.

In Crohn‟s disease the inflammation is classically described as transmural. „Rosethorn‟ ulcers are deep ulcers that traverse beyond the lamina propria and have a characteristic appearance. Apthous ulcers occur anywhere from the mouth to the anus.

Short-bowel syndrome is why we try to avoid surgery whenever possible with Crohn‟s disease, but in very severe cases where less than 50 cm of small bowel remains, malabsorption of essential fat soluble vitamins (A, D, E and K) as well as other essential nutrients requires lifelong parenteral nutrition. Crohn‟s colitis is not uncommon but the small bowel is more often involved and usually seen by the time a colitis occurs.

Crohn‟s spreads as „skip lesions‟ and, unlike ulcerative colitis, the spread is not usually in a continuous fashion. All of these options are possible in Crohn‟s disease. It is usually not possible to differentiate between inflammatory and fibrous strictures and the best course of management is a trial of medical therapy.

If this fails then surgery is indicated. The main aim of surgery is bowel preservation and strictureplasty is the primary option, if not then, small bowel resection together with strictureplasty.

210. A 48-year-old female had a laparoscopic cholecystectomy 1 day ago, and has severe abdominal pain. You diagnose a bile duct injury. Which of the following is true regarding this complication? Select one answer only. It has a lower incidence than after open cholecystectomy« YOUR ANSWER If major, it is usually treated with balloon dilatation It may be prevented by routine cholangiography It occurs in 10% of cases It usually results from mistaking the CBD for the cystic duct« CORRECT ANSWER.

Bile duct injury occurs in 0.5% of laparoscopic procedures and 0.25% of open procedures and it does not seem to be prevented by routine cholangiography. It is difficult to treat but may require hepaticojejunostomy. Major duct transection is obvious early because of abdominal pain. Minor injury with a stricture causes biliary cirrhosis and malabsorption.

211. You are asked to see a 76-year-old man in casualty who has presented with severe searing abdominal pain radiating to the back. He had a similar episode 2 days previously which self- resolved. He is a lifelong smoker and has had 2 myocardial infarcts in the past. On inspection,

------Dr Mohammed Shamsul Islam Khan, Medical Officer, Clinical Neuro-Surgery, National Institute of Neuro-Sciences and Hospital Sher-E-Bangla Nagar, Dhaka-1207, Bangladesh. Mobile: +880 1713 455 662, E-mail: [email protected]

MyPasTest: MRCS A Online - Jan Exam 2015 03. Anatomy; Aabdomen, Pelvis & Perineum (290Qs) ------he is obese which makes examination difficult. Bruising around the umbilicus is noted. He is tachycardic and in pain. Which of the following is the most likely diagnosis? Haemorrhagic pancreatitis« YOUR ANSWER Infarcted descending colon Liver infarction Page | Ruptured abdominal aortic aneurysm« CORRECT ANSWER 211 Ruptured spleen.

Both liver and spleen are invested with peritoneum, whereas the other three structures are to be found attached to the posterior abdominal wall covered by the peritoneum.

Note that bleeding from the liver and spleen will result in peritoneal blood, but pancreatic and aortic aneurysmal bleeding will cause a retroperitoneal haematoma – often manifesting as flank (Grey-Turner‟s sign) or peri-umbilical (Cullen‟s sign) discoloration.

Despite the descending colon being retroperitoneal it is not associated with either of these two eponymous signs. In this scenario, the most likely diagnosis is a ruptured AAA due to the presence of a warning bleed and the history of cardiovascular risk factors.

212. A 28-year-old professional cyclist presents to the urology clinic complaining of penile and scrotal numbness, associated with urinary incontinence. Which nerve has been injured? Single best answer - select one answer only. Genitofemoral nerve« YOUR ANSWER Obturator nerve Post-ganglionic parasympathetic nerves Pudendal nerve« CORRECT ANSWER Sympathetic nerves.

The pudendal nerve is formed by the ventral rami of the sacral spinal nerves from S2, S3 and S4. It maintains voluntary urinary continence by facilitating contraction of the external urethral sphincter. It also helps to maintain voluntary faecal continence via contraction of the external anal sphincter. Cyclists may develop pudendal nerve entrapment syndrome, which can present with various symptoms including pain, urinary and faecal incontinence and sensory disturbances of the perineum and genitalia.

213. You are seeing a patient in clinic with complete rectal prolapse, and she asks about the potential complications of conservative treatment of this condition. Which of the following would you tell her is the commonest complication? Select one answer only. Incarceration« YOUR ANSWER Incontinence« CORRECT ANSWER Rectal carcinoma Strangulation ------Dr Mohammed Shamsul Islam Khan, Medical Officer, Clinical Neuro-Surgery, National Institute of Neuro-Sciences and Hospital Sher-E-Bangla Nagar, Dhaka-1207, Bangladesh. Mobile: +880 1713 455 662, E-mail: [email protected]

MyPasTest: MRCS A Online - Jan Exam 2015 03. Anatomy; Aabdomen, Pelvis & Perineum (290Qs) ------Ulceration.

Partial rectal prolapse occurs in children. Complete rectal prolapse is a disease of elderly women. All of the conditions listed are complications of complete rectal prolapse. The commonest of these is incontinence which may actually be the patient's presenting symptom. Page | 214. A 26-year-old woman presents to the Emergency Department with right iliac fossa peritonism, pyrexia and vomiting. A diagnostic laparoscopy reveals appendicitis which is duly 212 resected. Regarding the Alvarado score, which of the following is correct? Single best answer - select one answer only. Appendicitis is associated with a right shift in the neutrophil profile« YOUR ANSWER Appendicitis is associated with migratory right iliac fossa pain« CORRECT ANSWER Appendicitis is unlikely with a score over 5 Does not count rebound tenderness in the marking scheme Is associated with a higher mortality if the score is over 13.

Acute appendicitis has an associated mortality of 0.25% and a morbidity of 7.7%. The peak incidence is towards 30 years of age with a second peak at 70 years. It has an overall incidence of 10% of the population. It is marginally more common in males (1.6x). A high retrocaecal appendix may mimic cholecystitis, and appendicitis itself may be caused by a caecal carcinoma which occludes the appendix lumen. The Alvarado score is a clinical and laboratory based scoring system marked out of 10 which assesses the likelihood of the diagnosis of appendicitis. If <5 appendicitis is unlikely, 5-6 is equivocal and may require scanning and >7 is strongly predictive. Migratory RIF pain (1 point), anorexia (1 point), N and V (1point), RIF tenderness (2 points), rebound tenderness (1 point), fever (1 point), leucocytosis (2 points), left shift in neutrophils (1 point).

215. Which of the following is correct regarding the testis? It is covered by the thick fibrous tunica vaginalis« YOUR ANSWER It usually descends after birth Sympathetic nerve fibres reach the organ via the splanchnic nerves« CORRECT ANSWER The testicular lymph drains into the internal iliac nodes The testicular vein runs alongside the artery in the spermatic cord.

The testis is usually descended by birth having developed from the gonadal ridge on the posterior abdominal wall. Its thick, whitish covering is the tunica albuginea. Lymphatic drainage follows the arterial supply to the para-aortic nodes. The spermatic cord contains the pampiniform plexus – a single testicular vein is formed more distally over psoas major. The nerve supply is via the lesser splanchnic nerve and the coeliac ganglion with postganglionic fibres accompanying the testicular arteries.

216. A 47-year-old female with a history of alcohol abuse presents with jaundice, hepatomegaly and abdominal ascites. She is diagnosed with portal hypertension. Which structure lies

------Dr Mohammed Shamsul Islam Khan, Medical Officer, Clinical Neuro-Surgery, National Institute of Neuro-Sciences and Hospital Sher-E-Bangla Nagar, Dhaka-1207, Bangladesh. Mobile: +880 1713 455 662, E-mail: [email protected]

MyPasTest: MRCS A Online - Jan Exam 2015 03. Anatomy; Aabdomen, Pelvis & Perineum (290Qs) ------immediately anterior to the origin of the hepatic portal vein? Single best answer - select one answer only. Duodenum« YOUR ANSWER Gall bladder Pancreatic body Page | Pancreatic head 213 Pancreatic neck« CORRECT ANSWER.

The hepatic portal vein is formed of the confluence of the splenic and superior mesenteric veins, just posterior to the pancreatic neck. There is some anatomical variance, specifically that the inferior mesenteric vein may also confluence to form the portal vein.

217. A 2-year-old child attends the clinic because the parents are worried that one of his testes is missing. On examination the testis is absent from the right hemiscrotum. It is palpable within the inguinal canal and cannot be coaxed into the scrotum. What is the best course of action? Advise an orchidopexy to reduce the risk of testicular malignancy« YOUR ANSWER Inform the parents that the risk of testicular malignancy is high as long as the testis is in the inguinal canal. Keep the child under review for one year and perform an orchidopexy if the testis has not descended Perform an orchidopexy and fix the opposite testis at the same time« CORRECT ANSWER Reassure the parents that the testis will always descend and no action is required.

Maldescended testis occurs in 2% of boys. The right side is more commonly affected. Maldescended testes are usually hypoplastic. There is an increased incidence of malignancy, infection and trauma.

Treatment is usually surgical with orchidopexy. Such a treatment does not significantly reduce the malignancy risk. Other treatment modalities include human chorionic gonadotrophin (HCG) and luteinising hormone-releasing hormone (LHRH).

By the age of 2, the testis is unlikely to descend. It is preferable to perform an orchidopexy at this stage before the child becomes of school age. Up to a third of patients with a unilateral undescended testis have a contralateral patent processus vaginalis (and therefore increased risk of contralateral cryptorchidism) - contralateral orchidopexy is therefore also performed.

218. Which of the following best describes the male urethra? Bends 90° in moving from the membranous to prostatic part« YOUR ANSWER Has an external sphincter proximal to the verumontanum Has penile, membranous and prostatic parts« CORRECT ANSWER Is entirely lined with transitional cell epithelium Is narrowest at the prostatic part.

------Dr Mohammed Shamsul Islam Khan, Medical Officer, Clinical Neuro-Surgery, National Institute of Neuro-Sciences and Hospital Sher-E-Bangla Nagar, Dhaka-1207, Bangladesh. Mobile: +880 1713 455 662, E-mail: [email protected]

MyPasTest: MRCS A Online - Jan Exam 2015 03. Anatomy; Aabdomen, Pelvis & Perineum (290Qs) ------

The male urethra has penile (spongy), membranous, prostatic and pre-prostatic parts. It is widest at the prostatic part. It is narrowest at the external urethral meatus. Just proximal to this is a dilatation called the navicular fossa. This is lined with stratified squamous epithelium.

The right angle bend in the urethra is between the penile and membranous parts - important to know when attempting urethral catheterisation. The external urethral sphincter is distal to the Page | verumontanum, so during a trans urethral resection of the prostate the resection is limited to above the verumontanum to avoid incontinence. 214

219. A 75-year-old gentleman attends clinic with frequency of micturition, terminal dribbling and nocturia. He has a Prostate Specific Antigen (PSA) level of 10 nmol/l. He is due to have a prostatectomy. The following is applicable to the Prostate gland, PSA and Prostatectomy: A PSA of 10 nmol/l indicates a 40% risk of Prostate cancer« YOUR ANSWER Digital rectal examination has a 90% predictive value for disseminated disease« CORRECT ANSWER Early incontinence following prostatectomy occurs in 25% of patients Mortality following prostatectomy is 1% 60% of men with localized moderately differentiated prostate cancer are symptom free at 10 years.

Each ejaculatory duct passes obliquely through the prostate gland to open on the verumontanum. The prostate is mainly supplied from the prostatic branch of the inferior vesical artery (a branch of the internal iliac), with small branches from the middle rectal and internal pudendal vessels. Most carcinomas develop in the peripheral zone.

The prostate provides about 30% of the volume of seminal fluid, most of it comes from the seminal vesicles. A PSA of 4-10 nmol/ml indicates a prostate cancer risk of 22% and above 10 nmol/ml a risk of 63%. Digital rectal examination has an 80-93% predictive value for widespread disease.

40% of men with moderately differentiated localised disease are symptom free at 10 years. Complications from prostatectomy are early and late. Early ones include mortality of 0.1%, bleeding, pulmonary embolus and myocardial infarction. Late complications include early incontinence in up to 33% of patients, erectile dysfunction in up to 50% and significant stress incontinence in <5% by one year.

220. A 3-week-old boy presents with vomiting, jaundice and dehydration. Investigations reveal hypokalaemia and metabolic alkalosis. What is the most appropriate initial management? Single best answer - select one answer only. Correction of metabolic derangements« YOUR ANSWER (Correct) Feeding jejunostomy Ramstedt’s pyloromyotomy Total parenteral nutrition Upper GI endoscopy.

------Dr Mohammed Shamsul Islam Khan, Medical Officer, Clinical Neuro-Surgery, National Institute of Neuro-Sciences and Hospital Sher-E-Bangla Nagar, Dhaka-1207, Bangladesh. Mobile: +880 1713 455 662, E-mail: [email protected]

MyPasTest: MRCS A Online - Jan Exam 2015 03. Anatomy; Aabdomen, Pelvis & Perineum (290Qs) ------The incidence of infantile pyloric stenosis (IHPS) is 3–4/1000 (1 in 300–900) live births. It is more common in males and has a slightly higher incidence in the first born. If the mother had IHPS, the risk is 20%, the father IHPS 5%. The pylorus is increased in length and diameter with hypertrophy of the circular muscle layer and the autonomic nerves.

There may be jaundice in 5–10 % of cases due to a reduction of glucuronyl transferase. The Page | tumour is diagnosed as a palpable tumour on test feed in 90% of cases depending on the experience of the clinician, a history of projectile vomiting and hungry feeding with no bile in 215 the vomitus.

The classical electrolyte abnormality is a hypokalaemic hypochloraemic alkalosis. Surgery should be undertaken only after careful correction of the abnormalities. The umbilicus should be excluded from the operative field because of risk of Staphylococcus aureus infection. The duodenal limit of the pyloromyotomy is the prepyloric vein of Mayo.

221. A 12-year-old boy presents with acute testicular pain and a „blue dot‟ on the scrotum. Which embryological remnant is affected? Single best answer - select one answer only. Gartner’s duct« YOUR ANSWER Hydatid of Morgagni« CORRECT ANSWER Meckel’s diverticulum Urachal cyst Urachal sinus.

The Hydatid of Morgagni (also known as the appendix testis) is an embryological remnant of the Mullerian duct that is present in 90% of males. It is located at the upper pole of the testis, attached to the tunica vaginalis.

It can become torted, leading to acute testicular pain and occasionally requiring surgical treatment. Gartner‟s duct is found in females and is a remnant of the mesonephric duct, occasionally remaining in the vaginal wall or the broad ligament.

222. Which of the vessels listed below drain(s) into the inferior vena cava? Single best answer question – choose ONE true option only. Renal veins« YOUR ANSWER (Correct) Inferior mesenteric vein Superior mesenteric vein Splenic vein All of the above.

------Dr Mohammed Shamsul Islam Khan, Medical Officer, Clinical Neuro-Surgery, National Institute of Neuro-Sciences and Hospital Sher-E-Bangla Nagar, Dhaka-1207, Bangladesh. Mobile: +880 1713 455 662, E-mail: [email protected]

MyPasTest: MRCS A Online - Jan Exam 2015 03. Anatomy; Aabdomen, Pelvis & Perineum (290Qs) ------The renal veins drain into the inferior vena cava (remember the renal vein is longer on the left). The superior and inferior mesenteric veins, along with the splenic vein join to form the portal vein and drain proximally into the liver rather than directly into the inferior vena cava.

223. A 32-year-old man is referred to your general surgical clinic complaining of difficulty in swallowing both solids and liquids over the past 2 years. In addition he complains of Page | retrosternal chest pain and 1 stone of weight loss over the last 6 months. He is a moderate drinker and has a 20 pack year smoking history. Which of the following statements best 216 represents this gentleman‟s likely condition? Single best answer - select one answer only. The most likely differential diagnosis in this gentleman is oesophageal carcinoma.« YOUR ANSWER In oesophageal carcinoma, dysphagia is progressive like achalasia Oesophageal achalasia which can be treated by dilating the lower oesophagus and associated sphincter« CORRECT ANSWER Achalasia which is characterised by incomplete lower oesophageal sphincter relaxation, decreased LOS tone and increased peristalsis of the oesophagus Although Achalasia can be primary (affecting 1/100,000 individuals per year) it is much more commonly associated with other conditions.

Achalasia is a condition caused by failure of development of the lower oesophageal inhibitory neurons and involves the smooth muscle layers of the oesophagus and lower oesophageal sphincter (LOS). It is characterised by incomplete LOS relaxation, increased LOS tone and lack of oesophageal peristalsis.

In achalasia, patients have equal difficulty in swallowing both liquids and solids. In carcinoma, difficulties begin with swallowing solids and progress to include liquids. The most common form is primary achalasia but it also occurs associated with other conditions such as cancer or Chagas‟ disease but much more infrequently.

Treatment should initially be by lifestyle modification and medications to reduce LOS pressure (eg. nifedipine). Invasive treatments include balloon dilatation and also Heller‟s cardiomyotomy together with a partial fundoplication to prevent excessive reflux.

224. You are examining a patient with a scrotal lump, which is non-tender, surrounds the testis and is cystic. What is the most likely diagnosis? Single best answer - Select ONE correct answer only. Epidydimal cyst« YOUR ANSWER Gumma Hydrocoele« CORRECT ANSWER Teratoma Varicocoele.

Differential diagnosis of a scrotal lump can be divided into lumps separate to the testis, around the testis and within the testis. Lumps separate to the testis include epidydimal cysts,

------Dr Mohammed Shamsul Islam Khan, Medical Officer, Clinical Neuro-Surgery, National Institute of Neuro-Sciences and Hospital Sher-E-Bangla Nagar, Dhaka-1207, Bangladesh. Mobile: +880 1713 455 662, E-mail: [email protected]

MyPasTest: MRCS A Online - Jan Exam 2015 03. Anatomy; Aabdomen, Pelvis & Perineum (290Qs) ------epidydimitis-orchitis, spermatocoele, varicocoele and indirect inguinal hernia. Lumps within the testis include tumours, granuloma and gumma. Lumps around the testis include hydrocoeles.

225. A 38-year-old female undergoes an elective hysterectomy. From which embryological structure does the uterus develop? Allantois« YOUR ANSWER Page | Gubernaculum 217 Mullerian duct« CORRECT ANSWER Wolffian duct Processus vaginalis.

The Mullerian (or „paramesonephric‟) duct is the embryological, mesodermal precursor to the uterus, cervix, Fallopian tubes and the upper two-thirds of the vagina. It degenerates in male development in the presence of anti-Mullerian hormone produced by the testes. The Wolffian (or „mesonephric‟) duct is crucial to male urogenital development and regresses in females.

226. Which of the following is correct regarding the anatomy of the inferior vena cava (IVC): Commences at the level of the fourth lumbar vertebra« YOUR ANSWER Drains the portal vein at the porta hepatis Forms the anterior boundary of the epiploic foramen Lies posterior to the right renal artery Lies posterior to the third part of the duodenum« CORRECT ANSWER.

The IVC commences at the level of the fifth lumbar vertebra. Its path extends superiorly, posterior to the third part of the duodenum, over the right renal artery and forming the posterior wall of the epiploic foramen before grooving the visceral surface of the liver. It drains the hepatic veins from the substance of the liver, after the portal blood has passed through the liver sinusoids.

227. A 56-year-old alcoholic presents with massive upper gastrointestinal bleeding and is too unstable to undergo emergency gastroscopy. A Sengstaken-Blakemore tube is applied to achieve haemostasis. The bleeding has arisen from which vessels? Single best answer - select one answer only. Left gastric arteries« YOUR ANSWER Left gastric veins Oesophageal arteries Oesophageal veins Superficial oesophageal veins« CORRECT ANSWER.

Oesophageal varices are a sequala of portal hypertension. The superficial oesophageal veins drain into the left gastric vein, which continues to drain into the hepatic portal vein. The oesophageal veins do not drain into the portal system and so are not affected by portal

------Dr Mohammed Shamsul Islam Khan, Medical Officer, Clinical Neuro-Surgery, National Institute of Neuro-Sciences and Hospital Sher-E-Bangla Nagar, Dhaka-1207, Bangladesh. Mobile: +880 1713 455 662, E-mail: [email protected]

MyPasTest: MRCS A Online - Jan Exam 2015 03. Anatomy; Aabdomen, Pelvis & Perineum (290Qs) ------hypertension. Bleeding from oesophageal varices is frequently massive, and if the surgeon is unable to achieve haemostasis using variceal ligation or sclerotherapy, then tamponade is attempted using a Sengstaken-Blakemore tube.

228. You are assisting in an open elective abdominal aortic aneurysm repair and your consultant points to an artery that comes directly from the branch of the aorta given off at the level of T12. Which of these is that artery most likely to be? Select one answer only. Page | The gastroduodenal artery« YOUR ANSWER 218 The inferior pancreaticoduodenal artery The right gastric artery The splenic artery« CORRECT ANSWER The superior pancreaticoduodenal artery.

As soon as the aorta passes below the aortic hiatus, it gives off the celiac plexus (T12). The plexus has three direct branches: left gastric, hepatic and splenic (mnemonic: left-hand side). The right gastric and gastroduodenal arteries are branches of the hepatic artery. The superior pancreaticoduodenal artery is a branch of the gastroduodenal artery. The inferior pancreaticoduodenal artery is a branch of the superior mesenteric artery.

229. What is the most likely presentation of abdominal aortic aneurysms (AAA)? Single best answer - select one answer only. Acute lower limb ischaemia« YOUR ANSWER Gastrointestinal (GI) bleed Incidental finding« CORRECT ANSWER Severe abdominal pain radiating to the back Trash foot.

The commonest presentation of AAA is asymptomatic, as an incidental finding during clinical examination or a radiological investigation in approximately 75% of cases. Thrombosis and emboli can be thrown off from the aneurysm leading to acute lower limb ischaemia and trash foot. Severe abdominal pain radiating to the back is frequently the presenting symptom. This is a sign of expansion or, if sudden and acute, of potential rupture. An aorto-enteric fistula (though rare) can lead to an aneurysm presenting as a gastrointestinal (GI) bleed.

230. A 5-year-old African child presents with a ventral abdominal wall hernia and hypothyroidism. What is the most likely diagnosis? Single best answer - select one answer only. Epigastric hernia« YOUR ANSWER Inguinal hernia Paraumbilical hernia Spigelian hernia Umbilical hernia« CORRECT ANSWER. ------Dr Mohammed Shamsul Islam Khan, Medical Officer, Clinical Neuro-Surgery, National Institute of Neuro-Sciences and Hospital Sher-E-Bangla Nagar, Dhaka-1207, Bangladesh. Mobile: +880 1713 455 662, E-mail: [email protected]

MyPasTest: MRCS A Online - Jan Exam 2015 03. Anatomy; Aabdomen, Pelvis & Perineum (290Qs) ------

Umbilical hernias are most frequently seen in low birthweight and black infants. Although it is a feature of hypothyroidism, most infants with umbilical hernias do not have hypothyroidism. The majority of hernias will disappear by one year of age, strangulation is rare, and operation is only indicated if the hernia persists until age 3–4 years, causes symptoms, or becomes progressively larger. Page |

231. Your consultant is embarking on a lengthy laparotomy for an 87-year-old patient with 219 perforated diverticular disease. The anaesthetists are concerned about maintaining the patient‟s temperature during the operation. Which of the following would be most effective in reducing heat loss from the patient during the procedure? Single best answer - select one answer only. Humidification of inspired gases« YOUR ANSWER Lavage with warm saline Maintaining the environmental temperature at 20 ºC Use of a heated blanket set at 35 ºC Use of a heat-reflecting blanket« CORRECT ANSWER.

Anaesthesia obtunds the normal mechanisms of heat production, so the patient is vulnerable to hypothermia from conductive, convective and evaporative heat loss. Theatre temperature is usually maintained between 22 °C and 24 °C (but may need to be higher for neonatal surgery).

Humidification inspired gases and using warmed infusions attempt to raise/maintain the core temperature. Using a BAIR hugger (heated blanket) acts to warm the patient rather than reduce losses and can be on 3 settings low (32 deg +/- 3), medium (38 deg +/- 3) and high (43 deg +/- 3).

Covering exposed surfaces with reflective blankets will be most effective in minimising heat loss.

232. A young man developed a boil on his scrotum. Which of the following lymph nodes are most likely to enlarge in this patient due to lymphatic spread of infection? Single best answer question – choose ONE true option only. Internal iliac nodes« YOUR ANSWER Sacral nodes Superficial inguinal nodes« CORRECT ANSWER Lumbar nodes External iliac nodes.

The perineum and the external genitalia, including the scrotum and labia majora, drain to the superficial inguinal lymph nodes. However, in men, remember that the testes do not drain to the superficial inguinal lymph nodes! The lymphatic vessels for the testes travel in the spermatic cord and drain the testes into the lumbar nodes (ovaries also drain to lumbar nodes). The internal iliac nodes drain the pelvis and gluteal region. The lumbar nodes drain the internal pelvic organs. The sacral nodes drain the prostate gland, uterus, vagina, rectum and posterior pelvic wall; the external iliac nodes drain the lower limb.

------Dr Mohammed Shamsul Islam Khan, Medical Officer, Clinical Neuro-Surgery, National Institute of Neuro-Sciences and Hospital Sher-E-Bangla Nagar, Dhaka-1207, Bangladesh. Mobile: +880 1713 455 662, E-mail: [email protected]

MyPasTest: MRCS A Online - Jan Exam 2015 03. Anatomy; Aabdomen, Pelvis & Perineum (290Qs) ------

233. A 23-year-old male is stabbed in his abdomen. He is taken to theatre for an emergency laparotomy and is noted to have a significant liver injury. The consultant is concerned at the severity of the haemorrhage and clamps the hepatoduodenal ligament. Which vessels have been occluded? Coeliac trunk + hepatic portal vein« YOUR ANSWER Page | Hepatic artery + gastroduodenal artery 220 Hepatic artery + hepatic portal vein« CORRECT ANSWER Right heptic artery + hepatic portal vein Right hepatic artery + gastroduodenal artery.

The hepatoduodenal ligament is the part of the lesser omentum that attaches the porta hepatis to the superior duodenum. It contains the portal triad of the hepatic artery, hepatic portal vein and the common bile duct. It can be manually or instrumentally compressed in cases of acute and severe hepatic haemorrhage (known as Pringle‟s manoeuvre).

234. A 54-year-old woman presents to the emergency department with severe upper abdominal pain which is constant and localised to the RUQ. She is tachycardic and pyrexial with a +ve Murphy‟s sign. A diagnosis of cholecystitis is made. She is treated with antibiotics and is offered a 'hot' cholecystectomy. Concerning this diagnosis and its treatment, which of the following statements is correct? Single best answer - select one answer only. Boas’ sign is pain in the right upper quadrant which radiates to the right iliac fossa as inflammatory fluid tracks down the right pericolic gutter« YOUR ANSWER Hartmann’s pouch may be found at the junction of the cystic duct and common hepatic duct The cystic artery may be found in Calot’s triangle« CORRECT ANSWER The bile duct lies in the free edge of the Ultrasound is more sensitive than CT in the diagnosis of cholecystitis.

Hartmann‟s pouch is found at the junction of the gall-bladder neck and the cystic duct. It is a pathological variant where a stone may become impacted. The cystic artery runs across a triangle formed by the liver, common hepatic duct and cystic duct – Calot‟s triangle. The common hepatic duct is anterior to the portal vein and the bile duct does runs in the free edge of the lesser omentum.

Boas‟ sign is hypersensitivity below the right scapula and can also be caused by phrenic irritation. Although CT scanning has a higher sensitivity and specificity (90-95%) when compared with USS (sens 88% and spec 80%) it is more expensive and can‟t visualise non- calcified gallstones.

235. You are consenting a patient for laparoscopic cholecystectomy and she asks which is the commonest of the complications you have mentioned (as listed below). How would you respond? Select one answer only. Enterocutaneous fistula« YOUR ANSWER Haemorrhage« CORRECT ANSWER

------Dr Mohammed Shamsul Islam Khan, Medical Officer, Clinical Neuro-Surgery, National Institute of Neuro-Sciences and Hospital Sher-E-Bangla Nagar, Dhaka-1207, Bangladesh. Mobile: +880 1713 455 662, E-mail: [email protected]

MyPasTest: MRCS A Online - Jan Exam 2015 03. Anatomy; Aabdomen, Pelvis & Perineum (290Qs) ------Incisional hernia Pneumothorax Small bowel obstruction.

All of the problems listed are recognised complications of laparoscopic cholecystectomy. Page | Haemorrhage is the commonest of these. Obstruction occurs due to adhesions or port site hernia. Pneumothorax may occur as a result of a diaphragmatic injury using from the diathermy 221 hook. Diathermy injury may cause a fistula.

236. You are treating a patient with ischaemic bowel due to an inferior mesenteric artery thrombosis. Which one of the following regions of the bowel will be affected? Single best answer - Select one correct answer only. Ascending colon« YOUR ANSWER Ileum Jejunum Descending colon« CORRECT ANSWER Third part of duodenum.

The inferior mesenteric artery supplies the left third of transverse colon, descending and sigmoid colon and rectum to the dentate line. The remainder of the sections of bowel mentioned are supplied by the superior mesenteric artery.

237. The following are all definite indications for splenectomy, except: Single best answer - select one answer only. Benign splenic neoplasm« YOUR ANSWER Portal hypertension« CORRECT ANSWER Splenic artery aneurysm Splenic echinococcus cyst Splenic vein thrombosis with left-sided portal hypertension.

Definite indications are when splenectomy is essential. These include: 1. primary benign or malignant neoplasm of the spleen 2. splenic abscess 3. splenic echinococci cyst 4. splenic venous thrombosis with left-sided portal hypertension and gastric varices 5. splenic artery aneurysm 6. radical operation for malignant lesion of the stomach, tail of the pancreas or hepatic flexure 7. staging for Hodgkin's disease (rare) 8. splenic injury not amenable for salvage

238. A 36-year-old female presents with haematuria, uraemia and a palpable right kidney. Ultrasound scan confirms the presence of multiple renal cysts and she proceeds to ------Dr Mohammed Shamsul Islam Khan, Medical Officer, Clinical Neuro-Surgery, National Institute of Neuro-Sciences and Hospital Sher-E-Bangla Nagar, Dhaka-1207, Bangladesh. Mobile: +880 1713 455 662, E-mail: [email protected]

MyPasTest: MRCS A Online - Jan Exam 2015 03. Anatomy; Aabdomen, Pelvis & Perineum (290Qs) ------nephrectomy. What structure lies immediately anterior to the right renal artery as it enters the renal hilum? Single best answer - select one answer only. Ascending colon« YOUR ANSWER Duodenum Pancreatic head Page | Right renal vein« CORRECT ANSWER 222 Right ureter.

The renal vein lies immediately anterior to the renal artery as it enters the hilum. The ureter is a posterior relation to the artery.

239. In hepatobiliary surgery which of the following is correct? Single best answer - Select ONE true answer only. The common bile duct runs anterior to the portal vein in the free edge of the lesser omentum« YOUR ANSWER (Correct) The hepatic artery runs to the right of the common bile duct The common bile duct contains non-striated muscle in its wall Calot’s triangle is bordered by the cystic duct, common bile duct and liver The cystic artery usually arises from the left hepatic artery.

The common bile duct contains no muscle. Calot‟s triangle is bounded by the cystic duct, liver and common hepatic duct.

240. A 78-year-old male presents with acute retention of urine after complaining of symptoms of difficulty in passing urine and dribbling over the past few months. The first line of management is to insert a urinary catheter to drain the bladder. Which is the narrowest part of the normal male urethra? Single best answer - select one answer only. Membranous« YOUR ANSWER (Correct) Penile Pre-prostatic Prostatic Verumontanum.

The membranous portion of the urethra is the narrowest part, and passes through the external urethral sphincter. The verumontanum is not part of the urethra, but is the collective name for the ductile entrances into the prostatic urethra.

241. You are about to embark on an open appendicectomy for a 56-year-old man. Whilst undertaking your pre-operative WHO checklist it appears the diathermy plate has not yet been applied. Which of the following most accurately describes the principles of surgical diathermy? Coagulation is produced by a continuous current with a square wave form« YOUR ANSWER Is dangerous with earthed metal ------Dr Mohammed Shamsul Islam Khan, Medical Officer, Clinical Neuro-Surgery, National Institute of Neuro-Sciences and Hospital Sher-E-Bangla Nagar, Dhaka-1207, Bangladesh. Mobile: +880 1713 455 662, E-mail: [email protected]

MyPasTest: MRCS A Online - Jan Exam 2015 03. Anatomy; Aabdomen, Pelvis & Perineum (290Qs) ------Involves a high frequency current in the range of 30MHz to 300 MHz Is contraindicated in patients with a pacemaker With bipolar diathermy two electrodes are combined in the instrument as opposed to monopolar diathermy« CORRECT ANSWER. Page | Diathermy uses the heating effect of electrical current at a frequency of 300kHz to 3 MHz. Although diathermy can be used in a patient with a pacemaker, the pad of the monopolar 223 diathermy should be positioned well away from the pacemaker.

Bipolar diathermy which combines two electrodes in the forceps does not interfere with pacemakers as the current does not travel through the patient and may be more suitable. Coagulation involves an interrupted pulse of current (50-100/second) and a square wave form whereas cutting is produced by a continuous current with a sinus wave form.

242. A 45-year-old man was admitted with right sided abdominal pain and his investigations include an abdominal ultrasound scan. The ultrasound reveals a right sided hydronephrosis with a dilated right ureter. Which of the inflammatory processes listed below might impinge upon the right ureter and cause obstruction? Single best answer question – choose ONE true option only. Acute appendicitis« YOUR ANSWER A perforated caecal carcinoma Crohn’s disease affecting the terminal ileum All of the above« CORRECT ANSWER None of the above.

Although one should think of a calculus in the distal right ureter as the cause of his pain. One must consider acute appendicitis, Crohns disease, and a perforated caecal carcinoma as possible causes of his symptoms and ultrasonic findings of a right sided hydronephrosis and hydroureter. This is because anteriorly the right ureter is related to the terminal ileum, caecum, appendix, and ascending colon and their .

243. A 25-year-old gentleman complains of shortness of breath which worsens on walking. A chest X-ray reveals an elevated left diaphragm with right mediastinal shift. What would this X- ray show? A diaphragm that is partially supplied with motor fibres by the intercostal nerves« YOUR ANSWER Contraction during forced expiration The diaphragm has its left dome at a higher level than its right dome Diaphragmatic paresis« CORRECT ANSWER Pneumothorax.

The central part of the diaphragm is mainly derived from the septum transversum, whereas its periphery has body wall components. The intercostal nerves provide sensory fibres to the

------Dr Mohammed Shamsul Islam Khan, Medical Officer, Clinical Neuro-Surgery, National Institute of Neuro-Sciences and Hospital Sher-E-Bangla Nagar, Dhaka-1207, Bangladesh. Mobile: +880 1713 455 662, E-mail: [email protected]

MyPasTest: MRCS A Online - Jan Exam 2015 03. Anatomy; Aabdomen, Pelvis & Perineum (290Qs) ------peripheral parts of the diaphragm, but the phrenic nerves supply all the motor fibres. The diaphragm contracts during inspiration but relaxes during expiration and is displaced upwards by raised intra-abdominal pressure. The caval hiatus lies in the central tendon, whereas the oesophageal hiatus is surrounded by muscle fibres of the diaphragmatic crura.

The best answer is diaphragmatic paresis. This can cause shortness of breath and can be secondary to trauma, neurological disease, iatrogenic injury, malignancy and viral infection. A Page | tension pneumothorax is a surgical emergency and presents with shortness of breath with tracheal deviation and mediastinal shift away from the side in question. In this situation, clinical 224 diagnosis is important rather than a chest x-ray.

244. Concerning injuries to the bladder and urethra, which one of the following is true? Single best answer - select one answer only. A diagnostic peritoneal lavage should be performed beforecystography if an intraperitoneal bladder rupture is suspected« YOUR ANSWER A urethral catheter may be passed in the absence of pubic symphysis disruption and urethral haemorrhage« CORRECT ANSWER Bladder injuries are commonly caused by direct blows to the lower abdomen following urination Intravenous urography is the first line of investigation in patients with suspected bladder or urethral injuries following pelvic trauma Urgent CT scanning is indicated for intraperitoneal bladder rupture with peritonitis.

Bladder injuries can be caused by a direct blow to the lower abdomen, especially when the bladder is full or by penetration of a bone fragment in pelvic fractures. The first line of investigation in suspected bladder or urethral injuries is plain radiography to exclude pelvic fractures. Fracture of the pelvic bone or disruption of the pelvic symphysis indicates the possibility of associated bladder or urethral injury.

If an intraperitoneal bladder rupture is suspected then cystography should be done before diagnostic peritoneal lavage because a laparotomy will be required to repair such a lesion, if present, making lavage unnecessary. The signs of urethral injury caused by pelvic fracture are blood at the urethral meatus, perineal bruising and inability to pass urine. A urethral catheter must not be passed in such patients as it aggravates the injury and can also introduce infection. In general, a urethral catheter can be passed in a patient with a pelvic fracture if there is no blood at the meatus or if the pubic symphysis is not severely disrupted on plain radiography.

The safest way to treat urethral injuries caused by a pelvic fracture is to pass a suprapubic catheter either percutaneously or by cystotomy if the patient requires a laparotomy for other reasons or if the bladder is impalpable. Patients with intraperitoneal ruptures with peritonism are best treated by laparotomy and drainage by suprapubic catheter for about 7 days. Broad range antibiotics should be given. Extraperitoneal ruptures can be treated conservatively with appropriate drainage of the bladder.

------Dr Mohammed Shamsul Islam Khan, Medical Officer, Clinical Neuro-Surgery, National Institute of Neuro-Sciences and Hospital Sher-E-Bangla Nagar, Dhaka-1207, Bangladesh. Mobile: +880 1713 455 662, E-mail: [email protected]

MyPasTest: MRCS A Online - Jan Exam 2015 03. Anatomy; Aabdomen, Pelvis & Perineum (290Qs) ------245. A 69-year-old female presents with a firm mass on the posterior aspect of the anal canal. Biopsy confirms the presence of anal malignant melanoma and she proceeds to abdominoperineal resection. Which group of lymph nodes primarily drain the anal canal? Single best answer - select one answer only. External iliac lymph nodes« YOUR ANSWER Internal iliac lymph nodes Page | Mesorectal lymph nodes 225 Para-aortic lymph nodes Superficial inguinal lymph nodes« CORRECT ANSWER.

Above the pectinate line lymph drains initially to the internal iliac nodes. Below the pectinate line tissues drain to the superficial inguinal nodes.

246. A 25-year-old gentleman presents to the clinic with a swelling in the left hemiscrotum. Clinical examination reveals it to be smooth and located within the scrotum. The swelling is transilluminant and the testis is not palpable. What is the ideal management of this case? Single best answer - select one answer only. Needle aspiration« YOUR ANSWER Ultrasound scan and needle aspiration Ultrasound scan, beta HCG and alpha fetoprotein blood tests and if positive orchidectomy and/or chemotherapy Ultrasound scan, beta HCG and alpha fetoprotein blood tests and Jabolay’s repair Ultrasound scan, beta HCG and alpha fetoprotein blood tests and if positive further discussion in the multi-disciplinary team meeting« CORRECT ANSWER.

Hydrocele is abnormal accumulation of fluid within the tunica vaginalis. Secondary hydroceles may result from testicular injury, tumour, heart failure or obstruction of the retroperitoneal lymphatics. Congenital hydroceles communicating along the patent processus vaginalis with the hernia should be repaired for fear of strangulation.

Jaboulay procedure is excision and eversion of the hydrocoele sac and Lords procedure is plication of the sac. In this age group, a secondary hydrocoele caused by testicular malignancy is a real possibility. Non-seminomatous germ cell tumours are commoner in the age group of 20-30 years for which the tumour markers alpha fetoprotein and beta HCG are elevated.

247. A 21-year-old man presents to A&E with central abdominal pain radiating to the back. Serum amylase levels are significantly elevated and he is diagnosed with acute severe necrotising pancreatitis confirmed on a CT-scan. Which of the following best describes the anatomy of the pancreas? Gives attachment to the small bowel messentery« YOUR ANSWER Has an uncinate process lying anterior to the superior mesenteric vein Has the inferior mesenteric vein passing behind the neck Lies in the transpyloric plane« CORRECT ANSWER ------Dr Mohammed Shamsul Islam Khan, Medical Officer, Clinical Neuro-Surgery, National Institute of Neuro-Sciences and Hospital Sher-E-Bangla Nagar, Dhaka-1207, Bangladesh. Mobile: +880 1713 455 662, E-mail: [email protected]

MyPasTest: MRCS A Online - Jan Exam 2015 03. Anatomy; Aabdomen, Pelvis & Perineum (290Qs) ------Overlies the right kidney.

The head of the pancreas is related to the hilum but does not overlie the right kidney. It is, however, anterior to the left kidney. The transpyloric plane (L1) transects the pancreas obliquely, passing through the midpoint of the neck, with most of the head below the plane, and most of the body and tail above. Page |

The transverse mesocolon is attached to the head, neck and body of the pancreas. The uncinate 226 process lies posterior to the superior mesenteric vessels, and the inferior mesenteric vein passes behind the body of the pancreas, where it joins the splenic vein.

248. In a subcostal flank approach to the kidney, which of the following may be incised to increase upward mobility of the 12th rib? Single best answer question – choose ONE true option only. The latissimus dorsi muscle« YOUR ANSWER The lumbodorsal fascia The quadratus lumborum muscle The costovertebral ligament« CORRECT ANSWER The intercoastal muscles between ribs 11 and 12.

The costovertebral ligament is a strong fascial attachment between the transverse process of the first and second lumbar vertebrae and the inferior margin of the 12th rib. It is encountered only in posterior approaches to the kidney and it can be incised to produce a greater degree of mobility of the 12th rib thus providing greater exposure and access to the structures which reside within the upper portion of the retroperitoneum.

249. Which of the following is correct concerning the inguinal canal? Single best answer - select one answer only. In females the inguinal canal transmits only the round ligament of the uterus« YOUR ANSWER The deep inguinal ring is an hiatus in the external oblique aponeurosis The inferior epigastric vessels are found lateral to the deep inguinal ring The ilioinguinal nerve is found within the spermatic cord The posterior wall is formed by the conjoint tendon and the transversalis fascia« CORRECT ANSWER.

The inguinal canal is a short oblique passage through the anterior abdominal wall. The relatively weak transversalis fascia and the conjoint tendon form the posterior wall. The deep ring is an hiatus in the transversalis fascia, with the inferior epigastric vessels lying immediately medial to it. The superficial inguinal ring is a triangular hiatus in the external oblique aponeurosis. In females the inguinal canal carries the round ligament of the uterus and the ilioinguinal nerve. In males, the ilioinguinal nerve is carried in the inguinal canal but is not found within the spermatic cord.

------Dr Mohammed Shamsul Islam Khan, Medical Officer, Clinical Neuro-Surgery, National Institute of Neuro-Sciences and Hospital Sher-E-Bangla Nagar, Dhaka-1207, Bangladesh. Mobile: +880 1713 455 662, E-mail: [email protected]

MyPasTest: MRCS A Online - Jan Exam 2015 03. Anatomy; Aabdomen, Pelvis & Perineum (290Qs) ------250. As part of the procedure for mobilising the splenic flexure of the colon, the lesser sac needs to be entered. What is the most accurate anatomical description of the lesser sac? Single best answer - select one answer only. Its left lateral border is formed by the falciform ligament« YOUR ANSWER Its posterior relations include the kidney Page | Its superior relation is the posterior aspect of the stomach It communicates with the general peritoneal cavity via the epiploic foramen« CORRECT ANSWER 227 The posterior wall is formed partly by the lesser omentum.

The lesser sac is a peritoneal cavity located behind the stomach. It may be entered by passing a finger into the epiploic foramen, immediately behind the portal vein in the free edge of the lesser omentum. The greater omentum and the pancreas form its posterior wall. The gastrosplenic and lienorenal ligaments form its left lateral border. The falciform ligament arises from the anterior abdominal wall and demarcates the left and the right lobe of the liver.

251. A 64-year-old, previously healthy male presents with a palpable anorectal mass and rectal bleeding. On endoscopic examination, you notice a large mass 1cm proximal to the dentate line, which was 3cm from the anal verge. Following staging investigations, the MDT elect to offer curative surgery. Which is the most appropriate procedure? Abdominoperineal resection« YOUR ANSWER (Correct) Hartmann’s procedure Low anterior resection Rectosigmoid colectomy Transanal endoscopic mucosal resection.

Abdominoperineal resection is the most appropriate surgical treatment of anorectal cancers that lie within 5cm of the anal verge.

(http://training.seer.cancer.gov/colorectal/treatment/rectum.html). Hartmann‟s procedure is resection of the sigmoid colon, colostomy, and oversewing of the distal rectal stump. Transanal endoscopic mucosal resection can be used to excise superficial rectal polyps or small neoplasms.

252. A 23-year-old male presents with hypovolaemic shock following penetrating abdominal trauma. He undergoes emergency laparotomy and is noted to have a liver laceration. The surgeon clamps the hepatoduodenal ligament just anterior to the epiploic foramen. What structure lies posterior to the epiploic foramen? Abdominal aorta« YOUR ANSWER Common bile duct First part of the duodenum Inferior vena cava« CORRECT ANSWER Left lobe of the liver.

------Dr Mohammed Shamsul Islam Khan, Medical Officer, Clinical Neuro-Surgery, National Institute of Neuro-Sciences and Hospital Sher-E-Bangla Nagar, Dhaka-1207, Bangladesh. Mobile: +880 1713 455 662, E-mail: [email protected]

MyPasTest: MRCS A Online - Jan Exam 2015 03. Anatomy; Aabdomen, Pelvis & Perineum (290Qs) ------Pringle‟s Manoeuvre (clamping or grapsing of the hepatoduodenal ligament) is performed to achieve rapid haemostasis in cases of severe hepatic trauma. The ligament contains the common bile duct, the hepatic portal vein and the hepatic artery proper. It lies anterior to the epiploic foramen which itself lies anterior to the inferior vena cava.

253. You are discussing open pelvic fractures with a colleague. What is the mortality associated with these fractures? Single best answer - select one answer only. Page | 20%« YOUR ANSWER 228 30% 40% 50%« CORRECT ANSWER 60%.

Pelvic fractures are a major cause of morbidity with 10–25% mortality. In association with rectal lacerations, pelvic fractures are classified as compound with a resulting increased mortality. Open pelvic fractures are associated with a mortality of 50%.

254. One boundary of the (epiploic foramen or foramen of Winslow) is: Single best answer question – choose ONE true option only. Anteriorly the hepatic artery« YOUR ANSWER (Correct) Posteriorly the hepatic vein Superiorly the quadrate lobe of the liver Inferiorly the hepatic vein None of the above.

The boundaries of the omental foramen are: anteriorly, the portal vein, hepatic artery, and bile duct (all in the free edge of the lesser omentum); posteriorly, the inferior vena cava and right crus of the diaphragm; superiorly, the caudate lobe of the liver; and inferiorly, the superior part of the duodenum, portal vein, hepatic artery, and bile duct.

255. A 17-year-old male presents with right iliac fossa pain, fever and vomiting. He is assumed to have acute appendicitis, but during laparotomy he is noted to have an inflamed diverticulum on the antimesenteric border of the distal ileum. Meckel‟s diverticulum is a remnant of which embryological sturcture? Single best answer - select one answer only. Cloaca« YOUR ANSWER Mullerian duct Urachus Vitelline duct« CORRECT ANSWER Wolffian duct.

Meckel‟s diverticulum occurs in 2% of the population. Inflammation of the diverticulum can mimic the signs and symptoms of acute appendicitis, so examination of the ileum should always be undertaken if the appendix appears normal at the subsequent laparotomy. It is a ------Dr Mohammed Shamsul Islam Khan, Medical Officer, Clinical Neuro-Surgery, National Institute of Neuro-Sciences and Hospital Sher-E-Bangla Nagar, Dhaka-1207, Bangladesh. Mobile: +880 1713 455 662, E-mail: [email protected]

MyPasTest: MRCS A Online - Jan Exam 2015 03. Anatomy; Aabdomen, Pelvis & Perineum (290Qs) ------remnant of the vitelline duct which joins the yolk sac and midgut lumen of the developing foetus.

256. The coeliac branch of the posterior vagal trunk sustained iatrogenic injury during repair of a hiatus hernia. The damage to this nerve would affect the muscular movements, as well as some secretory activities, of the gastrointestinal tract. Which segment is LEAST likely to be affected by the nerve damage? Single best answer question – choose ONE true option only. Page | Ascending colon« YOUR ANSWER 229 Caecum Jejunum Ileum Sigmoid colon« CORRECT ANSWER.

The vagus nerve supplies parasympathetic fibres to all of the abdominal organs, which receive blood from the coeliac trunk or superior mesenteric artery. This means that the vagus supplies parasympathetics to the entire gastrointestinal tract, up to the last part of the transverse colon. The end of the transverse colon and all gastrointestinal structures distal to that point receive parasympathetic innervation from the pelvic splanchnic nerves and blood from the inferior mesenteric artery. So, the ascending colon, caecum, jejunum and ileum would all be affected by damage to the vagus nerve. The sigmoid colon, which receives parasympathetic innervation from the pelvic splanchnics, would not be affected.

257. A 40-year-old man is involved in a road traffic accident. He complains of having haematuria and clinically is tachycardic and hypotensive. A CT scan of the abdomen reveals a ruptured left kidney with retroperitoneal haemorrhage. He requires an emergency nephrectomy. You call the consultant who asks you about the anatomical relations and approach for nephrectomy. How would you respond? Single best answer - select one answer only. It is best approached through the 12th rib and simple nephrectomy is indicated in cases of non- functioning kidneys« YOUR ANSWER The adrenal gland lies under the medial border below the hilum and radical nephrectomy is indicated in cases of non-functioning kidneys The adrenal gland lies over the medial border above the hilum and simple nephrectomy is indicated in cases of non-functioning kidneys« CORRECT ANSWER The lower pole is separated from the peritoneum by the tail of the pancreas The renal artery which lies in front of the renal vein should be ligated.

The best approach for a ruptured kidney is through the anterior abdominal wall by a midline incision. The left kidney is related to the 12th rib posteriorly. The upper pole of the left kidney may overlie the 11th rib on X-ray. It overlies the iliohypogastric and ilioinguinal nerves. The renal artery lies posterior to the renal vein.

The tail of the pancreas separates the hilum of the left kidney from the peritoneum. Radical nephrectomy is indicated for localized renal carcinoma and involves removal of the kidney with surrounding Gerota‟s fascia. Simple nephrectomy is indicated in cases of non-functioning kidneys.

------Dr Mohammed Shamsul Islam Khan, Medical Officer, Clinical Neuro-Surgery, National Institute of Neuro-Sciences and Hospital Sher-E-Bangla Nagar, Dhaka-1207, Bangladesh. Mobile: +880 1713 455 662, E-mail: [email protected]

MyPasTest: MRCS A Online - Jan Exam 2015 03. Anatomy; Aabdomen, Pelvis & Perineum (290Qs) ------

258. The aorta passes into the abdomen at the level of the twelfth thoracic vertebra. Which structure accompanies it through the diaphragm? Single best answer - select one answer only. Anterior and posterior vagal trunks« YOUR ANSWER Branches of the right phrenic nerve Page | Left phrenic nerve Oesophagus 230 Thoracic duct« CORRECT ANSWER.

The diaphragm has three major openings. The IVC and branches of the right phrenic nerve pass through the caval opening at the level of T8 vertebra. The oesophagus, anterior and posterior vagal trunks and the oesophageal arteries pass through the oesophageal hiatus at the level of the T10 vertebra. The aorta, azygous vein and thoracic duct pass through the aortic hiatus at the level of the T12 vertebra. The left phrenic nerve pierces the left hemidiaphragm.

259. You are called to the delivery room with your registrar to see a baby because the midwife says he has “no abdominal wall”. When you get there you find a baby with an abdominal defect with loops of bowel outside the abdomen, and no covering layers. Which one of the following statements is true? Single best answer question – choose ONE true option only The child has an umbilical hernia« YOUR ANSWER The child has gastroschisis« CORRECT ANSWER The child has an exomphalos The mortality rate of this condition is 50% The condition is often associated with other congenital anomalies.

An umbilical hernia is covered by skin. The defect is herniation of the abdominal contents through the umbilical fibromuscular ring. Umbilical hernias usually occur in infants and reach their maximal size by the first month of life. Most hernias of this type close spontaneously by the first year of life, with only a 2-10% incidence in children older than 1 year. An exomphalos always has a sac – which may be intact or ruptured.

The sac has three layers Peritoneum, Wharton‟s jelly, and amnion. The sac may contain intestinal loops, bladder, and liver. Unlike gastroschisis it is often associated with other congenital anomalies. Gastroschisis has no sac, and is rarely associated with congenital defects, but it may be associated with intestinal atresia. Management initially consists of covering the defect with Clingfilm to prevent heat and moisture loss, while establishing IV access for IV fluids.

260. A 78-year-old male presents with difficulty of micturition. Prostate specific antigen levels are significantly raised and on digital rectal examination he is noted to have a hard craggy prostate gland. In which histological portion of the prostate gland is adenocarcinoma most likely to develop? Single best answer - select one answer only. Anterior fibromuscular zone« YOUR ANSWER ------Dr Mohammed Shamsul Islam Khan, Medical Officer, Clinical Neuro-Surgery, National Institute of Neuro-Sciences and Hospital Sher-E-Bangla Nagar, Dhaka-1207, Bangladesh. Mobile: +880 1713 455 662, E-mail: [email protected]

MyPasTest: MRCS A Online - Jan Exam 2015 03. Anatomy; Aabdomen, Pelvis & Perineum (290Qs) ------Central zone Peripheral zone« CORRECT ANSWER Transitional zone Urogenital sinus. Page | 80-90% of all prostatic adenocarcinomas occur in the peripheral zone, with 10-20% in the transitional zone and 2.5% in the central zone. The urogenital sinus is the embryological 231 precursor of the gland.

261. A 45-year-old male presents with a swelling in the right groin. He is diagnosed with an inguinal hernia and proceeds to surgery. During the procedure, you notice a direct hernia through the posterior wall of the inguinal canal. Which structure lies in the medial one-third of the posterior wall of the inguinal canal? Aponeurosis of external oblique« YOUR ANSWER Conjoint tendon« CORRECT ANSWER Lacunar ligament Superficial ring Transversus abdominis.

The conjoint tendon is formed by the confluence of tissues from the internal oblique and transverses abdominis muscles of the abdominal wall. It lies in the medial one-third of the inguinal canal, posterior to the superficial ring. The lateral one-third of the posterior wall is formed of the deep inguinal ring. The lacunar ligament forms the medial aspect of the floor of the canal, whereas the aponeurosis of the external oblique forms the anterior wall. Transversus abdominis muscle forms the roof.

262. The infrarenal abdominal aorta is the commonest site for abdominal aortic aneurysm. Ruptured abdominal aortic aneurysm (AAA) is the 13th commonest cause of death in the western world. What is the most common cause of AAA? Single best answer - select one answer only. Atherosclerosis« YOUR ANSWER (Correct) Diabetes mellitus Inflammatory Marfan syndrome Syphilis.

Abdominal aortic aneurysms (AAAs) are caused by atherosclerosis in most cases. There is a higher incidence of artherosclerosis in diabetics. Connective tissue disorders including Marfan, inflammatory aneurysms and infective causes such as syphilis are amongst the rare causes.

263. Prior to performing a hernia repair your consultant is quizzing you regarding the inguinal region and asks for the alternative names for the anatomical structures. Which one of the following is correctly paired?

------Dr Mohammed Shamsul Islam Khan, Medical Officer, Clinical Neuro-Surgery, National Institute of Neuro-Sciences and Hospital Sher-E-Bangla Nagar, Dhaka-1207, Bangladesh. Mobile: +880 1713 455 662, E-mail: [email protected]

MyPasTest: MRCS A Online - Jan Exam 2015 03. Anatomy; Aabdomen, Pelvis & Perineum (290Qs) ------The femoral canal is also known as Alcock’s canal« YOUR ANSWER The inguinal ligament is also known as Astley Cooper’s ligament The inguinal ligament is also known as Poupart’s ligament« CORRECT ANSWER The transversalis fascia is also known as Scarpa’s fascia Page | The vas deferens is also known as the duct of Santorini. 232 The inguinal ligament is also known as Poupart‟s ligament. The pectineal ligament is also known as Astley Cooper‟s ligament, and should not be confused with the suspensory ligaments of the breast, also known as the ligaments of Cooper. The duct of Santorini is the accessory pancreatic duct. The transversalis fascia and Scarpa‟s fascia are not related structures. Alcock‟s canal is a fascial tunnel on the lateral wall of the ischiorectal fossa, which conveys the pudendal nerve and vessels.

264. On your post-operative ward round you discover that one of your patients has third-space fluid loss following a cholecystectomy. How would you describe 3rd space losses? Can be up to 30 ml/kg/hour« YOUR ANSWER Has a fluid composition similar to interstitial fluid« CORRECT ANSWER Includes losses from enterocutaneous fistulae Is best replaced by dextrose-saline Is unrelated to the degree of tissue injury.

Third-space loss is the temporary internal loss of extracellular fluid into a space that does not participate in the normal transport of nutrients or waste products. In moderately major operations, such as cholecystectomy, such a loss would be approximately 3 ml/kg/hour. In more extensive operations, such as aortic aneurysm repair, third-space fluid loss could initially be 10–20 ml/kg/hour.

Dextrose-saline is a hypotonic solution, so when the dextrose is metabolised it increases the free-water content and so leads to hyponatraemia. Balanced salt solutions should be used to replace thirdspace losses. Transcellular losses that might be measured include ascites, pleural effusions and intraintestinal losses.

265. Complications of pelvic fractures include all of the following, except: Single best answer - select one answer only. Extraperitoneal urinary extravasation« YOUR ANSWER Fat embolus Madura foot« CORRECT ANSWER Rectal perforation Rupture of the internal iliac vein.

Pelvic fractures are commonly associated with bladder and urethral injuries. Damage to iliac arteries and veins can precipitate fatal hypovolaemic shock. Bladder injuries may be extra- or

------Dr Mohammed Shamsul Islam Khan, Medical Officer, Clinical Neuro-Surgery, National Institute of Neuro-Sciences and Hospital Sher-E-Bangla Nagar, Dhaka-1207, Bangladesh. Mobile: +880 1713 455 662, E-mail: [email protected]

MyPasTest: MRCS A Online - Jan Exam 2015 03. Anatomy; Aabdomen, Pelvis & Perineum (290Qs) ------intraperitoneal, depending on the mechanism of injury and state of bladder filling at the time. Madura foot is due to mycetoma pedis, a chronic fungal infection.

266. An 80-year-old gentleman is referred by his general practitioner with symptoms of frequency, nocturia, hesitancy and terminal dribbling. Clinical examination reveals a smooth bilobed enlarged prostate suggestive of benign prostatic hyperplasia. The serum Prostatic Specific Antigen level is 3. Which one of the following statements is true regarding the best Page | management option for this patient with benign prostatic hyperplasia? Single best answer - 233 select one answer only. Observation with 6 monthly follow-up« YOUR ANSWER Transurethral resection Ultrasound-guided core biopsy Diuretics 5-alpha reductase inhibitors or alpha-1 adrenergic antagonists« CORRECT ANSWER.

The prevalence of benign prostatic hyperplasia is nil in men younger than 30 years, 50% in the 51-60 years age range and over 85% in men in their eighties. Symptom severity poorly correlates with objective investigations such as urinary flow rate, prostatic volume, post-void residual bladder volume and urodynamically assessed outflow tract obstruction.

Proliferation of the transitional zone is seen with lateral lobe enlargement. Median lobe enlargement is due to periurethral stromal proliferation. Benign prostatic hyperplasia consists of 50% connective tissue, 25% smooth muscle and 25% epithelium. The risk of acute urinary retention is 1-2% per year and treatment consists of observation, use of drugs such as 5-alpha reductase inhibitors, alpha-1 receptor antagonists and surgery: transurethral resection or prostactectomy.

267. A 64-year-old male presents with a penile swelling and pain. He is diagnosed with a penile squamous cell carcinoma and he undergoes staging with discussion in the Urology multi- disciplinary team meeting. A pelvic MRI shows lymph nodes to be enlarged (2 cm). Which lymph nodes are likely to be involved? External iliac lymph nodes« YOUR ANSWER Internal iliac lymph nodes Para-aortic lymph nodes Sacral lymph nodes Superficial inguinal lymph nodes« CORRECT ANSWER.

The external genitalia drain to the superficial inguinal lymph nodes, with the exception of the testes, which reveal their intra-abdominal embryological origin by draining to the para-aortic lymph nodes.

268. A 58-year-old female presents with a large, pulsatile abdominal mass, hypotension and back pain. She undergoes emergency endovascular aneurysm repair. At which vertebral level does the aorta bifurcate to give the common iliac arteries? Single best answer - select one answer only. ------Dr Mohammed Shamsul Islam Khan, Medical Officer, Clinical Neuro-Surgery, National Institute of Neuro-Sciences and Hospital Sher-E-Bangla Nagar, Dhaka-1207, Bangladesh. Mobile: +880 1713 455 662, E-mail: [email protected]

MyPasTest: MRCS A Online - Jan Exam 2015 03. Anatomy; Aabdomen, Pelvis & Perineum (290Qs) ------L1« YOUR ANSWER L2 L3 L4« CORRECT ANSWER Page | L5. 234 The aorta enters the abdomen under the median arcuate ligament at T12 and divides into the common iliac arteries at L4 (intercristal plane). In its course through the abdomen, the aorta gives off the three single ventral gut arteries (coeliac, superior and inferior mesenteric) and passes under the left renal vein. Unless the patient is obese, abdominal aortic pulsations are normally palpable.

269. A 28-year-old man presents with severe right sided abdominal pain together with peritonism in the right iliac fossa. His clinical picture and haematological profile are consistent with appendicitis but at operation an inflamed Meckel‟s diverticulum is identified. Which of the following statements is true regarding Meckel‟s diverticulum? Single best answer - select one answer only. It is a partial persistence of the median umbilical ligament« YOUR ANSWER It arises on the mesenteric border of the small bowel It usually lies within 100 cm of the ileocaecal valve« CORRECT ANSWER It is present in 0.2% of individuals It contains heterotopic pancreatic tissue in 20% of cases.

A Meckel‟s diverticulum is a partial persistence of the vitellointestinal duct and there may be a fibrous connection to the umbilicus. The median umbilical ligament or „Urachus‟ is a connection between the umbilicus and the bladder. Meckel‟s diverticulum arises on the antimesenteric border of the small bowel usually within 100 cm of the ileocaecal valve. It is present in 2% of individuals; 10% contain heterotopic gastric mucosa and very rarely heterotopic pancreatic mucosa. They may present with intussusception, bleeding, perforation, or inflammation.

270. A 21-year-old woman develops a perinephric abscess following an episode of severe pyelonephritis. Which of the following structures is not likely to be in contact with the purulent exudates? cisterna chyli « YOUR ANSWER (Correct) costodiaphragmatic angle of the pleural cavity ilioinguinal nerve psoas muscle subcostal nerve.

The kidneys lie retroperitoneally in the upper posterior abdominal wall. They are mostly covered by the costal margin. Both kidneys are in contact posteriorly with the costodiaphragmatic

------Dr Mohammed Shamsul Islam Khan, Medical Officer, Clinical Neuro-Surgery, National Institute of Neuro-Sciences and Hospital Sher-E-Bangla Nagar, Dhaka-1207, Bangladesh. Mobile: +880 1713 455 662, E-mail: [email protected]

MyPasTest: MRCS A Online - Jan Exam 2015 03. Anatomy; Aabdomen, Pelvis & Perineum (290Qs) ------recess, psoas muscle, subcostal and ilioinguinal nerves. The cisterna chyli is found on the right of the aorta in front of L1 and L2. It is at a similar level to the kidneys and lies between them without direct contact.

271. You are called to a patient involved in an RTA and asked to insert a urethral catheter. You are cautious, suspecting a possible urethral injury and run through the possible indicative signs Page | in your head. Which of the following would not suggest urethral injury? 235 Battle’s sign « YOUR ANSWER (Correct) Blood at the external urethral meatus ‘Butterfly’ haematoma Haematuria High riding prostate.

The inability to void, an unstable pelvic fracture, blood at the external urethral meatus, a ‘butterfly’ haematoma, or a high riding prostate on digital rectal examination (DRE) are indications for the surgeon to request a retrograde urethrogram to confirm that the urethra is intact prior to inserting a urethral catheter. In the case of a disrupted urethra a suprapubic catheter should be inserted.

272. Which of the following is correct regarding the spleen? Is a retroperitoneal structure« YOUR ANSWER May be absent and replaced with multiple splenunculi« CORRECT ANSWER May be injured by a stab wound over the left eigth rib The splenic artery passes to the splenic hilum in the gastrosplenic ligament Venous drainage from the spleen passes directly to the inferior vena cava.

The spleen lies in the concavity of the left hemidiaphragm with its long axis lying along the tenth rib, extending between ribs nine and eleven. It is fully invested in peritoneum, making it a peritoneal structure. The arterial supply is the splenic artery, which reaches the spleen via the lienorenal ligament. Venous drainage is via the splenic vein into the portal vein. Occasionally the spleen may be replaced with multiple splenunculi.

273. A 56-year-old lady has been referred to your surgical clinic with symptoms of waterbrash and a new onset night cough. She has tried maximal medical management with her GP and a diagnosis of reflux oesophagitis has been made on OGD. Which of the following statements is true regarding this condition? Single best answer - select one answer only. Does not require more than alteration of lifestyle to treat« YOUR ANSWER If untreated may cause stricturing of the oesophagus« CORRECT ANSWER Is always present with hiatus hernia Is preferably treated with surgery Is treated surgically principally by attempting to narrow the gastro-oesophageal junction.

------Dr Mohammed Shamsul Islam Khan, Medical Officer, Clinical Neuro-Surgery, National Institute of Neuro-Sciences and Hospital Sher-E-Bangla Nagar, Dhaka-1207, Bangladesh. Mobile: +880 1713 455 662, E-mail: [email protected]

MyPasTest: MRCS A Online - Jan Exam 2015 03. Anatomy; Aabdomen, Pelvis & Perineum (290Qs) ------The mainstay of treatment for symptomatic reflux oesophagitis is with acid suppression therapy. If untreated, stricturing is common. Modern surgical management is with a Nissen‟s fundoplication where the fundus is wrapped around the distal oesophagus with plication of the oesophageal hiatus. The GOJ itself is not touched. The vast majority of hiatus herniae are asymptomatic.

274. A 78-year-old lady presents to the emergency department 3 years following a Hartmaans Page | procedure for perforated diverticular disease. She has never wanted a reversal of her stoma but 236 has recently noticed a tender lump adjacent to the stoma. A diagnosis of a parastomal hernia has been made. Which of the following statements best reflects this condition? Single best answer - select one answer only. Is usually asymptomatic« YOUR ANSWER (Correct) Is more common when the stoma is brought through the rectus muscle Occurs in 30% of colostomies Should always be repaired Should be repaired using a mesh.

Parastomal herniae occur in 10% of colostomies, are usually symptomless and require no treatment. Sepsis seems to play little part. It is usually a technical error of making the opening too big and not bringing it out through the rectus muscle which acts as a strong contractile barrier to herniation. Mesh repair can be undertaken, but the presence of foreign material around a stoma site leads to a high risk of infection, so is rarely carried out.

275. During a left radical nephrectomy performed via a transabdominal route, excessive traction on which of the following structures might be expected to produce a significant injury to the spleen? Single best answer question – choose ONE true option only. Left adrenal gland« YOUR ANSWER Splenorenal and splenocolic ligaments« CORRECT ANSWER Splenorenal ligament Splenocolic ligament None of the above.

There is typically a peritoneal extension running from the perirenal fascia which covers the upper pole of the left kidney to the inferior splenic capsule. This peritoneal extension is known as the splenorenal or lienorenal ligament. Similarly, the splenocolic ligament attaches the base of the hilum to the left transverse mesocolon and to the splenic flexure, on the colonic surface of the spleen. Just as with the splenocolic ligament one must take care not to exert undue tension on the splenorenal ligament during a left radical nephrectomy, and thus avoid any inadvertent tearing of the spleen.

276. The femoral sheath: Single best answer question – choose ONE true option only

------Dr Mohammed Shamsul Islam Khan, Medical Officer, Clinical Neuro-Surgery, National Institute of Neuro-Sciences and Hospital Sher-E-Bangla Nagar, Dhaka-1207, Bangladesh. Mobile: +880 1713 455 662, E-mail: [email protected]

MyPasTest: MRCS A Online - Jan Exam 2015 03. Anatomy; Aabdomen, Pelvis & Perineum (290Qs) ------Surrounds the femoral canal and encloses the femoral nerve« YOUR ANSWER The medial wall is pierced by the great saphenous vein« CORRECT ANSWER Is divided by two ventral septa into two compartments The lateral compartment contains the femoral vein Page | The femoral sheath ends about 1 cm inferior to the inguinal ligament. 237 The femoral sheath is an oval, funnel shaped fascial tube which encloses the proximal parts of the femoral vessels, which inferior to the inguinal ligament. It also surrounds the femoral canal, but it does not enclose the femoral nerve. The femoral sheath ends about 4 cm inferior to the inguinal ligament. The medial wall of the femoral sheath is pierced by the great saphenous vein and lymphatic vessels. The sheath is divided by two vertical septa into three compartments: i. lateral (containing the femoral artery), ii. intermediate (containing the femoral vein, and iii. medial compartment or space called the femoral canal.

277. A 63-year-old obese man has been referred to your clinic with symptoms of acid reflux, upper abdominal pain and a new onset nocturnal cough. He has tried maximum dose proton pump inhibitor treatment for 6 months with little relief. Which of the following statements seems the most appropriate? Single best answer - select one answer only. The oesophagus is a muscular tube approximately 25 cm long comprised of an inner longitudinal muscle and an outer circular muscle coat which overlap in the middle part.« YOUR ANSWER The lower oesophageal sphincter is a distinct anatomical condensation of muscle which together with the angle of His acts to present reflux of gastric acid back into the distal oesophagus. The best way of investigating gastro-oesophageal reflux disease is ambulatory pH monitoring where a probe is placed within the oesophagus itself and provides a continuous reading of the level of oesophageal acidity.« CORRECT ANSWER Barrett’s oesophagus is diagnosed on the anatomical endoscopic appearance of the Z line. There are 3 predictable sites of oesophageal constriction which relate to the cricopharyngeal sphincter (15cm from incisors), where it is crossed by the aortic arch (22cm) and where it passes through the diaphragm (38cm).

24 hour ambulatory monitoring was introduced in 1969 and has become the best way for monitoring the pattern of exposure of acid in the oesophagus, and the gold standard for gastro- oesophageal reflux disease. While excellent at diagnosing this, Barrett‟s oesophagus needs to be diagnosed by endoscopy, with multiple biopsies – it cannot be diagnosed from pH monitoring or the position of the Z line alone.

The monitor is placed so that the pH probe lies 5 cm above the oesophageal sphincter, therefore being able to record the presence of acid within the oesophagus itself. The lower oesophageal sphincter is a physiological rather than an anatomical sphincter and there is no distinct condensation of muscle here.

For an in-depth description please see http://www.ncbi.nlm.nih.gov/pmc/articles/PMC1774230/.

------Dr Mohammed Shamsul Islam Khan, Medical Officer, Clinical Neuro-Surgery, National Institute of Neuro-Sciences and Hospital Sher-E-Bangla Nagar, Dhaka-1207, Bangladesh. Mobile: +880 1713 455 662, E-mail: [email protected]

MyPasTest: MRCS A Online - Jan Exam 2015 03. Anatomy; Aabdomen, Pelvis & Perineum (290Qs) ------

There is an inner circular muscle coat and an outer longitudinal coat which are skeletal in the upper part and smooth in the lower part with considerable overlap in the middle. In addition to the 3 levels of constriction described there is a fourth which lies at 27cm from the incisors and corresponds to the left principle bronchus. Page | 278. A 72-year-old man is diagnosed with an adenocarcinoma of the head of pancreas. He undergoes a Whipple‟s procedure. Which of the following best describes the third part of the 238 duodenum? Is anterior to the right ureter« YOUR ANSWER Is anterior to the inferior mesenteric artery« CORRECT ANSWER Is anterior to the root of the mesentery of the small bowel Is anterior to the superior mesenteric vessels Is immediately anterior to the right renal artery. The third part of the duodenum lies in the subcostal plane (L3) and follows the inferior margin of the pancreatic head. The third part of the duodenum lies posterior to the superior mesenteric vessels. It overlies the aorta and the beginning of the inferior mesenteric artery. The root of the small bowel mesentery is attached near its termination on the left. The second part of the duodenum overlies the right ureter and renal vessels in the hilum of the right kidney, separated from the renal artery by the vein.

279. A 23-year-old female presents with right iliac fossa pain and hypovolaemic shock. Beta human chorionic gonadotrophin levels are significantly raised and she is diagnosed with an ectopic pregnancy. Where is an ectopic implantation most likely to occur? Single best answer - select one answer only. Ampulla of uterine tube« YOUR ANSWER (Correct) Infundibulum of uterine tube Intramural portion of uterine tube Isthmus of uterine tube Rectouterine pouch.

The uterine (or Fallopian) tube begins at the infundibulum and progresses medially to include the ampulla, the isthmus and the intramural portion. Ectopic implantation is possible in all of the sites mentioned above but approximately 80% occur in the ampulla of the uterine tube.

280. A 57-year-old female presents with large bowel obstruction and is diagnosed with an ascending colon adenocarcinoma. During the subsequent right hemicolectomy you are advised not to injure a layer of fascia that lies posterior to the ascending colon. What fascia is this? Single best answer - select one answer only. Colles’ fascia« YOUR ANSWER Denonvillier’s fascia Gerota’s fascia« CORRECT ANSWER

------Dr Mohammed Shamsul Islam Khan, Medical Officer, Clinical Neuro-Surgery, National Institute of Neuro-Sciences and Hospital Sher-E-Bangla Nagar, Dhaka-1207, Bangladesh. Mobile: +880 1713 455 662, E-mail: [email protected]

MyPasTest: MRCS A Online - Jan Exam 2015 03. Anatomy; Aabdomen, Pelvis & Perineum (290Qs) ------Scarpa’s fascia Spigelian fasci.

Gerota‟s fascia is a layer of connective tissue that encapsulates the kidney and the adrenal gland. Colles‟ fascia affixes the root of the penis, Denonvillier‟s fascia separates the rectum from the prostate gland, and the Scarpa and Spigelian fascia are found in the anterior abdominal Page | wall. 239 281. A 78-year-old female presents with obstructive jaundice, weight loss and general malaise. Computerised tomography of the abdomen reveals a mass in the head of the pancreas, involving the common bile duct. Into which part of the gastrointestinal tract do the fluids produced by the pancreas drain? First part of the duodenum« YOUR ANSWER First part of the jejunum Fourth part of the duodenum Second part of the duodenum« CORRECT ANSWER Third part of the duodenum.

The hepatopancreatic ampulla enters the second part of the duodenum at the major duodenal papilla. It is through this papilla that endoscopic retrograde cholangiopancreatography can enable the relief of obstructive jaundice via the placement of a biliary stent.

282. Which of the following best describes the rectum? Arises as a continuation of the sigmoid colon at the first sacral vertebra« YOUR ANSWER Has a well-developed mesentery; the mesorectum Has lymph drainage to the internal iliac and pre-aortic nodes« CORRECT ANSWER Is supplied by the middle and inferior rectal arteries, but not the superior Is covered by peritoneum along its entire length.

The rectum arises as a continuation of the sigmoid colon at the level of the third sacral vertebra. The blood supply to the rectum is predominantly from the superior rectal artery, with variable contributions from the middle and inferior rectal vessels. The inferior rectal vessels may often provide a significant blood supply to the rectum.

The peritoneum covers two-thirds of the anterior surface and one third of the lateral surface. There is no rectal mesentery – the mesorectum is a clinical term for the visceral pelvic fascia that contains pararectal lymph nodes. Lymph drainage continues from there to the internal iliac and pre aortic nodes.

283. In a patient with gastric carcinoma, the radiologist reported that the lymph nodes around the coeliac trunk are enlarged. The coeliac trunk: Single best answer question – choose ONE true option only. Is the arterial supply to the embryonic mid-gut« YOUR ANSWER

------Dr Mohammed Shamsul Islam Khan, Medical Officer, Clinical Neuro-Surgery, National Institute of Neuro-Sciences and Hospital Sher-E-Bangla Nagar, Dhaka-1207, Bangladesh. Mobile: +880 1713 455 662, E-mail: [email protected]

MyPasTest: MRCS A Online - Jan Exam 2015 03. Anatomy; Aabdomen, Pelvis & Perineum (290Qs) ------Is accompanied by the coeliac vein Arises from the lateral surface of the abdominal aorta, at about the level of the first lumbar vertebra Gives rise to the splenic, left gastric and common hepatic arteries« CORRECT ANSWER Is surrounded by the aorticorenal ganglion. Page | The coeliac trunk is a short thick trunk, about 1.25 cm in length, which arises from the front of the aorta, just below the aortic hiatus of the diaphragm and, passing nearly horizontally forward, 240 divides into three large branches, the left gastric, the hepatic and the splenic. It occasionally gives off one of the inferior phrenic arteries. It is covered by the lesser omentum. On the right side it is in relation to the right coeliac ganglion and the caudate process of the liver; on the left side, to the left coeliac ganglion and the cardiac end of the stomach. Below, it is in relation to the upper border of the pancreas and the splenic vein.

284. You are assisting in a nephrectomy using a posterior approach. Which of the following is important during this approach? Single best answer - Select ONE correct answer. The adrenal glands are at easy risk of injury« YOUR ANSWER The parietal pleura does not extend below the twelfth rib The perinephric fat surrounds the renal fascia The renal artery lies anterior to the renal vein The subcostal neurovascular bundle is relatively protected from risk of injury« CORRECT ANSWER.

The lower border of the parietal pleura crosses the twelfth rib at the lateral border of the erector spinae, and passes in horizontally to the lower border of the twelfth thoracic vertebrae. There is, therefore, a triangle of pleura in the costoverterbral angle below the medial part of the twelfth rib, behind the upper pole of the kidney. The subcostal neurovascular bundle is relatively protected from risk of injury because it lies in the costal groove. The renal fascia of Gerota surrounds the perinephric fat, which lies outside the renal capsule. The renal arteries lie behind the renal veins. The adrenal glands are anatomically well protected.

They lie anterosuperior to the upper part of each kidney, within their own compartment of the renal fascia. Care must be taken during a nephrectomy to avoid damage to the adrenals, but these are not easily damaged by a careful surgeon using a posterior approach.

285. An elderly gentleman presents with progressive dysphagia and weight loss. He is a lifetime heavy smoker and his GP is concerned. Which of the following statements best represents his diagnosis of oesophageal carcinoma? Single best answer - select one answer only. It has wide geographical variation in incidence rates and is increasing in frequency in the UK« YOUR ANSWER (Correct) Squamous cell carcinomas of the oesophagus usually occur in the upper 2/3 of the oesophagus and are associated with GORD The development of a hoarse or husky voice is a concerning feature likely to be due to direct invasion of the aryepiglottic folds

------Dr Mohammed Shamsul Islam Khan, Medical Officer, Clinical Neuro-Surgery, National Institute of Neuro-Sciences and Hospital Sher-E-Bangla Nagar, Dhaka-1207, Bangladesh. Mobile: +880 1713 455 662, E-mail: [email protected]

MyPasTest: MRCS A Online - Jan Exam 2015 03. Anatomy; Aabdomen, Pelvis & Perineum (290Qs) ------Palliation of symptoms is seldom necessary as most patients die soon after developing dysphagia Diagnosis is usually established by a barium swallow alone.

Oesophageal carcinoma although relatively uncommon in the UK is increasing in incidence. If found in the lower 1/3 of the oesophagus, it is usually of the adenocarcinoma type and associated with GORD whilst if found in the upper 2/3 is of a similar origin as head and neck Page | cancers is of squamous cell origin and associated with smoking. 241

A hoarse voice is indeed a worrying feature but is usually due to involvement of the recurrent laryngeal nerve. Diagnosis may be suggested by a barium swallow but definitive diagnosis requires histology obtained via an OGD. Palliation is one of the most important aims of all treatments of oesophageal cancers especially when cure is not possible.

286. A 72-year-old male undergoes left nephrectomy after diagnosis with renal cell carcinoma. Which muscle lies immediately posterior to the left kidney? Single best answer - select one answer only. Erector spinae« YOUR ANSWER Iliacus Internal oblique Psoas major« CORRECT ANSWER Transversus abdominis.

The psoas major lies posterior to the kidneys bilaterally. Iliacus joins with it to act as a hip flexor. Internal oblique and transverses abdominis form part of the anterior abdominal wall, whilst erector spinae group support the trunk.

287. Concerning the uterine artery: Single best answer question – choose ONE true option only. It lies anterior and inferior to the ureter near the lateral portion of the fornix« YOUR ANSWER It lies anterior and superior to the ureter near the lateral portion of the fornix« CORRECT ANSWER The right ureter is more vulnerable because it is very close to the lateral aspect of the cervix Usually arises from the umbilical artery The point at which it crosses the ureter is 4cm superior to the ischial spine.

The uterine artery usually arises separately from the internal iliac artery, but it may also arise from the umbilical artery. The uterine artery crosses anterior and superior to the ureter near the lateral portion of the fornix. This is of surgical importance as the ureter is in danger of being damaged at the time of a hysterectomy. The point of crossing of the uterine artery and ureter is approximately 2 cm superior to the ischial spine. The left ureter is particularly vulnerable as it lies very close to the lateral aspect of the cervix.

------Dr Mohammed Shamsul Islam Khan, Medical Officer, Clinical Neuro-Surgery, National Institute of Neuro-Sciences and Hospital Sher-E-Bangla Nagar, Dhaka-1207, Bangladesh. Mobile: +880 1713 455 662, E-mail: [email protected]

MyPasTest: MRCS A Online - Jan Exam 2015 03. Anatomy; Aabdomen, Pelvis & Perineum (290Qs) ------288. A 64-year-old man is admitted with a severe haematemesis. Upper GI endoscopy identifies a posterior gastric ulcer. Which vessel is likely to be the source of the bleeding? Single best answer question – choose ONE true option only. Splenic artery« YOUR ANSWER (Correct) Left gastroepiploic artery Page | Inferior pancreaticoduodenal artery Oesophageal branch of the left gastric artery 242 Gastroduodenal.

A posterior gastric ulcer may adhere to, and ulcerate, the splenic artery as this runs along the upper border of the pancreas, resulting in massive haemorrhage. A lesser curve gastric ulcer may implicate the left gastric artery – the gastroepiploic vessels lie along the greater curve of the stomach.

A posterior duodenal ulcer may erode the gastroduodenal artery – „the ulcer of duodenal haemorrhage‟ – the inferior pancreaticoduodenal artery supplies the lower part of the second part of the duodenum, well clear of the site of ulceration. Oesophageal varices commonly extend into the upper stomach and are, of course, venous in origin.

289. A 17-year-old male is injured during a road traffic accident. During your clinical examination you notice reduced breath sounds, percussive hyperresonance and left tracheal deviation. Where should you eventually insert a chest drain? Single best answer - select one answer only. Fifth intercostal space, mid-axillary line, left chest wall« YOUR ANSWER Fifth intercostal space, mid-axillary line, right chest wall« CORRECT ANSWER Fourth intercostal space, mid-axillary line, left chest wall Fourth intercostal space, anterior axillary line, right chest wall Sixth intercostal space, anterior axillary line, left chest wall.

This patient has a tension pneumothorax and insertion of a chest drain is mandatory following needle decompression. His trachea is deviated to the left, suggesting increased pressure in the right hemithorax. Chest tubes should be inserted into the „safe zone‟, more specifically, the 5th intercostal space, mid-axillary line.

290. Your consultant shows you the porta hepatis and asks you what the most posterior stucture passing through it is. What would you respond? Single best answer - select one correct answer only. Left hepatic artery« YOUR ANSWER Left hepatic duct Portal vein« CORRECT ANSWER Right hepatic artery Right hepatic duct.

------Dr Mohammed Shamsul Islam Khan, Medical Officer, Clinical Neuro-Surgery, National Institute of Neuro-Sciences and Hospital Sher-E-Bangla Nagar, Dhaka-1207, Bangladesh. Mobile: +880 1713 455 662, E-mail: [email protected]

MyPasTest: MRCS A Online - Jan Exam 2015 03. Anatomy; Aabdomen, Pelvis & Perineum (290Qs) ------

The structures at the porta hepatis from anterior to posterior are the left and right hepatic ducts, left and right hepatic arteries and portal vein.

Courtesy: Page |

Dr Mohammed Shamsul Islam Khan 243 MBBS (CMC), FCPS-II (Neuro-Surgery) Medical Officer, Clinical Neuro-Surgery National Institute of Neuro-Sciences and Hospital Sher-E-Bangla Nagar, Dhaka-1207 Bangladesh. Mobile: +880 1713 455 662 E-mail: [email protected]

------Dr Mohammed Shamsul Islam Khan, Medical Officer, Clinical Neuro-Surgery, National Institute of Neuro-Sciences and Hospital Sher-E-Bangla Nagar, Dhaka-1207, Bangladesh. Mobile: +880 1713 455 662, E-mail: [email protected]